bstrandable NCLEX Pharmacology 2 of 5

1 tsp to ml

5 ml

tbsp to ml

15 ml

1 oz to ml

30 ml

1 cup (8 oz) to ml

240 ml

1 soda (12 oz) to ml

360 ml

1 pint to ml

480 ml

1 quart to ml

960 ml

1 mg to ng

1000 ng

1g to mg

1000 mg

1 gr to mg

60 mg

1 kg to lb

2.2 lb

1 L to kg

1 kg

Andrenergics Med Names

Norepinephrine, Dopamine, Epinephrine, Dobutamine

Andrenergics Actions

Stimulates Beta-2 Receptors in Lungs (causes brochdilation)

Andrenergics Uses

COPD, Cardiac Arrest

Andrenergics Side Effects

Dysrhythmias, Tremors, Anticholinergic Effects (dry mouth, urinary retention)

Andrenergics Nursing Considerations

Monitor B/P, Monitor Peripheral Pluses, Check Output

Antianxiety Med Names

Librium, Xanax, Ativan, Vistaril, Equanil

Antianxiety Actions

affect neurotransmitters (CNS depressent)

Antianxiety Uses

anxiety disorders, manic episodes, panic attacks

Antianxiety Side Effects

Sedatiosn, confustion, hepatic dysfunction

Benzodiazepines

-pams

Antianxiety Nursing Considerations

Potential for addiction/overdose, Avoid Alcohol, Monitor LFT

Antacids Med Names

MOM, Maalox, Amphojel

Antacids Action

neutralize gastric acids

Antacids Uses

peptic ulcer, indigestion, reflex esophagitis

Antacids Side Effects

constipation, diarrhea, acid rebound

Antacids Nursing Consideration

Interferes with absorption of antibiotics, iron preps, INH, oral contraceptives
Monitor bowel functions

Antidysrhythmics Med Names

Atropine, Lidocaine, Pronestyl, Quinidine, Isuprel

Antidysrhythmics Action

interfere with electrical exciablility of heart

Antidysrhythmics Uses

A-Fib and flutter, tachycardia, PVCs

Antidysrhythmics Side Effects

Lightheadedness, hypotension, bradycardia, urnary retention

Antidysrhythmics Nursing Consideration

Monitor VS and Cardiac Rhythm

Beta Blockers

-lol
can cause bronchspasms
airway is a consideration

Antibiotics General

Allergies, Super Infections, Organ Toxicity

Antibiotics Teaching

Take until gone
Do culture and sensitivity first
Encourage fluids
Check expiration date

Antibiotics: Aminoglycosides Med Names

Gentamycin, Neomycin, Streptomycin, Tobramycin

Antibiotics: Aminoglycosides Action

inhibits protein synthesis in gram-negative bacteria

Antibiotics: Aminoglycosides Uses

Pseudomonas, E. coli

Antibiotics: Aminoglycosides Side Effects

ototoxicity, nephrotoxicty, nausea, vomiting, diarrhea

Antibiotics: Aminoglycosides Nursing Considerations

Checking 8th cranial nerve (hearing)
Check Renal Function (BUN)
Take for 7-10 days
Encourage Fluids (3,000ml/day when pushing)

Antibiotics: Cephalosporins Med Names

Ceclor, Ancef, Keflex, Rocephin, Mefoxin

Antibiotics: Cephalosporins Actions

inhibits synthesis of bacterial cell wall

Antibiotics: Cephalosporins Uses

tonsillitis, otitis media, peri-operative prophylaxis, meningitis

Antibiotics: Cephalosporins Side Effects

Bone Marrow Depression, Superinfections, Rash

Antibiotics: Cephalosporins Nursing Consideration

Take w food
Cross allergy with PCN
Avoid alcohol
Obtain Culture and Sensitivity before first dose

Antibiotics: Aminoglycosides

-mycin

Antibiotics: Cephalosporins

Ceph-, Cef, Kef

Antibiotics: Fluoroquinolones Med Names

Cipro

Antibiotics: Fluoroquinolones Actions

interferes with DNA replication in gram-negative bacteria

Antibiotics: Fluoroquinolones Uses

E. coli, Pseudomonas, S. aureus

Antibiotics: Fluoroquinolones Side Effects

diarrhea, decreased WBC and hematocrit, elevated liver enzymes (AST ALT), elevated alkaline phosphatase

Antibiotics: Fluoroquinolones Nursing Consideration

Culture and Sensity before starting therapy
Encourage Fluids (3000ml.day)
Take 1 h before or 2 h after meals

Antibiotics: Macrolide Med Names

erthromycin, Cleocin (clindamycin)

Antibiotics: Macrolide Actions

binds to cell membrane and changes protein function

Antibiotics: Macrolide Uses

acute infections, acne, URI, prophylaxis before dental procedures if allergic to PCN

Antibiotics: Macrolide Side Effects

diarrhea, confusion, hepatotoxicity, superinfections

Antibiotics: Macrolide Nursing Consderations

Take 1 h before or 2-3 h after meals
Monitor LFT
Take w full glass of water

Antibiotics: Penicillins Med Names

amoxicillin, ampicillin, augmentin

Antibiotics: Penicillins Actions

inhibits synthesis of cell wall

Antibiotics: Penicillins Uses

moderate to severe infections, syphilis, gonococcus infections, Lyme disease

Antibiotics: Penicillins Side Effect

stomatitis, diarrhea, allergic reactions, renal and hepatic changes

Antibiotics: Penicillins Nursing Considerations

Check for hypersensitivity
Give 1-2 h before or 2-3 h after meals
Cross allergy to cephalosporins

Antibiotics: Sulfonamides Med Names

Gantrisin, Bactrim, Azulfidine

Antibiotics: Sulfonamides Actions

anatgonize essential component of folic acid synthesis

Antibiotics: Sulfonamides Uses

ulcerative colitis, Crohn's, otitis media, UTIs

Antibiotics: Sulfonamides Side Effects

peripheral neuropathy, crystalluria, photosensitivity, GI upset, stomatitis

Antibiotics: Sulfonamides Nursing Considerations

Take with meals
Encourage fluids
Good mouth care

Antibiotics: Tetracyclines Med Names

Vibramycin, Panmycin

Antibiotics: Tetracyclines Actions

inhibits protein syntheis

Antibiotics: Tetracyclines Uses

infections, acne, prophylaxis for ophthalmia neonatorum

Antibiotics: Tetracyclines Side Effects

discoloration of primary teeth if taken by the mother during pregnancy, glossitis, rash, phototoxic reaction

Antibiotics: Tetracyclines Nursing Consideration

Take 1 h before or 2-3 after meals
Do not take antacid, mile, iron
SUNBLOCK - avoid sunlight
Monitor renal function
Note Expiration Date - Toxic after date

Furadantin (nitrofurantoin) Uses

UTIs

Furadantin (nitrofurantoin) Side Effects

asthma attacks, diarrhea

Furadantin (nitrofurantoin) Nursing Considerations

Anti-infective
Give w food or milk
Monitor pulmonary status
Take w/ cranberry juice

Pyridium Uses

UTIs

Pyridium Side Effects

Headache, Vertigo, Changes urine orange

Pyridium Nursing Considerations

Taken with meals
Urinary tract analgesic

Anti Impotence (Erectile Dysfunction) Med Names

Viagra, Levitra, Cialis

Anti Impotence Side Effects

headaches, hypotension, priapism

Anti Impotence Contraindications

nitrates, alpha blockers

Anti Impotence Nursing Considerations

can not take with grapefruit juice
call PCP if erection lasting longer than 4 h

Anticholinergics Med Names

Pro-Banthine, Atropine, Scopace

Anticholinergics Actions

dilates pupil, causes bronchodialtion adn decreased secreations, decreases mobility and GI secreations

Anticholinergics Uses

opthalmic exam, motion sickness, pre-operative

Anticholinergics Side Effects

blurred vision, dry mouth, urinary retention, change in heart rate

Anticholinergics Nursing Considerations

Monitor output
Contraindicated with glaucoma
Give 30 min before meals, at bedtime or 2 hours after meals

Anticoagulant Coumadin Action

interferes with synthesis of vitamin 5-dependent clotting factors

Anticoagulant Heparin Action

blocks conversion of prothrobin to thrombin

Anticoagulant Heparin Uses

pulmonary embolism, venous thrombosis, prophylaxis after acute MI

Anticoagulant Heparin Side Effect

hematuria, tissue irritation

Anticoagulant Heparin Nursing Considerations

Monitor clotting time or PTT
Normal 20-45 sec
Therapeutic level 1.5-2.5 times control
Antagonist- protmaine sulfate
Give SC or IV - Do NOT aspirate or Massage!

Anticoagulant Coumadin Uses

pulmonary embolism, venous thrombosis, prophylaxis after acute MI

Anticoagulant Coumadin Side Effects

Hemorrhage, alopecia

Anticoagulant Coumadin Nursing Considerations

Monitor PT
Normal 9-12 sec
Therapeutic level 1.5 times control
Antagonist- Vit K
Monitor for bleeding
Give PO

Anticonvulsants Med Names

Dilantin, Luminal, Depakote, Tegretol, Klonopin

Anticonvulsants Actions

decreases flow of calcium and sodium across neuronal membranes

Anticonvulsants Uses

seizures

Anticonvulsants Side Effects

Respiratory depression, aplastic anemia, gingival hypertrophy, ataxia

Anticonvulsants Nursing Considerations

Don't d/c abruptly
Monitor I&O
Caution with use of meds that lower seizure thresholds (ex. MOAIs, antipyscotic)
No alcohol
Urine is pink/reddish

Dilantin - Anticonvulsant

Don't give too fast - cardiac arrest
NEVER mix with another med
Urine Pink

Mag Sulfate - Anticonvulsant

Common given in OB
Deep Tendon Reflexes be evaluated
SE: Resp Arrest

Depakote - Anticonvulsant

No carbonated beverages

Antidepressants: MAOIs Med Names

Marplan, Parnate, Nardil

Antidepressants: MAOIs Action

causes increased concentration of neurotransmitters

Antidepressants: MAOIs Uses

depression, chronic pain

Antidepressants: MAOIs Side Effects

hypertensive crisis when taken with tyramine-containing foods (cheese, liver, beer, wine), photosensitivity

Antidepressants: MAOIs Nursing Considerations

Avoid foods containing tyramine
Monitor Output
Takes 4 wks to work
Do not take with cold medications or CNS stimulates

Antidepressants: SSRIs Med Names

Paxil, Prozac, Zoloft

Antidepressants: SSRIs Actions

inhibits CNS uptake of serotonin

Antidepressants: SSRIs Uses

depression, obsessive compulsive disorders, bulimia

Antidepressants: SSRIs Side Effects

Anxiety, gi upset, change in appetite and bowel function, urinary retention

Antidepressants: SSRIs Nursing Considerations

suicide precautions
takes 4 weeks to fully work
take in am
turn urine pink

Hypertensive Crisis

Severe Headache
Palpitations
Diaphoretic
Stiff Neck

Antidepressants: Tricyclics Med Names

Elavil, Tofranil, Norpamin

Antidepressants: Tricyclics Actions

inhibits reuptake of neurotransmitters

Antidepressants: Tricyclics Uses

depression, sleep apnea

Antidepressants: Tricyclics Side Effects

sedation, anticholinergic effect, confusion, postural hypotension, urinary retention

Antidepressants: Tricyclics Nursing Considerations

Monitor for suicide
takes 2-6 weeks to work
take at bedtime
monitor vital signs
position slowly
wean off medication
no alcohol
no sun- use sunscreen

Antidepressants: Heterocyclics Med Names

Wellbutrin, Desyrel

Antidepressants: Heterocyclics Uses

depression and smoking cessation

Antidepressants: Heterocyclics Side Effects

Wellbutrin: agitation
Desyrel: sedation

Antidepressants: Heterocyclics Nursing Considerations

avoid alcohol
CNS depressants
wean off slowly

Insulin: Fast Acting Names

Regular - can be given IV or Pump
Humulin R

Insulin: Fast Acting Onset

.5-1h

Insulin: Fast Acting Peak

2-4h

Insulin: Fast Acting Duration

6-8h

Insulin: Intermediate Acting Names

NPH
Humulin N

Insulin: Intermediate Acting Onset

2h

Insulin: Intermediate Acting Peak

6-12h

Insulin: Intermediate Acting Duration

18-26h

Insulin: Slow Acting Names

Ultralente
Humulin U

Insulin: Slow Acting Onset

4h

Insulin: Slow Acting Peak

8-20h

Insulin: Slow Acting Duration

24-36h

Insulin: Combination Names

Humulin 70/30

Insulin: Combination Onset

.5h

Insulin: Combination Peak

2-12h

Insulin: Combination Duration

24h

Antidiabetic Agents: Med Names

Diabinese, Orinase, Dymelor, Micronase

Antidiabetic Agents: Actions

Stimulates insulin release from beta cells in pancreas

Antidiabetic Agents: Uses

Type 2 diabetes

Antidiabetic Agents: Side Effects

Hypoglycemia, allergic skin reactions, GI upset

Antidiabetic Agents: Nursing Considerations

take before breakfast
Monitor glucose levels
avoid alcohol

Hypoglycemic Agent: Med Name

GlacGen (glucagon)

Hypoglycemic Agent: Action

stimulates liver to change glycogen to glucose

Hypoglycemic Agent: Uses

Hypoglycemia

Hypoglycemic Agent: Side Effects

hypotension, bronchospasms

Hypoglycemic Agent: Nursing Considerations

May repeat in 15 min
Give carbs orally to prevent secondary hypoglycemic reactions

Antidiarrheals: Med Names

Kaopectate, Lomotil, Imodium, Paregoric

Antidiarrheals: Actions

slows peristalsis, increases tone of sphincters

Antidiarrheals: Uses

diarrhea

Antidiarrheals: Side Effects

constipation, anticholinergic effects

Antidiarrheals: Nursing Considerations

do not use with abdominal pain
monitor for urinary retention
give 2h before or 3h after other medications

Antiemetics: Med Names

Tigan, Compazine, Reglan, Antivert, Dramine

Antidiarrheals: Action

increases GI motility, blocks effect of dopamine in chemoreceptor trigger zone

Antidiarrheals: Uses

vomiting

Antidiarrheals: Side Effects

sedation, anticholinergic effects

Antidiarrheals: Nursing Considerations

Use before chemo
When used with viral infections may cause Rye's syndrome

Antifungals: Med Names

Amphotericin B, Nystatin

Antifungals: Action

impairs cell membrane

Antifungals: Uses

candidiasis, oral thrush, histoplasmosis

Antifungals: Side Effects

hepatotoxicity, throbocytopenia, leukopenia, hemorrhage, pruritus

Antifungals: Nursing Considerations

Give w food
monitor liver function
good oral hygiene

Antigout Agents: Med Names

Colchicine, Probenecid, Allopurinol

Antigout Agents: Actions

decreases production and resorption of uric acid

Antigout Agents: Uses

Gout

Antigout Agents: Side Effects

agranulocytosis, GI upset, renal calculi

Antigout Agents: Nursing Consideration

monitor kidney stones
Give w milk, food, antacids

Antihistamines: Med Names

Chlor-Trimeton, Benadryl, Phenergan

Antihistamines: Action

block effects of histamine

Antihistamines: Uses

allergic rhinitis, allergic reactions to blood

Antihistamines: Side Effects

drowsiness, dry mouth, photosensitivity, bronchospasm

Antihistamines: Nursing Considerations

give w food
use sunscreen
avoid alcohol
assess respirations

Antihyperlipidemic: Med Names

Questran, Lipitor

Antihyperlipidemic: Actions

inhibits cholesterol and triglyceride synthesis

Antihyperlipidemic: Uses

elevated cholesterol, reduce incidence of cardiovascular disease

Antihyperlipidemic: Side Effects

constipation, fat-soluble, vitamin deficiency

Antihyperlipidemic: Nursing Considerations

Take at bedtime or 30 mins after meals
Administer 1 h before or 4-6h after other meds

Antihypertensives: Ace Inhibitors Med Names

-pril (think arteries)

Antihypertensives: Ace Inhibitors Actions

blocks ACE in lungs

Antihypertensives: Ace Inhibitors Uses

hypertension, CHF

Antihypertensives: Ace Inhibitors Side Effects

GI upset, orthostatic hypotension, dizziness

Antihypertensives: Ace Inhibitors Nursing Considerations

give 1 hour before meals, or 3 hrs after
Change positioning slowly
Monitor VS

Antihypertensives: Beta Blockers Med Names

-lol (beats)

Antihypertensives: Beta Blockers Actions

blocks beta-adrenergic receptors

Antihypertensives: Beta Blockers Uses

hypertension, angina, SVT

Antihypertensives: Beta Blockers Side Effects

changes in HR, hypotension, bronchospasms

Antihypertensives: Beta Blockers Nursing Considerations

Masks signs of shock and hypoglycemia
Take with meals
Do not dc abruptly

Antihypertensives: Calcium Channel Blockers Med Names

-pine, Cardizem

Antihypertensives: Calcium Channel Blockers Actions

inhibits movement of calcium across cell membrane

Antihypertensives: Calcium Channel Blockers Uses

angina, hypertension, interstitial cystitis

Antihypertensives: Calcium Channel Blockers Side Effects

hypotension, renal failure, angioedema

Antihypertensives: Calcium Channel Blockers Nursing Considerations

monitor vs
caution position change
monitor edema (face swelling, SOB)

Antihypertensives: Angiotensin II Receptor Blockers Med Names

-sartan

Antihypertensives: Angiotensin II Receptor Blockers Action

blocks vasoconstriction and aldosterone effects of aniotensin II

Antihypertensives: Angiotensin II Receptor Blockers Uses

Hypertension, heart failure, MI, diabetic neuropathy, stroke prevention

Antihypertensives: Angiotensin II Receptor Blockers Side Effects

Hypotension, dizziness, GI distress

Antihypertensives: Angiotensin II Receptor Blockers Nursing Considerations

monitor vs
do not chew or divide sustained release tablets

Antihypertensives: Alpha 1 Adrenergic Blockers Med Names

-osin

Antihypertensives: Alpha 1 Adrenergic Blockers Actions

Selective blockade of alpha-1 reception in peripheral blood vessels

Antihypertensives: Alpha 1 Adrenergic Blockers Uses

hypertension, benign prostatic hypertrophy, reflex tachycardia

Antihypertensives: Alpha 1 Adrenergic Blockers Side Effects

reflex tachycardia, orthostac hypotension, nasal congestion

Antihypertensives: Alpha 1 Adrenergic Blockers Nursing Considerations

administer first dose bedtime to avoid fainting
Monitor BUN, weight and edema
Change position slowly

Bipolar Disorder: Med Names

Lithium (salt), Tegretol (mood stablizer), Depakote (mood stablizer)

Bipolar Disorder: Action

reduces catecholamine release

Bipolar Disorder: Uses

Manic episodes

Bipolar Disorder: Side Effects

GI upset, tremors, polydipsia, polyuria

Bipolar Disorder: Nursing Considerations

monitor serum levels
give with meals
increase fluid intake

Antineoplastic: Alkylating Agents Med Names

Cisplatin, Myleran, Cytoxan

Antineoplastic: Alkylating Agents Action

interferes with rapidly reproducing DNA

Antineoplastic: Alkylating Agents Uses

leukemia, multiple myeloma

Antineoplastic: Alkylating Agents Side Effects

hepatotoxicity, bone marrow suppression

Antineoplastic: Alkylating Agents Nurisng Considerations

Checking hematopoietic function
Force fluids
Good mouth care

Antineoplastic: Antimetabolites Med Names

5-FU, Methotrexate

Antineoplastic: Antimetabolites Actions

inhibits DNA polymerase

Antineoplastic: Antimetabolites Uses

acute lymphatic leukemia, cancer of colon, breast, pancreas

Antineoplastic: Antimetabolites Side Effects

nausea, vomiting, oral ulcerations, bone marrow suppression, alopecia

Antineoplastic: Antimetabolites Nursing Consideration

Monitor hematopoietic function
Good mouth care
Discuss body image changes

Antineoplastic: Antitumor Med Names

Adriamycin, Bleomycin, Actinomycin D

Antineoplastic: Antitumor Action

interferes with DNA and RNA sythesis

Antineoplastic: Antitumor Uses

cancer

Antineoplastic: Antitumor Side Effects

bone marrow suppression, alopecia, stomatitis

Antineoplastic: Antitumor Nursing Considerations

Monitor VS
Give antiemetic before administration

Antineoplastic: Hormonal Agents Med Names

Nolvadex, Testosterone

Antineoplastic: Hormonal Agents Actions

changes hormone input into sensitive cells

Antineoplastic: Hormonal Agents Uses

Cancer

Antineoplastic: Hormonal Agents Side Effects

leukopenia, bone pain, hypercalcemia

Antineoplastic: Hormonal Agents Nursing Considerations

Check CBC
Monitor serum calcium

Antineoplastic: Vinca Alkaloids Med Names

Velban (vinblastine), Oncovin (vincristine)

Antineoplastic: Vinca Alkaloids Action

interferes with cell division

Antineoplastic: Vinca Alkaloids Uses

cancer

Antineoplastic: Vinca Alkaloids Side Effects

stomatitis, alopecia, loss of deep tendon reflexes, bone marrow suppression

Antineoplastic: Vinca Alkaloids Nursing Considerations

Give antiemetic before administration
Check reflexes
Given with Zyloprim to decrease uric acid

Antiparkinson Agents Med Names

(-dopa) Artane, Cogentin, L-Dopa, Parlodel, Sinemet

Antiparkinson Agents Actions

converted to dopamine, stimulates postoynaptic dopamine receptors

Antiparkinson Agents Uses

Parkinson's Disease

Antiparkinson Agents Side Effects

dizziness, ataxia, atropine like effects

Antiparkinson Agents Nursing Consideration

Monitor for urinary retention
Large doeses of Vit B6 reverse effects
Avoid use of CNS depressants

Antiplatelet Agents Med Names

Aspirin, Persantine, Plavix

Antiplatelet Agents Actions

interferes with platelet aggregation

Antiplatelet Agents Uses

venous thrombosis, PE

Antiplatelet Agents Side Effects

hemorrhage, throbocytopenia

Antiplatelet Agents Nursing Considerations

Check for signs of bleeding
Give with food or milk

Antipsychotic Agents Med Names

Thorazine-low, Trilafon-medium, Haldol-high

Antipsychotic Agents Actions

blocks dopamine receptors in basal ganglia

Antipsychotic Agents Uses

acute and chronic psychoses

Antipsychotic Agents Side Effects

akathisia, dyskinesia, dystonia, parkinson's syndrome, tardive dyskinesias, leukpenia

Antipsychotic Agents Nruisng Considerations

Check CBC
Monitor VS
Avoid alcohol and caffeine

Atypical Antipsychotic Agents Med Names

Clozaril, Risperdal

Atypical Antipsychotic Agents Actions

interferes with binding of dopamine in the brain

Atypical Antipsychotic Agents Uses

Acute and chronic psychoses

Atypical Antipsychotic Agents Side Effects

extrapyramidal, anticholinergic, sedatives, ortho hypotension

Atypical Antipsychotic Agents Nursing Considerations

Monitor Blood Work
Chang position slowly
Use sunscreen
Monitor VS - Airway

Antipyretic Med Name

Tylenol

Antipyretic Action

antiprostaglandin activity in hypothalamus

Antipyretic Uses

fever

Antipyretic Side Effects

GI irritation, Liver dysfuction

Antipyretic Nursing Considerations

Monitor LFT & ALT
Aspirin contraindicated under 21 yo due to risk of Reye's syndrome

Antithyroid Agents Med Names

Tapazole, SSKI

Antithyroid Agents Action

reduce vasularity of thyroid, inbibits release of thyroid into circulation

Antithyroid Agents Uses

Hyperthroidism, myxedema

Antithyroid Agents Side Effects

leukopenia, rash, thrombocytopenia

Antithyroid Agents Nursing Considerations

Bitter Taste
May cause burning in mouth
Give with meals
Check CBC

Thyroid Replacement Agents Med Names

Synthroid, Cytomel

Thyroid Replacement Agents Action

increases metabolic rate

Thyroid Replacement Agents Uses

hypothyroidism, Graves' disease

Thyroid Replacement Agents Side Effects

nervousness, tachycardia, weight loss

Thyroid Replacement Agents Nursing Consideration

Monitor weight
Take in am
Monitor pulse and BP
Enhances antideppresants and anticouglant stronger
Decreases insulin and digoxin

Antitubercular Agents Med Names

First Line -INH, Rifampin, Ethambutol, Streptomycin
Second Line - PAS, PZA

Antitubercular Agents Action

inhibits cell and protein synthesis

Antitubercular Agents Uses

TB, to prevent disease in person exposed to organism

Antitubercular Agents Side Effects

hepatitis, peripheral neuritis

Antitubercular Agents Nursing Considerations

Check LFT
Vit B6 given for peripheral neuritis
Used in combination

Antitussive Agents Med Names

Benylin DM, Robitussin

Antitussive Agents Actions

suppress cough reflex by inhibiting cough reflex in medulla, decrease viscosity of secretions

Antitussive Agents Uses

coughs due to URI, COPD

Antitussive Agents Side Effects

bradycardia, respiratory depression, drowsiness, dizziness, anticholinergic effects

Antitussive Agents Nursing Considerations

Monitor Cough
Avoid alcohol

Antivirals Med Names

Zovirax, AZT, Videx, Famvir, Cytovene

Antivirals Action

inhibits DNA and RNA relpications

Antivirals Uses

recurrent HSV, HIV infections

Antivirals Side Effects

headache, dizziness, GI symptoms

Antivirals Nursing Considerations

Encourage fluids
Not a cure, but relieves symptoms

Attention Disorder Agents Med Names

Ritalin, Dexedrine

Attention Disorder Agents Action

increases level of catecholamines

Attention Disorder Agents Uses

ADD, narcolepsy

Attention Disorder Agents Side Effects

restlessness, insomnia, tachycardia, palitations

Attention Disorder Agents Nursing Consideration

Monitor growth rate
Monitor liver enzymes
Give in am

Bone-Reabsorption Inhibitors: Biophosphonates Med Names

Alendronate (fosamax), Risedronate (actonel), Ibandronate (boniva)

Bone-Reabsorption Inhibitors: Biophosphonates Action

inhibits resorption of bone

Bone-Reabsorption Inhibitors: Biophosphonates Uses

osteoprosis, paget's disease

Bone-Reabsorption Inhibitors: Biophosphonates Side Effects

esophagitis, arthralgia

Bronchodilators Med Names

Aminophylline, Brethine, Atrovent, Proventil, Primatene mist

Bronchodilators Action

decreases activity of phosphodiesterase

Bronchodilators Uses

COPD, preterm labor

Bronchodilators Side Effects

tachycardia, dysrhythmias, palpitations, anticholinergic effects

Bronchodilators Nursing Considerations

Monitor BP and HR
When used with steroid inhaler, use bronchodilater first
May aggravate diabetes

Carbonic Anhydrase Inhibitors Med Names

Diamox

Carbonic Anhydrase Inhibitors Action

decreases production of aqueous humor in ciliary body

Carbonic Anhydrase Inhibitors Uses

open-angle glaucoma

Carbonic Anhydrase Inhibitors Side Effects

blurred vision, lethargy, depression

Carbonic Anhydrase Inhibitors Nursing Considerations

Monitor for systemic effects

Cardiac Glycosides Med Names

Lanoxin (digoxin)

Cardiac Glycosides Action

increases force of myocardial contraction, slows rate

Cardiac Glycosides Uses

left-sided HF

Cardiac Glycosides Side Effects

bradycardia, nausea, vomiting, visual disturbances

Cardiac Glycosides Nursing Consideration

Take apical pulse
Notify physician if adult <60, child <90-110
Monitor potassium level (3.5-5)

Digoxin Therapeutic Levels

0.5-2

Digoxin Antidote

Digibind

Cytoprotective Agents Med Names

Carafate

Cytoprotective Agents Action

forms a barrier on ulcer surface

Cytoprotective Agents Uses

duodenal ulcer

Cytoprotective Agents Side Effects

constipation, vertigo

Cytoprotective Agents Nursing Considerations

Take 1h before food
Give 2h becofe or after other meds

Diuretics Med Names

HydroDIURIL, Aldactone, Lasix, Mannitol

Diuretics Actions

inhibits reabsorption of sodium and water, blocks effects of aldosterone

Diuretics Uses

CHF, renal disease

Diuretics Side Effects

dizziness, orth hypotension, leukopenia

Diuretics Nursing Considerations

Take with food or milk
Take in am
Monitor fluid and electrolytes

Electrolytes Med Names

Os-Cal, Slow Mag, K-Dur, Sodium Chloride

Electrolytes Actions

replaces needed electrolytes

Electrolytes Uses

fluid and electrolyte imbalances, osteoporosis

Electrolytes Side Effects

electrolyte imbalances

Electrolytes Nursing Considerations

Monitor fluid and electrolyte levels

Iron Preparations Med Names

Feosol (ferrous sulfate) Imferon

Iron Preparations Actions

increase availability of iron for hemoglobin

Iron Preparations Uses

iron-deficiency anemia

Iron Preparations Side Effects

constipation, dark stools, tachycardia

Iron Preparations Nursing Considerations

Take on empty stomach
Vit C increases absorption
Monitor Hct and Hgb

Glucocorticoids Med Names

Solu-Cortef (hydrocortisone) Decadron, Prednisone

Glucocorticoids Action

stimulates formation of glucose, alters immune response

Glucocorticoids Uses

Addison's disease, Crohn's disease, COPD, leukemias

Glucocorticoids Side Effects

susceptible to infection and masks infection, insomnia, hypoglycemia, hypokalemia, psychoses, depression, stunted growth

Glucocorticoids Nursing Considerations

Monitor fluid and electrolyte balance
Don't dc abruptly
Monitor for signs of infection

Immunosuppressants Med Names

Sandimmune (cyclosporine)

Immunosuppressants Action

prevents production of T cells and their response to interleukin-2

Immunosuppressants Uses

prevents rejection for transplanted organs

Immunosuppressants Side Effects

hepatotxicity, nephrotoxicity, leukonpenia, thrombovytopenia

Immunosuppressants Nursing Considerations

Take once daily in am
Used with adrenal corticosteroids
Monitor renal and liver function tests
Take w milk or juice
Stay away from sick ppl
Good Handwashing

Laxatives/Stool Softeners Med Names

Cascara, Dulcolax, Colace, MOM

Laxatives/Stool Softeners Action

absorb water increasing bulk, lubricate surface of stool, stimulate peristalsis

Laxatives/Stool Softeners Uses

constipation, preparation for sx and procedures

Laxatives/Stool Softeners Side Effects

diarrhea, dependence, abdominal cramps, hypermagnesemia

Laxatives/Stool Softeners Nursing Considerations

Contraindicated w symptoms of acute abd
Monitor fluid and electrolytes
Chronic uses may cause hypokalemia
Encourage fluid

Narcotics Med Names

Morphione, Codeine, Demerol, Dilaudid, Precodan (oxycodone)

Narcotics Action

acts on CNS receptor cells

Narcotics Uses

moderate to severe pain, preoperative, postoperative

Narcotics Side Effects

dizziness, sedation, respiratory depression, hypotension, constipation

Narcotics Nursing Considerations

Safety precautions
Avoid alcohol
Monitor VS
Narcan

Antianginals Med Names

Nitro, Isordil

Antianginals Action

relaxes smooth muscle, decreases venous return

Antianginals Uses

angina, peri-operative hypoertension, HF

Antianginals Side Effects

hypotension, tachycardia, headache, dizziness

Antianginals Nursing Considerations

Check expiration date
Teach when to take medication
May take q 5 min x 3 doses
Wet with saliva and place under tongue

NSAIDS Med Names

Motrin, Indocin, Naprosyn

NSAIDS Action

inhibits prostaglandin synthesis

NSAIDS Uses

arthritis, mild to moderate pain, fever

NSAIDS Side Effects

GI upset, dizziness, headaches, bleeding, fluid retention

NSAIDS Nursing Considerations

Take w food or after meals
Monitor liver and renal functions
Use cautiously with aspirin allergy
Check for bleeding

Thrombolytics Med Names

Strepokinase, Urokinase, Tissue Plasminogen Activator (TPA)

Thrombolytics Action

dissolves or lyses blood clots

Thrombolytics Uses

acute pulmonary emboli, thrombosis, MI

Thrombolytics Side Effects

Bleeding, bradycardia, dysrhythmias

Thrombolytics Nursing Considerations

Monitor for bleeding
Have amino caproic acid available - Antidote
Check pulse, color, sensation of extremities
Monitor EKG

H2-Receptor Blockers Med Names

Tagamet, Zantac

H2-Receptor Blockers Action

inhibits action of histamine and gastric acid secretions

H2-Receptor Blockers Uses

ulcers, gastroesophageal reflux

H2-Receptor Blockers Side Effects

dizziness, confustion, hypotension, impotence

H2-Receptor Blockers Nursing Considerations

Take w meals and hs
Smoking decreases effectiveness
Monitor LFT and CBC

-"dipine

Ca Channel blockers
HTN
angina
dysrhythmias

-"afil

erectile dysfunction

-"caine

anesthetics

-"pril

ACE inhibitor
HTN
CHF

-"pam", -"lam

benzodiazepine
anxiety
sedative
anti-convulsent

-"statin

antilipemic
lower chol.
lower triglycerides
take in the evening

-"asone, -"solone

corticosteroid
suppress inflammation

-"alol

Beta blocker
angina
HTN
Adverse Effects (4 Bs)
B- bradycardia
B- BP decreased
B- BG masked
B- Bronchoconstriction

-"cillin

penicillin

-"ide

Oral hypolglycemic

-"prazole

PPI
suppress gastric acid
antiulcer
don't give w/ food, 1-2 hrs between meals

-"vir

antiviral
HIV

-"ase" -"plase

thrombolytic
dissolve clots

-"azine

antiemetic

-"phylline

Bronchodilator

-"arin

anticoagulant

-"tidine

Histamine H2 receptor agonist
suppress secretion of gastric acid
GERD
antiulcer

-"zine

antihistamine

-"cycline

antibiotic

-"mycin

aminoglycoside

-"floxacin

antibiotic

-"tyline

Tricyclic antidepressents

-"pram", -"ine

SSRIs

-"nitr

Nitrates
angina
vasodilation

-"sulf

sulfonamides
suppress bacterial growth

-"zide

thiazide diuretics
HTN
Pulm. edema

-"thy

thyroid hormones
take in the AM

-"line

bronchodilators
COPD
bronchospasms

-"terol

Beta 2 agonist
bronchodilator
sympathomimetic

-"pium

anticholinergic
bronchodilator

-"phylline

methylxanthines
bronchodilator

-"lukast

leukotrenes
anti-inflammatory

-"crome

Mast Cells stabilizers
anit-inflammatory

Vasoconstrictors

Ex. epinephrine, ephedrine, isoprote
N- nervousness
A- anxious
S- BG increased
C- cardiac arrest
A- allergic reaction
R- resp broncodilator

-"barbitol

barbituates
CNS depressant, long acting

Norepinephrine (levofed)

given as last effort of saving life
S- stimulation of alpa/beta receptors
H- hypovolemia must be corrected prior to administration
O- output, increases urinary output
C- Constricts blood vessels
K- Keep eye on patient (monitor every 5-15 min)

Dopamine

vasoconstrictor
1st line of drug in sever hypotension

-"sartan

ARBs
Angiotensin II receptor Blockers
CHF
HTN
blocks aldosterone
avoid suanas, hot tubs
take at same time everyday

N-acetylcysteine (mucomyst)

acetaminophen antidote

physostigmine

anticholinergics antidote

chelation therapy

arsenic antidote
lead antidote
mercury antidote

flumazenil (mucomyst)

acetaminophen antidote

glucagon (high dose)

Beta blocker antidote

glucagon, CaChl, Ca gluconate

Ca Channel blockers antidote

atropine

cholinergic antidote

amyl nitrate, Na thiosulfate

cyanide antidote

digibind

digoxin antidote

protamine sulfate

anoxoparin (lovenox) antidote
heparin

ethanol

ethylene glycol antidote

glucagon

hypoglycemic agents antidote

deferoxamine mesylate

iron antidote

Ca gluconate

Mg Sulfate antidote

atropine sulfate

mushrooms, muscarinic antidote
nerve gas antidote

naloxone

opiod (morphine, codeine) antidote

antivenin, polyvalent

snake bite antidote

antivenin, equine

black widow antidote

Na bicarbonate

TCAs antidote

phytonadione (Vit K)

warfarin antidote

TPN

RN Resp:
check BG every 4hrs
high risk for infection
dressing change 48-72hrs
IV tubing/fluid change every 24hrs
substitute with 10% DW

Cardiac glycosides

digoxin
Heart failure, AFib
adverse reaction:
Halos
anorexia
N/V
Hold if HR <60 adult, <70 child, <90 infant

lispro (Humalog)

rapid acting insulin
onset <15 min.

regular (Humalin R)

short-acting
onset 0.5-1hr
only one given via IV
Clear

NPH (Humalin N)

Intermediate
Onset 1-2 hrs
Cloudy

I Glargine (Lantus)

Long Acting
Onset 1 hr
give at bedtime
don't mix with other insulin

-"zocin

peripherally acting anti-adrenergic
HTN
take at bedtime

Antihypertensives

Take med at same time every day
avoid saunas, hot tubs

Captopril

take 1hr before meals

ACE-I

stop diuretic 24-48 hrs prior to taking ACE-I
monitor K levels
A- angioedema (treat with epinephrine)
C- dry persistent cough
E- electrolyte imbalance

Ca Channel Blocker

caution w/ PT taking digoxin/beta-blockers
don't consume grapefruit juice
IV give over 2-3 minutes

Beta Blockers

do not give in same line with furosemide

Digoxin toxicity

Dc Dig and K sparring diuretics
Treat dysrhythmias with phenytoin
Bradycardia with atropine

Niratetes

life threatening if taken with Viagra
headache
keep in dark container
replace every 6 months
may be used 5-10min before exercise

Adenosine (Adenocard)

antidysrhythmic
slows conduction time through AV node
rapid IVP 1-2 sec

Amiodarone (Cordorone)

antidysrhythmic
prolongs repolarization
relaxes smooth muscle

Atropine

antidysrhythmic
increases firing of SA node & conduction through AV node

Centrally acting alpha 2 agonist

Clonidine, Guanfacine, Methyldopa
works in the brain to reduce peripheral vascular resistance
black sore tongue
leukopenia
take at bedtime

Statins

multiple drug interactions
can prolong bleeding in patients taking coumadin

Cholesterol absorption inhibitor

Ezetimibe
take 1hr before or 4hrs after another antilipemic

SSRIs (ZLPP)

Zoloft-elderly
Lexapro-
Paxil-not for elderly
Prozac-children
antidepressant
bruxism
restless leg syndrome

SNRIs

Cymbalta
caution with children and adolescents
may cause suicidality
decrease libido

TCAs

severe case of depression
Elavil
anticholinergic effects
decrease libido

MAOIs

N-nardil
P-parmate-
M-marplan
avoid foods with high tyramine

serotonin syndrome

fever,
muscle stiffness
ALOC
hallucinations

Ferrous Sulfate

Iron supplement
do not take with food, milk, antacid
take with VitC
High fiber diet to decrease constipation

M.U.C.H.

Ca Channel blockers
M-meal, give with meal
U- Under 100 systolic BP, hold
C- Ca++, needs to be monitored
H- HTN, used for HTN

-"mide

loop diuretic
High K diet

-"zides

thiazide diuretics

-"tone

K+ sparring diuretics

Mannitol

osmotic diuretic
ICP
Watch for headache

Desmporessin (DDAVP)

used for DI
reabsorbs of H2O in kidneys
watch for fluid overload

Ace Inhibitors

PRIL" Captopril, Enalapril, Afosiopril
Antihypertensive. Blocks ACE in lungs from converting angiotensin I to angiotensin II (powerful vasoconstrictor). Decreases BP, Decreased Aldosterone secretions, Sodium and fluid loss.
Check BP before giving (hypote

Beta Blockers

***
DO NOT give with ASTHMA, BRONCHIAL CONSTRICTIVE DISEASE!!!
***
"LOL" Antenolol, metoprolol. labetolol
Antihypertensive. Blocks beta-adrenergic receptors in heart, decreases excitability of the heart, reduces caridac workload and O2 consumption. Lowers

Calcium Channel Blocker

CA" Calan, Procardia, Cardizem
Slow the HR and decrease BP (check HR and BP before). Inhibits mvmt of Calcium ions across membrane or cardiac and arterial muscle cells. Results in slowed impulse conduction, depression of myocardial constractility, dilati

Diuretics

K+ and NA+ follows the walter --> electrolytes imbalances

Potassium Chloride

Check labs before giving, Never give IV PUSH
Give with FOOD (GI distresss))
Extreme caution if pt. receiving potassium-sparing diuretic.
NOT for pts with RENAL DISEASE.

Morphine

IV for chest pain
Check Respirations

Narcan

Counteracts Morphine - opioid narcotics.

Atopine (Caridiac Use)

Tx Heart Blocks (increase HR)
4D's:
Dry Mouth, Dry Stool, Dilated Pupils, Dilated Bladder (urinary retention)

Digitalis

Strengthens cardiac muscle contraction
Antiarrhythmic
CANT MIX WITH ANYTHING
TOXICITY:
Pulse (slows - Check Apical Pulse before giving)
Photosensitivity
Puke (vomiting, nausea)
Yellow-Green Halos
Light Flashes

Lidocaine

Decrease cardiac irritability
Titate IV
Use with V. TACH

Nitro

Vasodilator
Angina (1 tab 3x, 5 min apart)
Keep med dry, cool, only effective for 6 mo. (Tingling under tongue = good)
ORTHOSTATIC HYPOTENSION

Aminophylline

Bronchodilator - Decreases SOB
Therapeutic levels 10-20 mcg/dl
TOXIC > 20
CANT MIX WITH ANYTHING

Amphotericin B

Antifungal
IVPB SLOW - 2+ hrs
Hydrate pt - very toxic to kidneys, monitor BUN.
Check Hearing
Check K+ levels.

Aminoglycosides

Gentamycin, Kanamycin sulfate, Neomycin sulate, Steptomycin Sulfate
OTOTOXICITY
KIDNEY DAMAGE

Bronchodilators

Epinephrine (adrenalin)
Careful with Cardiac Pts

Anticoagulants

Heparin (Antecdote - Protamine Sulfate)
Coumadin (Antecdote - Vit K.)
Bleeding precautions
PPT - Heparin
PT/INR - Coumadin

Anticholinergics

Atropine
GI - Slows motility, spasm
Eyes - Dilates pupils
DO NOT GIVE TO GLAUCOME PTS
Heart - Increase HR
Resp - bronchodilator (Atrovent)

Cancer Drugs

Wear gloves, mask when preparing or giving. Can be absorbed throught skin

Sulfa Drugs

Drink lots of fluids
Can cause blood dyscrasias - REPORT SORE THROAT, RASH IMMEDIATELY.
UTI (Bactrim, sulfisoxazole)
Ulcerative colitis (Sulfasalazine)

Urinary Antispasmodics

Pyridium (cystitis) - Orange/Red urine

Penicillines

Take all 10 days

Cortisone (Steroids)

Wt gain, Na+ retention, K+ Loss
NEVER STOP SUDDENLY

Dilantin

Anti-convolsant
Therapeutic level 10-20 (same as amminophylline)
Toxicity > 20
Causes hyperplasia of gums - bleeding gums
Give extra VIT. D

Tylenol

Bad for Liver - DO NOT GIVE TO ALCOHOLICS

Premarin

1. Hormone Replacement - Estrogen
Tx Menopause Sxs
SD - Vascular Headaches
Periods continue
2. Prempro (comination estrogen and progesteron)
No periods

Allupurinol (Zyloprim)

G - Gulp 3L fluid per day
O - (NO) Organ Meats
U - Output up to 2 L/day
T - Teach

Miotics

CONSTRICT (Pilo-Carpine)
For glaucoma - increased outflow of aqueaous humor

Mydriatics

DILATES (Atropine)
Dont use in pts with gaucoma (build up IOP)

Antacids

Aluminum - Constipation
Magnesium - Diarrhea

Lithium

Bipolar Disorder
Decrease Na+ (risk of toxicity)
Increase Na+ levels (causes sodium diuresis)
Toxic Level > 2 mEq/L

Librium and Valume

To prevent DT's in acute alcohol withdrwal
Can produce physical dependency.

Direct - Acting Vasodilator

Hydralazine (apresoline)
Relaxes smooth muscle of blood vessels, lowing peripheral esistance.
Teach pt to check pulse d/t change in C.O.
If HR <60 Contact physician.

Psychomotor symptoms

Anticholinergic effects are common adverse effects of antidepressants such as imipramine (Tofranil). These effects may include:
a. Psychomotor symptoms
b. Tachycardia, hypertension, and increase in respiratory rate
c. Tardive dyskinesias
d. Blurred vision

TCA antidepressants

Psychomotor symptoms, tachycardia, hypertension, increase in respiratory rate, and tardive dyskinesias are potential adverse effects of ______?

Decrease drug dependence and assess status

The parents of a patient receiving methylphenidate (Ritalin) express concern that the health care provider has suggested the child have a "holiday" from the drug. The nurse explains that the drug-free holiday is designed to:
a. Reduce the risk of drug tox

drug holiday

The ____ is to decrease the risk of dependence and to evaluate behavior.

An increased susceptibility to infections

A 17-year-old male has started valproic acid (Depakene) for treatment of bipolar disorder. While he is taking this drug, he should be carefully monitored for:
a. Unusual abdominal pain, especially in the upper quadrant areas
b. An increased susceptibility

St. John's wort

Taking ____ with an MAOI could result in hypertensive crisis; patients should always consult with their health care provider before taking any medications or OTC drugs/herbal remedies.

Read labels of food and over-the-counter drugs.

Which of the following would be a priority component of the teaching plan for a patient prescribed * phenelzine (Nardil) for treatment of depression?
a. Headache may occur.
b. Hyperglycemia may occur.
c. Read labels of food and over-the-counter drugs.
d.

MAOI

Nardil is an ____. This class of drugs has many drug and food interactions that may cause a hypertensive crisis.

A month or longer

A patient experiencing moderate depression is placed on sertraline (Zoloft). The nurse should counsel the patient to expect full effects from the drug in:
a. 2-3 days
b. 1 week
c. A month or longer
d. Within 24 hours after starting the drug

SSRIs

Antidepressant drugs such as the ____ may not have full effects for a month or longer but some improvement in mood and depression should be noticeable after beginning therapy. DRUG OF CHOICE for depression.

depression

Lack of energy, sleep disturbances, abnormal eating patterns
( OVER EATING) Feelings of despair, guilt, hopelessness
Most common mental health disorder of elderly adults. Is associated w/ dysfunction of neurotransmitters in the brain connected with cognit

Postpartum Depression

Intense mood changes associated with hormonal changes

BIPOLAR DISORDER

once known as Manic depression- Shift from emotions of extreme depression to extreme rage & agitation.

suicidal ideation

Always ask pt about this before giving antidepressant. Children at greatest risk for this.

Seasonal Affective Disorder (SAD)

Enhanced release of melatonin due to lower light levels

Give Ritalin in the morning. do not give Ritalin after 4pm.

Do not give Ritalin after ?

SSRIs

1st line of defense against depression

lithium (Eskalith)

Medication use for Mania symptoms of bipolar

Lithium

Must have salt or it will cause retention of this drug? patients should not avoid salt when taking this drug because it reduces its effects. salt is needed.

electroconvulsive therapy (ECT)

therapy for *severely depressed patients in which a brief electric current is sent through the brain. serious complications are seizure activity and anesthesia

TCAs

Block or inhibit the reuptake of NE and serotonin (or 5-HT) Are less dangerous than MAOI's. Sedation is a reported complaint at beginning of therapy.

repetitive transcranial magnetic stimulation (rTMS)

this treatment requires surgical implant of device. somatic treatment

black box warning

2004 FDA issued this. To be included at beginning of drug package inserts and drug information sheets. These warnings apply especially to children, who are at greater risk for suicidal ideation.

TCA's

atypical use of ____ is for the treatment of enuresis (bed wetting)

Clomipramine (Anafranil)

are used to treat obsessive compulsive behavior. (OCD)

paroxetine, Paxil

Antidepressant. Weight gain. pack it on ____

Zoloft

Selective serotonin reuptake inhibitor (SSRI) drug.can reduce your sex drive. its been nicknamed "so soft".

venlafaxine, Effexor

SNRI. an atypical antidepressant; off labeled use for hot flashes associated with menopause.

SSRI

These should not be started within 14 days of taking MAOI. use with a MAOI can cause serotonin sydrome (autonomic hyperactivity, hyperthermia, rigidity, diaphoresis, and neuroleptic malignant syndrome, hypertensive crisis.

Serotonin syndrome

may occur when taking another drug that affects reuptake of serotonin, causing sertonin to accumulate in the body. Can also be caused from SSRI with MAOI. Symptoms include confusion, anxiety, restlessness, hyperpyrexia, ataxia.

Tyramine

rich in wines and cheese, yogart, beef or chicken liver; these foods are contraindicated in patients on MAO inhibitors

Diarrhea and ataxia
Hypotension and edema
Slurred speech and muscle weakness

Which of the following symptoms would indicate to the
nurse that a patient is experiencing lithium toxicity? (Select
all that apply.)
1. Diarrhea and ataxia
2. Hypotension and edema
3. Hypertension and dehydration
4. Increased appetite, increased energy,

lithium toxicity

dehydration can lead to?

b. 0.5 to 2.0 ng/mL

A newly admitted client takes digoxin 0.25 mg/day. The nurse knows that which is the serum therapeutic range for digoxin?
a. 0.1 to 1.5 ng/mL
b. 0.5 to 2.0 ng/mL
c. 1.0 to 2.5 ng/mL
d. 2.0 to 4.0 ng/mL

a. It is in the high (elevated) range.

The client's serum digoxin level is 3.0 ng/mL. What does the nurse know about this serum digoxin level?
a. It is in the high (elevated) range.
b. It is in the low (decreased) range.
c. It is within the normal range.
d. It is in the low average range.

d. Pulse below 60 beats/min and irregular rate

The nurse is assessing the client for possible evidence of digitalis toxicity. The nurse acknowledges that which is included in the signs and symptoms for digitalis toxicity?
a. Pulse (heart) rate of 100 beats/min
b. Pulse of 72 with an irregular rate
c.

a. Increase the serum digoxin sensitivity level

The client is also taking a diuretic that decreases her potassium level. The nurse expects that a low potassium level (hypokalemia) could have what effect on the digoxin?
a. Increase the serum digoxin sensitivity level
b. Decrease the serum digoxin sensit

b. Headaches

When a client first takes a nitrate, the nurse expects which symptom that often occurs?
a. Nausea and vomiting
b. Headaches
c. Stomach cramps
d. Irregular pulse rate

c. Decrease heart rate and decrease myocardial contractility.

The nurse acknowledges that beta blockers are as effective as antianginals because they do what?
a. Increase oxygen to the systemic circulation.
b. Maintain heart rate and blood pressure.
c. Decrease heart rate and decrease myocardial contractility.
d. De

b. The beta blocker should NOT be abruptly stopped; the dose should be tapered down.

The health care provider is planning to discontinue a client's beta blocker. What instruction should the nurse give the client regarding the beta blocker?
a. The beta blocker should be abruptly stopped when another cardiac drug is prescribed.
b. The beta

c. To block the beta1-adrenergic receptors in the cardiac tissues

The beta blocker acebutolol (Sectral) is prescribed for dysrhythmias. The nurse knows that what is the primary purpose of the drug?
a. To increase the beta1 and beta2 receptors in the cardiac tissues
b. To increase the flow of oxygen to the cardiac tissue

a. "Apply the patch to a nonhairy area of the upper torso or arm.

A client is to be discharged home with a transdermal nitroglycerin patch. Which instruction will the nurse include in the client's teaching plan?
a. "Apply the patch to a nonhairy area of the upper torso or arm."
b. "Apply the patch to the same site each

d. Client stating that pain is 0 out of 10

A nurse is monitoring a client with angina for therapeutic effects of nitroglycerin. Which assessment finding indicates that the nitroglycerin has been effective?
a. Blood pressure 120/80 mm Hg
b. Heart rate 70 beats per minute
c. ECG without evidence of

d. Chest pain

The nurse is monitoring a client during IV nitroglycerin infusion. Which assessment finding will cause the nurse to take action?
a. Blood pressure 110/90 mm Hg
b. Flushing
c. Headache
d. Chest pain

d. "I can take up to five tablets at 3-minute intervals for chest pain if necessary.

Which statement made by the client demonstrates a need for further instruction regarding the use of nitroglycerin?
a. "If I get a headache, I should keep taking nitroglycerin and use Tylenol for pain relief."
b. "I should keep my nitroglycerin in a cool,

a. Client states that she has no chest pain.

Which client assessment would assist the nurse in evaluating therapeutic effects of a calcium channel blocker?
a. Client states that she has no chest pain.
b. Client states that the swelling in her feet is reduced.
c. Client states the she does not feel d

d. "This medication will work for 24 hours and you will need to change the patch daily.

What statement is the most important for the nurse to include in the teaching plan for a client who has started on a transdermal nitroglycerin patch?
a. "This medication works faster than sublingual nitroglycerin works."
b. "This medication is the stronge

c. Apply the nitroglycerin patch for 14 hours and remove it for 10 hours at night.

What will the nurse instruct the client to do to prevent the development of tolerance to nitroglycerin?
a. Apply the nitroglycerin patch every other day.
b. Switch to sublingual nitroglycerin when the client's systolic blood pressure elevates to more than

c. Assess blood pressure.

Before the nurse administers isosorbide mononitrate (Imdur), what is a priority nursing assessment?
a. Assess serum electrolytes.
b. Measure blood urea nitrogen and creatinine.
c. Assess blood pressure.
d. Monitor level of consciousness.

b. "It's best to keep it in its original container away from heat and light.

The client asks the nurse how nitroglycerin should be stored while traveling. What is the nurse's best response?
a. "You can protect it from heat by placing the bottle in an ice chest."
b. "It's best to keep it in its original container away from heat and

d. "I should sit or lie down after I take a nitroglycerin tablet to prevent dizziness.

Which statement indicates to the nurse that the client understands sublingual nitroglycerin medication instructions?
a. "I will take up to five doses every 3 minutes for chest pain."
b. "I can chew the tablet for the quickest effect."
c. "I will keep the

b. Apply the ointment to a nonhairy part of the upper torso.

What instruction should the nurse provide to the client who needs to apply nitroglycerin ointment?
a. Use the fingers to spread the ointment evenly over a 3-inch area.
b. Apply the ointment to a nonhairy part of the upper torso.
c. Massage the ointment in

b. Decrease the intravenous nitroglycerin by 10 mcg/min.

A client receiving intravenous nitroglycerin at 20 mcg/min complains of dizziness. Nursing assessment reveals a blood pressure of 85/40 mm Hg, heart rate of 110 beats/min, and respiratory rate of 16 breaths/min. What is the nurse's priority action?
a. Ass

b. Heart rate 58 beats per minute

The nurse is monitoring a client taking digoxin (Lanoxin) for treatment of heart failure. Which assessment finding indicates a therapeutic effect of the drug?
a. Heart rate 110 beats per minute
b. Heart rate 58 beats per minute
c. Urinary output 40 mL/hr

a. Administer ordered dose of digoxin.

A client's serum digoxin level is drawn, and it is 0.4 ng/mL. What is the nurse's priority action?
a. Administer ordered dose of digoxin.
b. Hold future digoxin doses.
c. Administer potassium.
d. Call the health care provider.

a. Evaluate digoxin levels.

A client is taking digoxin (Lanoxin) 0.25 mg and furosemide (Lasix) 40 mg. When the nurse enters the room, the client states, "There are yellow halos around the lights." Which action will the nurse take?
a. Evaluate digoxin levels.
b. Withhold the furosem

a. Loss of appetite with slight bradycardia

Which assessment finding will alert the nurse to suspect early digitalis toxicity?
a. Loss of appetite with slight bradycardia
b. Blood pressure 90/60 mm Hg
c. Heart rate 110 beats per minute
d. Confusion and diarrhea

b. To administer digoxin immune FAB

The nurse reviews a client's laboratory values and finds a digoxin level of 10 ng/mL and a serum potassium level of 5.9 mEq/L. What is the nurse's primary intervention?
a. To administer atropine
b. To administer digoxin immune FAB
c. To administer epineph

c. Monitor blood pressure continuously.

A client is to begin treatment for short-term management of heart failure with milrinone lactate (Primacor). What is the priority nursing action?
a. Administer digoxin via IV infusion with the Primacor.
b. Administer Lasix (furosemide) via IV infusion aft

c. Continue to monitor the client.

A client's recently drawn serum lidocaine drug level is 3.0 mcg/mL. What is the nurse's priority intervention?
a. Increase the lidocaine infusion.
b. Decrease the lidocaine infusion.
c. Continue to monitor the client.
d. Stop the IV drip for 1 hour.

c. Rapid IV bolus of Adenosine (Adenocard)

A client is admitted to the emergency department with paroxysmal supraventricular tachycardia. What intervention is the nurse's priority?
a. Administration of digoxin IV push
b. Administration of oxygen, 2 lpm
c. Rapid IV bolus of Adenosine (Adenocard)
d.

c. ECG

A nurse is caring for a client who has been started on ibutilide (Corvert). Which assessment is a priority for this client?
a. Blood pressure measurement
b. BUN and creatinine
c. ECG
d. Lung sounds

b. Crackles in the lungs

Which assessment finding will alert the nurse to possible toxic effects of amiodarone?
a. Heart rate 100 beats per minute
b. Crackles in the lungs
c. Elevated blood urea nitrogen
d. Decreased hemoglobin

b. Continuous blood pressures
d. Presence of chest pain

What must the nurse monitor when titrating intravenous nitroglycerin for a client? (Select all that apply.)
a. Continuous oxygen saturation
b. Continuous blood pressures
c. Hourly ECGs
d. Presence of chest pain
e. Serum nitroglycerin levels
f. Visual acui

b. Hypokalemia

A client is taking hydrochlorothiazide 50 mg/day and digoxin 0.25 mg/day. What type of electrolyte imbalance does the nurse expect to occur?
a. Hypocalcemia
b. Hypokalemia
c. Hyperkalemia
d. Hypermagnesemia

c. Hydrochlorothiazide

What would cause the same client's electrolyte imbalance?
a. High dose of digoxin
b. Digoxin taken daily
c. Hydrochlorothiazide
d. Low dose of hydrochlorothiaizde

d. Serum glucose (sugar)

A nurse teaching a client who has diabetes mellitus and is taking hydrochlorothiazide 50 mg/day. The teaching should include the importance of monitoring which levels?
a. Hemoglobin and hematocrit
b. Blood urea nitrogen (BUN)
c. Arterial blood gases
d. Se

c. High-ceiling (loop) diuretic

A client has heart failure and is prescribed Lasix. The nurse is aware that furosemide (Lasix) is what kind of drug?
a. Thiazide diuretic
b. Osmotic diuretic
c. High-ceiling (loop) diuretic
d. Potassium-sparing diuretic

a. Hypokalemia

The nurse acknowledges that which condition could occur when taking furosemide?
a. Hypokalemia
b. Hyperkalemia
c. Hypoglycemia
d. Hypermagnesemia

b. To increase the serum potassium level

For the client taking a diuretic, a combination such as triamterene and hydrochlorothiazide may be prescribed. The nurse realizes that this combination is ordered for which purpose?
a. To decrease the serum potassium level
b. To increase the serum potassi

b. Hyperkalemia

The client has been receiving spironolactone (Aldactone) 50 mg/day for heart failure. The nurse should closely monitor the client for which condition?
a. Hypokalemia
b. Hyperkalemia
c. Hypoglycemia
d. Hypermagnesemia

a. Have the client lie down when taking a nitroglycerin sublingual tablet.
b. Teach client to repeat taking a tablet in 5 minutes if chest pain persists.
e. Warn client against ingesting alcohol while taking nitroglycerin.

A client who has angina is prescribed nitroglycerin. The nurse reviews which appropriate nursing interventions for nitroglycerin (Select all that apply.)
a. Have the client lie down when taking a nitroglycerin sublingual tablet.
b. Teach client to repeat

b. Fasting blood glucose level of 140 mg/dL

Which laboratory value will the nurse report to the health care provider as a potential adverse response to hydrochlorothiazide (HydroDIURIL)?
a. Sodium level of 140 mEq/L
b. Fasting blood glucose level of 140 mg/dL
c. Calcium level of 9 mg/dL
d. Chloride

b. "This combination promotes diuresis but decreases the risk of hypokalemia.

What is the best information for the nurse to provide to the client who is receiving spironolactone (Aldactone) and furosemide (Lasix) therapy?
a. "Moderate doses of two different diuretics are more effective than a large dose of one."
b. "This combinatio

c. Administer 2 mEq potassium chloride per kilogram per day IV.

The nurse is assessing a client who is taking furosemide (Lasix). The client's potassium level is 3.4 mEq/L, chloride is 90 mmol/L, and sodium is 140 mEq/L. What is the nurse's primary intervention?
a. Mix 40 mEq of potassium in 250 mL D5W and infuse rapi

c. The fact that Lasix has shown efficacy in treating persons with renal insufficiency.

A nurse admits a client diagnosed with pneumonia. The client has a history of chronic renal insufficiency, and the health care provider orders furosemide (Lasix) 40 mg twice a day. What is most important to include in the teaching plan for this client?
a.

c. Fish

A client taking spironolactone (Aldactone) has been taught about the therapy. Which menu selection indicates that the client understands teaching related to this medication?
a. Apricots
b. Bananas
c. Fish
d. Strawberries

c. A 47-year-old client with anuria

Which client would the nurse need to assess first if the client is receiving mannitol (Osmitrol)?
a. A 67-year-old client with type 1 diabetes mellitus
b. A 21-year-old client with a head injury
c. A 47-year-old client with anuria
d. A 55-year-old client

c. A decrease in arterial pH

A nurse is caring for a client receiving acetazolamide (Diamox). Which assessment finding will require immediate nursing intervention?
a. A decrease in bicarbonate level
b. An increase in urinary output
c. A decrease in arterial pH
d. An increase in PaO2

b. Assess lung sounds before and after administration.
c. Assess blood pressure before and after administration.
d. Maintain accurate intake and output record.

A client is ordered furosemide (Lasix) to be given via intravenous push. What interventions should the nurse perform? (Select all that apply.)
a. Administer at a rate no faster than 20 mg/min.
b. Assess lung sounds before and after administration.
c. Asse

d. "Wear protective clothing and sunscreen while on this medication.

A client is prescribed Thalitone (chlorthalidone). What is the most important information the nurse should teach the client?
a. "Do not drink more than 10 ounces of fluid a day while on this medication."
b. "Take this medication on an empty stomach."
c.

c. Decreased aldosterone

A client with hyperaldosteronism is prescribed spironolactone (Aldactone). What assessment finding would the nurse evaluate as a positive outcome?
a. Decreased potassium level
b. Decreased crackles in the lung bases
c. Decreased aldosterone
d. Decreased a

c. Lungs clear.

A client with acute pulmonary edema receives furosemide (Lasix). What assessment finding indicates that the intervention is working?
a. Potassium level decreased from 4.5 to 3.5 mEq/L.
b. Crackles auscultated in the bases.
c. Lungs clear.
d. Output 30 mL/

a. Decreased intracranial pressure

Which assessment indicates a therapeutic effect of mannitol (Osmitrol)?
a. Decreased intracranial pressure
b. Decreased potassium
c. Increased urine osmolality
d. Decreased serum osmolality

c. Assess potassium levels.

Which intervention will the nurse perform when monitoring a client receiving triamterene (Dyrenium)?
a. Assess urinary output hourly.
b. Monitor for side effect of hypoglycemia.
c. Assess potassium levels.
d. Monitor for Hypernatremia.

a. It causes an alkaline urine, which facilitates the elimination of uric acid.

The client asks the nurse why the health care provider prescribed acetazolamide (Diamox), a diuretic, to treat gout. What is the nurse's best response?
a. It causes an alkaline urine, which facilitates the elimination of uric acid.
b. It increases alkalin

c. Stage 1 hypertension

A client's blood pressure (BP) is 145/90. According to the guidelines for determining hypertension, the nurse realizes that the client's BP is at which stage?
a. Normal
b. Prehypertension
c. Stage 1 hypertension
d. Stage 2 hypertension

a. Diuretic

The nurse acknowledges that the first-line drug for treating this client's blood pressure might be which drug?
a. Diuretic
b. Alpha blocker
c. ACE inhibitor
d. Alpha/beta blocker

c. Beta blockers and ACE inhibitors

The nurse is aware that which group(s) of antihypertensive drugs are less effective in African-American clients?
a. Diuretics
b. Calcium channel blockers and vasodilators
c. Beta blockers and ACE inhibitors
d. Alpha blockers

b. hydrochlorothiazide

The nurse knows that which diuretic is most frequently combined with an antihypertensive drug?
a. chlorthalidone
b. hydrochlorothiazide
c. bendroflumethiazide
d. potassium-sparing diuretic

a. Beta1 blocker

The nurse explains that which beta blocker category is preferred for treating hypertension?
a. Beta1 blocker
b. Beta2 blocker
c. Beta1 and beta2 blockers
d. Beta2 and beta3 blockers

d. Constant, irritating cough

Captopril (Capoten) has been ordered for a client. The nurse teaches the client that ACE inhibitors have which common side effects?
a. Nausea and vomiting
b. Dizziness and headaches
c. Upset stomach
d. Constant, irritating cough

b. Blocking angiotensin II from AT1 receptors

A client is prescribed losartan (Cozaar). The nurse teaches the client that an angiotensin II receptor blocker (ARB) acts by doing what?
a. Inhibiting angiotensin-converting enzyme
b. Blocking angiotensin II from AT1 receptors
c. Preventing the release of

b. Dizziness
c. Headache
e. Ankle edema

During an admission assessment, the client states that she takes amlodipine (Norvasc). The nurse wishes to determine whether or not the client has any common side effects of a calcium channel blocker. The nurse asks the client if she has which signs and s

a. "I will check my blood pressure daily and take my medication when it is over 140/90.

Which statement indicates that the client needs additional instruction about antihypertensive treatment?
a. "I will check my blood pressure daily and take my medication when it is over 140/90."
b. "I will include rest periods during the day to help me tol

a. Call the health care provider to switch the medication.

A nurse is caring for a client who is taking an angiotensin-converting enzyme inhibitor and develops a dry, nonproductive cough. What is the nurse's priority action?
a. Call the health care provider to switch the medication.
b. Assess the client for other

d. spironolactone (Aldactone)

The nurse is reviewing a medication history on a client taking an ACE inhibitor. The nurse plans to contact the health care provider if the client is also taking which medication?
a. docusate sodium (Colace)
b. furosemide (Lasix)
c. morphine sulfate
d. sp

b. Respiratory assessment

A client is prescribed a noncardioselective beta1 blocker. What nursing intervention is a priority for this client?
a. Assessment of blood glucose levels
b. Respiratory assessment
c. Orthostatic blood pressure assessment
d. Teaching about potential tachyc

c. The client who has stopped taking a beta blocker due to cost.

Which client will the nurse assess first?
a. The client who has been on beta blockers for 1 day.
b. The client who is on a beta blocker and a thiazide diuretic.
c. The client who has stopped taking a beta blocker due to cost.
d. The client who is taking a

d. Get up slowly from a sitting to a standing position.

The nurse is caring for a client with hypertension who is prescribed Clonidine transdermal preparation. What is the correct information to teach this client?
a. Change the patch daily at the same time.
b. Remove the patch before taking a shower or bath.
c

b. Notify the health care provider.

The client taking Methyldopa (Aldomet) has elevated liver function tests. What is the nurse's best action?
a. Document the finding and continue care.
b. Notify the health care provider.
c. Immediately stop the medication.
d. Change the client's diet.

c. Determine the client's history.

A client taking prazosin has a blood pressure of 140/90. The client is complaining of swollen feet. What is the nurse's best action?
a. Hold the medication.
b. Call the health care provider.
c. Determine the client's history.
d. Weigh the client.

c. Hypotension

A calcium channel blocker has been ordered for a client. Which condition in the client's history is a contraindication to this medication?
a. Hypokalemia
b. Dysrhythmias
c. Hypotension
d. Increased intracranial pressure

b. "Increasing fluid and fiber in your diet can help prevent the side effect of constipation.

A client who takes clonidine (Catapres) is to be discharged to home. Which instruction will the nurse include when teaching this client?
a. "Your blood pressure should be checked by a health care provider at least once a year."
b. "Increasing fluid and fi

c. To administer phentolamine (Regitine)

During assessment of a client diagnosed with pheochromocytoma, the nurse auscultates a blood pressure of 210/110 mm Hg. What is the nurse's best action?
a. To ask the client to lie down and rest
b. To assess the client?s dietary intake of sodium and fluid

a. Alteration in cardiac output related to effects on the sympathetic nervous system

Which is a priority nursing diagnosis for a client taking an antihypertensive medication?
a. Alteration in cardiac output related to effects on the sympathetic nervous system
b. Knowledge deficit related to medication regimen
c. Fatigue related to side ef

a. Coronary thrombosis
b. Acute myocardial infarction
c. Deep vein thrombosis (DVT)
d. Cerebrovascular accident (CVA) (stroke)
e. Venous disorders

When a newly admitted client is placed on heparin, the nurse acknowledges that heparin is effective for preventing new clot formation in clients who have which disorder(s)? (Select all that apply.)
a. Coronary thrombosis
b. Acute myocardial infarction
c.

a. protamine sulfate

A client who received heparin begins to bleed, and the physician calls for the antidote. The nurse knows that which is the antidote for heparin?
a. protamine sulfate
b. vitamin K
c. aminocaproic acid
d. vitamin C

a. A longer half-life than heparin

A client is prescribed enoxaparin (Lovenox). The nurse knows that low-molecular-weight heparin (LMWH) has what kind of half-life?
a. A longer half-life than heparin
b. A shorter half-life than heparin
c. The same half-life as heparin
d. A four-times short

c. Bleeding may increase when taken with aspirin.

The nurse is teaching a client about clopidogrel (Plavix). What is important information to include?
a. Constipation may occur.
b. Hypotension may occur.
c. Bleeding may increase when taken with aspirin.
d. Normal dose is 25 mg tablet per day.

d. Subcutaneously

A client is prescribed dalteparin (Fragmin). LMWH is administered via which route?
a. Intravenously
b. Intramuscularly
c. Intradermally
d. Subcutaneously

b. warfarin (Coumadin)

A client is being changed from an injectable anticoagulant to an oral anticoagulant. Which anticoagulant does the nurse realize is administered orally?
a. enoxaparin sodium (Lovenox)
b. warfarin (Coumadin)
c. bivalirudin (Angiomax)
d. lepirudin (Refludan)

b. Elevated INR range

A client is taking warfarin 5 mg/day for atrial fibrillation. The client's international normalized ration (INR) is 3.8. The nurse would consider the INR to be what?
a. Within normal range
b. Elevated INR range
c. Low INR range
d. Low average INR range

d. To suppress platelet aggregation

Cilostazol (Pletal) is being prescribed for a client with coronary artery disease. The nurse knows that which is the major purpose for antiplatelet drug therapy?
a. To dissolve the blood clot
b. To decrease tissue necrosis
c. To inhibit hepatic synthesis

b. abciximab (ReoPro)

A client is to undergo a coronary angioplasty. The nurse acknowledges that which drug is used primarily for preventing reocclusion of coronary arteries following a coronary angioplasty?
a. clopidogrel (Plavix)
b. abciximab (ReoPro)
c. warfarin (Coumadin)

c. Thrombolytic agent

A client is admitted to the emergency department with an acute myocardial infarction. Which drug category does the nurse expect to be given to the client early for the prevention of tissue necrosis following blood clot blockage in a coronary or cerebral a

b. Activated partial thromboplastin time (aPTT) of 120 seconds

A client is receiving an intravenous heparin drip. Which laboratory value will require immediate action by the nurse?
a. Platelet count of 150,000
b. Activated partial thromboplastin time (aPTT) of 120 seconds
c. INR of 1.0
d. Blood urea nitrogen (BUN) le

b. Administer vitamin K.

A client who has been taking warfarin (Coumadin) is admitted with coffee-ground emesis. What is the nurse's primary action?
a. Administer vitamin E.
b. Administer vitamin K.
c. Administer protamine sulfate.
d. Administer calcium gluconate.

a. Administer an additional dose of warfarin (Coumadin).

The client has an international normalized ratio (INR) value of 1.5. What action will the nurse take?
a. Administer an additional dose of warfarin (Coumadin).
b. Hold the next dose of warfarin (Coumadin).
c. Increase the heparin drip rate.
d. Administer p

c. "I will increase dark-green, leafy vegetables in my diet.

A client is receiving warfarin (Coumadin) for a chronic condition. Which client statement requires immediate action by the nurse?
a. "I will avoid contact sports."
b. "I will take my medication in the early evening each day."
c. "I will increase dark-gree

a. "I take aspirin daily for headaches.

A client is taking enoxaparin (Lovenox) daily. Which client statement requires additional monitoring?
a. "I take aspirin daily for headaches."
b. "I take ibuprofen (Motrin) at least once a week for joint pain."
c. "Whenever I have a fever, I take acetamin

b. Weigh the client before administration.

The client is receiving tirofiban (Aggrastat). What is an essential nursing intervention for this client?
a. Have protamine sulfate available in case of an overdose.
b. Weigh the client before administration.
c. Have vitamin K available in case of an over

c. Administer the medication into subcutaneous tissue.

A nurse is preparing to administer enoxaparin sodium (Lovenox) to a client for prevention of deep vein thrombosis. What is an essential nursing intervention?
a. Draw up the medication in a syringe with a 22-gauge, 1-� inch needle.
b. Utilize the Z-track m

b. "Dalteparin is a low-molecular-weight heparin that is more predictable in its effect and has a lower risk of bleeding.

The client asks what the difference is between dalteparin (Fragmin) and heparin. What is the nurse's best response?
a. "There is no real difference. Dalteparin is preferred because it is less expensive."
b. "Dalteparin is a low-molecular-weight heparin th

b. Administer protamine sulfate.

A client has been admitted through the emergency department and requires emergency surgery. The client has been receiving heparin. What nursing intervention is essential?
a. Teach the client about the phenytoin.
b. Administer protamine sulfate.
c. Assess

a. Assess for reperfusion dysrhythmias.

What nursing intervention is essential for the client receiving alteplase?
a. Assess for reperfusion dysrhythmias.
b. Monitor liver enzymes.
c. Administer vitamin K if bruising is observed.
d. Monitor blood pressure and stop the medication if blood pressu

b. Teach the client of potential drug interactions with anticoagulants.

A client who is taking warfarin (Coumadin) requests an aspirin for headache relief. What is the nurse's best response?
a. Administer 650 mg of acetylsalicylic acid (ASA) and reassess pain in 30 minutes.
b. Teach the client of potential drug interactions w

b. "It usually takes about 3 days to achieve a therapeutic effect for warfarin, so the heparin is continued until the warfarin is therapeutic.

A client is started on warfarin (Coumadin) therapy while still receiving intravenous heparin. The client questions the nurse about the risk for bleeding. How should the nurse respond?
a. "Your concern is valid. I will call the doctor to discontinue the he

d. "I should use a soft toothbrush for dental hygiene.

The nurse evaluates that the client understood discharge teaching regarding warfarin (Coumadin) based on which statement?
a. "I will double my dose if I forget to take it the day before."
b. "I should keep taking ibuprofen for my arthritis."
c. "I should

a. Perform all necessary venipunctures.

What intervention is essential before the nurse administers tenecteplase (TNKase)?
a. Perform all necessary venipunctures.
b. Administer aminocaproic acid (Amicar).
c. Have the client void.
d. Assess for allergies to iodine.

c. Risk for injury

Which nursing diagnosis would be possible for a client receiving intravenous heparin therapy?
a. Potential for fluid volume excess
b. Potential for pain
c. Risk for injury
d. Potential for body image disturbance

c. Hyperlipidemia

A client has a serum cholesterol level of 265 mg/dL, triglyceride level of 235 mg/dL, and LDL of 180 mg/dL. What do these serum levels indicate?
a. Hypolipidemia
b. Normolipidemia
c. Hyperlipidemia
d. Alipidemia

a. 150 to 200 mg/dL

The nurse knows that the client's cholesterol level should be within which range?
a. 150 to 200 mg/dL
b. 200 to 225 mg/dL
c. 225 to 250 mg/dL
d. Greater than 250 mg/dL

b. It is the desired level of HDL.

A client's high-density lipoprotein (HDL) is 60 mg/dL. What does the nurse acknowledge concerning this level?
a. It is lower than the desired level of HDL.
b. It is the desired level of HDL.
c. It is higher than the desired level of HDL.
d. It is a much l

b. homocysteine

The nurse realizes that which is the laboratory test ordered to determine the presence of the amino acid that can contribute to cardiovascular disease and stroke?
a. antidiuretic hormone
b. homocysteine
c. ceruloplasmin
d. cryoglobulin

d. Liver enzymes

A client is taking lovastatin (Mevacor). Which serum level is most important for the nurse to monitor?
a. Blood urea nitrogen
b. Complete blood count
c. Cardiac enzymes
d. Liver enzymes

b. Rhabdomyolysis

The client is taking rosuvastatin (Crestor). What severe skeletal muscle adverse reaction should the nurse observe for?
a. Myasthenia gravis
b. Rhabdomyolysis
c. Dyskinesia
d. Agranulocytosis

a. Inhibits absorption of dietary cholesterol in the intestines.

When a client is taking ezetimibe (Zetia), she asks the nurse how it works. The nurse should explain that Zetia does what?
a. Inhibits absorption of dietary cholesterol in the intestines.
b. Binds with bile acids in the intestines to reduce LDL levels.
c.

a. Relaxes the arterial walls within the skeletal muscles
b. May cause hypotension, chest pain, and palpitations

A client is diagnosed with peripheral arterial disease (PAD). He is prescribed isoxsuprine (Vasodilan). The nurse acknowledges that isoxsuprine does what? (Select all that apply.)
a. Relaxes the arterial walls within the skeletal muscles
b. May cause hypo

b. "I will increase fiber in my diet.

Which statement indicates the client understands discharge instructions regarding cholestyramine (Questran)?
a. "I will take Questran 1 hour before my other medications."
b. "I will increase fiber in my diet."
c. "I will weigh myself weekly."
d. "I will h

b. Administer aspirin 30 minutes before nicotinic acid.

The nurse plans which intervention to decrease the flushing reaction of niacin?
a. Administer niacin with an antacid.
b. Administer aspirin 30 minutes before nicotinic acid.
c. Administer diphenhydramine hydrochloride (Benadryl) with niacin.
d. Apply cold

b. "Take this medication at the same time each day.

The nurse is reviewing instructions for a client taking an HMG-CoA reductase inhibitor (statin). What information is essential for the nurse to include?
a. "Take this medication on an empty stomach."
b. "Take this medication at the same time each day."
c.

b. "You may experience headaches with this medication.

A client is prescribed gemfibrozil (Lopid) for treatment of hyperlipidemia type IV. What is important for the nurse to teach the client?
a. "Take aspirin before the medication if you experience facial flushing."
b. "You may experience headaches with this

d. "I will continue my exercise program to help increase my high-density lipoprotein serum levels.

Which statement made by the client indicates understanding about discharge instructions on antihyperlipidemic medications?
a. "Antihyperlipidemic medications will replace the other interventions I have been doing to try to decrease my cholesterol."
b. "It

c. Muscle pain.

A client is prescribed ezetimibe (Zetia). Which assessment finding will require immediate action by the nurse?
a. Headache.
b. Slight nausea.
c. Muscle pain.
d. Fatigue.

c. Have the client increase fluids and fiber in his diet.

A nurse is caring for a client taking cholestyramine (Questran). The client is complaining of constipation. What will the nurse do?
a. Call the health care provider to change the medication.
b. Tell the client to skip a dose of the medication.
c. Have the

d. "I should stir the powder in as small an amount of fluid as possible to maintain potency of the medication.

Which statement indicates to the nurse that the client needs further medication instruction about colestipol (Colestid)?
a. "The medication may cause constipation, so I will increase fluid and fiber in my diet."
b. "I should take this medication 1 hour af

b. Elevated liver function tests

Which assessment finding in a client taking an HMG-CoA reductase inhibitor will the nurse act on immediately?
a. Decreased hemoglobin
b. Elevated liver function tests
c. Elevated HDL
d. Elevated LDL

b. "These factors may put you at higher risk for myopathy.

A 70-year-old client who is taking several cardiac antidysrhythmic medications has been prescribed simvastatin (Zocor) 80 mg/day. What is essential information for the nurse to teach the client?
a. "This dose may lower your cholesterol too much."
b. "Thes

b. Hepatic disease

A client diagnosed with hypercholesterolemia is prescribed lovastatin (Mevacor). The nurse is reviewing the client's history and would contact the health care provider about which of these conditions in the client's history?
a. Chronic pulmonary disease
b

c. gemfibrozil (Lopid)

A nurse is caring for a client with elevated triglyceride levels who is unresponsive to HMG-CoA reductase inhibitors. What medication will the nurse administer?
a. cholestyramine (Questran)
b. colestipol (Colestid)
c. gemfibrozil (Lopid)
d. simvastatin (Z

a. Impaction

The nurse would question an order for cholestyramine (Questran) if the client has which condition?
a. Impaction
b. Glaucoma
c. Hepatic disease
d. Renal disease

c. Client is on oral contraceptives.

The nurse reviews the history for a client taking atorvastatin (Lipitor). What will the nurse act on immediately?
a. Client takes medications with grape juice.
b. Client takes herbal therapy including kava kava.
c. Client is on oral contraceptives.
d. Cli

Erythropoietin regulates the process of red blood cell formation. The nurse understands that this mechanism is activated by a reduction of oxygen reaching the:

Kidneys.

A client with a diagnosis of cancer is receiving epoetin alfa (Epogen, Procrit) as part of the treatment regimen. The nurse evaluates the effectiveness of this drug by:

Monitoring the hematocrit and hemoglobin levels.

The nursing care plan for a client receiving epoetin alfa (Epogen, Procrit) should include careful monitoring for symptoms of:

Angina, or a change in level of consciousness.

The nurse administers filgrastim (Neupogen) to the client. The nurse explains that this drug is used in the treatment of:

Neutropenia, or neutropenia secondary to chemotherapy.

The client receiving filgrastim (Neupogen) should be monitored for common adverse effects, which include:

Hypertension and skeletal pain.

Prior to the administration of sargramostim (Leukine), the nurse should assess:

CBC.

The client receiving chemotherapy is prescribed oprelvekin (Neumega) as part of the treatment regimen. The nurse explains that the function of this drug is to:

Stimulate platelet production.

The nurse teaches the client and caregivers to immediately report to the health care provider which of the following symptoms of adverse reaction to oprelvekin (Neumega)?

Cough; difficulty breathing

The nurse explains to the client that the development of pernicious anemia is caused by:

Blood Loss

The nurse prepares nutritional teaching for a group of clients. The nurse recognizes that the client most at risk for ____________ anemia is the client diagnosed with Insufficient Dietary Intake.

Folate-deficiency

The client experiences shock following a spinal cord injury. This type of shock is classified as:

Neurogenic.

The client in shock is prescribed an infusion of lactated Ringer's solution. The nurse recognizes that the function of this fluid in the treatment of shock is to:

Replace fluid, and promote urine output.

Nursing assessment of a client receiving serum albumin for treatment of shock should include:

Assessing lung sounds.

Teaching for a client receiving plasma protein fraction (Plasmanate) should include reporting which of the following possible adverse reactions?

Anaphylactic reaction

The nurse should closely monitor the client receiving Dextran 70 (Macrodex) for:

Bleeding

Which of the following requires intervention by the nurse caring for a client receiving Phenylephrine (Neosynephrine)?

Decreased urinary output

Dobutamine (Dobutrex) is used to treat a client experiencing cardiogenic shock. Nursing intervention includes:

Monitoring for cardiac dysrhythmias.

The nurse weighs the client receiving dobutamine (Dobutrex) therapy daily. The rationale for this is to:

Determine drug dosage.

. The client care plan of a client receiving dopamine should include having phentolamine (Regitine) available for use. The purpose of this drug is to:

Prevent necrosis if the IV extravasates.

. The nurse evaluates the effectiveness of client education as it relates to anaphylaxis therapy. Which of the following might indicate that teaching was successful? (Select all that apply.)

I will carry an EpiPen to administer when I experience a hypersensitivity episode to peanuts."
"I will seek emergency medical attention immediately if a single auto-injection of epinephrine fails to bring relief.

An elderly patient diagnosed with iron-deficency anemia will be taking ferrous sulfate (feosol, others) orally. the nurse will teach required administration guidlines to the patient including (select all that apply)

Take the tablets on an empty stomach if possible
increase fluid intake and increase dietary fiber while taking this medication
if liquid prepartions are used, dilute with water or juice and sip through a straw placed in back of the mouth.

To best monitor for therapeutic effects from filgrastim (Neupogen) should include:

Frequent observations for infection

The patient in hypovolemic shock is prescribed an infusion of lactated Ringers's The nurse recognizes the function of this fluid in the treatment of shock is to (select all that apply)

Replace fluid and promote urine output
Draw water into cells
Maintain vascular volume

The nurse evaluates the effectiveness of dapamine therapy for a patient in shock. Which of the following may indicate the treatment is successful (select all that apply)

Improved urine output and Increased blood pressure

A patient is recieving a cyrstalloid infusion (lactated ringer;s) for treatement of hypovolemic shock . Because of concerns for fluid volume overload, the nurse will frequently monitor (select all that apply)

Breath sounds, daily weight, level of consciousness.

Your patient has been switched from valproic acid (Depakote) to gabepentin (Neurontin). Which of the following is a false statement?
a. Gabapentin (Neurontin) is also used for bipolar disorder therapy
b. Gabapentin (Neurontin) requires more frequent hepat

b. Gabapentin (Neurontin) requires more frequent hepatic monitoring

In the administration of a drug such as levothyroxine (Synthroid), the nurse should teach the client:
A) That therapy typically lasts about 6 months.
B) That weekly laboratory tests for T4 levels will be required.
C) To report weight loss, anxiety, insomn

C) To report weight loss, anxiety, insomnia, and palpitations.

A patient with hyperthyroidism is taking propylthiouracil (PTU). The nurse will monitor the patient for:
A) gingival hyperplasia and lycopenemia.
B) dyspnea and a dry cough.
C) blurred vision and nystagmus.
D) fever and sore throat.

D) fever and sore throat.

A type I diabetic patient comes to the clinic for a follow-up appointment. The patient is taking NPH insulin, 30 units every day. A nurse notes that the patient is also taking metoprolol (Lopressor). What education should the nurse provide to the patient?

D) "Metoprolol may mask signs of hypoglycemia, so you need to monitor your blood glucose closely.

A physician has prescribed propylthiouracil (PTU) for a client with hyperthyroidism and the nurse develops a plan of care for the client. A priority nursing assessment to be included in the plan regarding this medication is to assess for:
a) relief of pai

d) signs and symptoms of hypothyroidism
Excessive dosing with propylthiouracil (PTU) may convert the client from a hyperthyroid state to a hypothyroid state. If this occurs, the dosage should be reduced. Temporary administration of thyroid hormone may be

A physician prescribes levothyroxine sodium (Synthroid), 0.15 mg orally daily, for a client with hypothyroidism. The nurse will prepare to administer this medication:
a) in the morning to prevent insomnia
b) only when the client complains of fatigue and c

a) in the morning to prevent insomnia
Levothyroxine (Synthroid) is a synthetic thyroid hormone that increases cellular metabolism. Levothyroxine should be given in the morning in a single dose to prevent insomnia and should be given at the same time each

A nurse is monitoring a client newly diagnosed with diabetes mellitus for signs of complications. Which of the following, if exhibited in the client, would indicate hyperglycemia and warrant physician notification?
a) polyuria
b) diaphoresis
c) hypertensi

a) polyuria
Classic symptoms of hyperglycemia include polydipsia, polyuria, and polyphagia. Options B, C, and D are not signs of hyperglycemia.

Myxedema, which includes fatigue, general weakness, and muscle cramps, is a symptom of which endocrine disorder treated with levothyroxine (Synthroid)?
a. Hyperthyroidism
b. Hypothyroidism
c. Cushing's syndrome
d. Addison's disease

b. Hypothyroidism

Which disease is characterized by increased body metabolism, tachycardia, increased body temperature, and anxiety, and treated with Prophylthiouracil (PTU)?
a.) Hashimoto's thyroiditis
b.) Graves' disease
c.) Addison's disease
d.) Cushing's syndrome

b.) Graves' disease

In the administration of hydrocortisone (Aeroseb-HC, Alphadern, Cetacort), it is vital that the nurse recognize that this drug might mask which symptoms?
a.) Signs and symptoms of infection
b.) Signs and symptoms of heart failure
c.) Hearing loss
d.) Skin

a.) Signs and symptoms of infection

When hydrocortisone use is discontinued, the nurse must recognize the possibility of what side effect, if this drug is stopped abruptly?
a.) Development of myxedema
b.) Circulatory collapse
c.) Development of Cushing's syndrome
d.) Development of diabetes

b.) Circulatory collapse

A client who is taking levothyroxine (Synthroid) begins to develop weight loss, diarrhea, and intolerance. The nurse should be aware that this might be an indication of what hormonal condition?
a.) Addison's disease
b.) Hyperthyroidism
c.) Cushing's syndr

b.) Hyperthyroidism

Which organ is destroyed when administering radioactive I-131?
a.) Pituitary gland
b.) Adrenals
c.) Parathyroid
d.) Hypothalamus

radioactive I-131 destroys the thyroid because the thyroid cells are the only cells in the body that can take up iodine

Of what precautions should a client receiving radioactive iodine-131 be made aware?
a.) Drink plenty of fluids, especially those high in calcium.
b.) Avoid close contact with children or pregnant women for one week after administration of drug.
c.) Be awa

b.) Avoid close contact with children or pregnant women for one week after administration of drug.

In the administration of a drug such as levothyroxine (Synthroid), the nurse must teach the client: (Select all that apply.)
a.) Therapy could take three weeks or longer.
b.) Periodic lab tests for T4 levels are required.
c.) Report weight loss, anxiety,

Therapy could take three weeks or longer.
Periodic lab tests for T4 levels are required.
Report weight loss, anxiety, insomnia, and palpitations.
A,B,C

21. The client scheduled for electroconvulsive therapy tells the nurse, "I'm so afraid. What will happen to me during the treatment?" Which of the following statements is most therapeutic for the nurse to make?
A. "You will be given medicine to relax you

A.
The patient will receive medication that relaxes skeletal muscles and produces mild sedation.

Which information should be given to the client taking phenytoin (Dilantin)?
A. Taking the medication with meals will increase its effectiveness.
B. The medication can cause sleep disturbances
C. More frequent dental appointments will be needed for specia

C. More frequent dental appointments will be needed for special gum care.
Gingival hyperplasia is a side effect of phenytoin. The client will need more frequent dental visits.
Answers A, B, and D do not apply to the medication; therefore, they are incorre

36. The nurse is teaching the client with insulin-dependent diabetes the signs of hypoglycemia. Which of the following signs is associ- ated with hypoglycemia?
A. Tremulousness
B. Slow pulse
C. Nausea
D. Flushed skin

Answer A is correct.
Tremulousness (a state of trembling or quivering) is an early sign of hypoglycemia.
Answers B,C, and D are incorrect because they are symptoms of hyperglycemia.

A patient taking Dilantin (phenytoin) for a seizure disorder is experiencing breakthrough seizures. A blood sample is taken to determine the serum drug level. Which of the following would indicate a sub-therapeutic level?
A. 15 mcg/mL.
B. 4 mcg/mL.
C. 10

B. 4 mcg/mL.
The therapeutic serum level for Dilantin is 10 - 20 mcg/mL. A level of 4 mcg/mL is sub-therapeutic and may be caused by patient non-compliance or increased metabolism of the drug. A leve of 15 mcg/mL is therapeutic. Choices C and D are expres

A patient arrives at the emergency department complaining of back pain. He reports taking at least 3 acetaminophen tablets every three hours for the past week without relief. Which of the following symptoms suggests acetaminophen toxicity?
A. Tinnitus.
B.

Answer: D
Acetaminophen in even modestly large doses can cause serious liver damage that may result in death. Immediate evaluation of liver function is indicated with consideration of N-acetylcysteine administration as an antidote. Tinnitus is associated

A nurse is caring for a cancer patient receiving subcutaneous morphine sulfate for pain. Which of the following nursing actions is most important in the care of this patient?
A. Monitor urine output.
B. Monitor respiratory rate.
C. Monitor heart rate.
D.

Answer: B
Morphine sulfate can suppress respiration and respiratory reflexes, such as cough. Patients should be monitored regularly for these effects to avoid respiratory compromise. Morphine sulfate does not significantly affect urine output, heart rate,

18. A nurse is administering blood to a patient who has a low hemoglobin count. The patient asks how long to RBC's last in my body? The correct response is.
A. The life span of RBC is 45 days.
B. The life span of RBC is 60 days.
C. The life span of RBC is

D. The life span of RBC is 120 days.

A client has been newly diagnosed with hypothyroidism and will take levothyroxine (Synthroid) 50 mcg/day by mouth. As part of the teaching plan, the nurse emphasizes that this medication:
A) Should be taken in the morning
B) May decrease the client's ener

A) Should be taken in the morning

Your patient is taking valproic acid (Depakote). Which of the following is a false statement?
a. Valproic acid requires hepatic monitoring
b. Valproic acid has the lowest seizure relapse rate when discontinued
c. Valproic acid is also used in migraine the

b. Valproic acid has the lowest seizure relapse rate when discontinued

Your patient has been switched from valproic acid (Depakote) to gabepentin (Neurontin). Which of the following is a false statement?
a. Gabapentin (Neurontin) is also used for bipolar disorder therapy
b. Gabapentin (Neurontin) requires more frequent hepat

b. Gabapentin (Neurontin) requires more frequent hepatic monitoring

Your patient has been stabilized taking only primidone (Mysoline). Which drug besides primidone may be assayed during his stay in the hospital to monitor his therapy?
a. pentobarbital
b. phenobarbital
c. valproic acid
d. phenytoin

b. phenobarbital

Fosphenytoin (Cerebyx)
a. is a controversial agent for depression
b. is used to control tremors due to Parkinsonism
c. can be administered intravenously
d. is ineffective after 5 days of therapy

c. can be administered intravenously

Antidote for warfarin overdose
a. protamine zinc insulin
b. protamine sulfate
c. vitamin K
d. warfarin

c. vitamin K

Antidote for heparin overdose
a. protamine sulfate
b. vitamin K
c. vitamin E
d. cyanocobolamine

a. protamine sulfate

Antiplatelet agents include all of the following except
a. acetylsalicylic acid
b. acetaminophen
c. ticlopidine (Ticlid)
d. dipyridamole (Persantine)

b. acetaminophen

A classic drug interaction, greatly involving an increased bleeding time, involves warfarin and
a. vitamin B-6
b. acetaminophen
c. acetylsalicylic acid
d. all of the above

c. acetylsalicylic acid

A young woman makes an appointment to see a physician at the clinic. She complains of tiredness, weight gain, muscle aches and pains, and constipation. The physician will likely order:
1. T3 and T4 serum level laboratory tests.
2. glucose tolerance test.

1. T3 and T4 serum level laboratory tests.
These complaints are strongly suggestive of thyroid disorder; T3 and T4 laboratory tests are the most useful diagnostic tests.

The patient asks about his lab test, which showed a high level of TSH and a low level of T4. You explain:
1. "It means that you have an inconsistency in your thyroid tests, and you will need more testing."
2. "I am sorry. You will have to ask your doctor

3. "The TSH is sending a message to your thyroid gland to increase production, but your thyroid isn't doing that."
The test determines if the problem is in the pituitary or in the thyroid. In this case the high TSH is coming from the pituitary as it shoul

The nurse instructs the patient is scheduled to have a radioactive iodine uptake test to:
1. watch for any signs of bleeding or swelling from the biopsy site.
2. avoid contact with others until notified otherwise.
3. wash hands with soap and water after e

3. wash hands with soap and water after each urination for 24 hours after the test.
Radiation dose is small and will not harm others.

The patient, newly diagnosed with hypothyroidism, seems very anxious to begin her drug regimen. The nurse's instructions include:
1. "Be certain that no dose is skipped."
2. "If a dose is skipped one day, double the dose the next day."
3. "Know the signs

3. "Know the signs and symptoms of hyperthyroidism."
Her enthusiasm may lead her to overdose on the thyroid replacement pills. She needs to be aware of the proper prescription and the reasons for following the prescribed dosage.

Mr. Jessie Ray, a newly admitted patient, has a seizure disorder which is being treated with medication. Which of the following drugs would the nurse question if ordered for him?
A. Phenobarbitol, 150 mg hs
B. Amitriptylene (Elavil), 10 mg QID.
C. Valproi

B.
Elavil is an antidepressant that lowers the seizure threshold, so would not be appropriate for this patient. The other medications are anti-seizure drugs.

11. A nurse is preparing the client's morning NPH insulin dose and notices a clumpy precipitate inside the insulin vial. The nurse should:
A. draw up and administer the dose
B. shake the vial in an attempt to disperse the clumps
C. draw the dose from a ne

C.
The nurse should always inspect the vial of insulin before use for solution changes that may signify loss of potency. NPH insulin is normally uniformly cloudy. Clumping, frosting, and precipitates are signs of insulin damage. In this situation, because

A patient who has type 2 diabetes has a glycated hemoglobin (HbA1c) result of 10%. A nurse should make which of these changes to the nursing care plan?
A) Refer to a diabetic educator, there is poor glycemic control.
B) Glycemic control is adequate, no ch

A) Refer to a diabetic educator, there is poor glycemic control.

Which of these instructions should the nurse provide when teaching a patient to mix regular insulin and NPH insulin in the same syringe?
A) Draw up the clear regular insulin first, followed by the cloudy NPH insulin.
B) It is not necessary to rotate the N

A) Draw up the clear regular insulin first, followed by the cloudy NPH insulin.
To ensure a consistent response, only NPH insulin is appropriate for mixing with a short-acting insulin. Unopened vials of insulin should be refrigerated; current vials can be

NPH INSULIN

intermediate acting insulin

A patient is scheduled to start taking insulin glargine (Lantus). On the care plan a nurse should include which of these outcomes related to the therapeutic effects of the medication?
A) Blood glucose control for 24 hours
B) Mealtime coverage of blood glu

A) Blood glucose control for 24 hours
Insulin glargine is administered as a once-daily subcutaneous injection for patients who have type 1 diabetes. It is used for basal insulin coverage, not mealtime coverage. It has a prolonged duration up to 24 hours w

At 5 PM a patient who is taking NPH insulin develops hunger, shakiness, and sweating. A nurse assesses the medication administration record (MAR) and should recognize that the patient's symptoms are related to an injection of NPH insulin at which of these

A) 2 AM
The patient is exhibiting symptoms of hypoglycemia at 5 PM. NPH has a peak action of 8 to 10 hours after administration. Based on the duration of action of NPH insulin, the patient's hypoglycemic symptoms are from the 8 AM injection of NPH insulin

A teaching plan for a patient who is taking lispro (Humalog) should include which of these instructions by the nurse?
A) "Inject this insulin with your first bite of food because it is very fast acting."
B) "The duration of action for this insulin is abou

A) "Inject this insulin with your first bite of food because it is very fast acting."
Lispro is a rapid-acting insulin and has an onset of 15 to 30 minutes with a peak action of about 2 hours, not 8 to 10 hours. Because of its rapid onset, it is administe

A patient who is newly diagnosed with type 1 diabetes asks a nurse, "How does insulin normally work in my body?" The nurse explains that normal insulin has which of these actions in the body?
A) Stimulates the pancreas to reabsorb glucose
B) Promotes synt

D) Promotes the passage of glucose into cells for energy
Insulin is a hormone that promotes the passage of glucose into cells, where it is metabolized for energy. Insulin does not stimulate the pancreas to reabsorb glucose or synthesize amino acids into g

A patient is taking glipizide (Glucotrol) and a beta-adrenergic medication. A nurse is teaching hypoglycemia awareness and should tell the patient that which of these symptoms may not occur?
A) Vomiting
B) Muscle cramps
C) Tachycardia
D) Chills

C) Tachycardia
Glipizide is a sulfonylurea oral hypoglycemic medication that acts to promote insulin release from the pancreas. Beta-adrenergic blockers can mask early signs of sympathetic system responses (most importantly, tachycardia) to hypoglycemia,

A nurse assesses a patient who is taking pramlintide (Symlin) with mealtime insulin. Which of these findings should require immediate follow-up by the nurse?
A) Skin rash
B) Sweating
C) Itching
D) Pedal edema

B) Sweating
Pramlintide is a new type of antidiabetic medication that is used as a supplement to mealtime insulin in type 1 and 2 diabetes. Hypoglycemia, which is manifested by sweating, tremors, and tachycardia, is the adverse reaction of most concern. S

Which of these characteristics should a nurse associate with a patient who has type 2 diabetes? (Select all that apply.)
A) Exercise and diet may be sufficient treatment
B) Is often obese with difficulty managing weight
C) Prone to ketosis and ketoacidosi

A
B
D
E

A nurse caring for a patient who has diabetic ketoacidosis recognizes which of these characteristics in the patient? (Select all that apply.)
A) Occurs mainly in type 2 diabetes patients
B) Altered fat metabolism leading to ketones
C) Arterial blood pH of

B
D
E

A postoperative patient has an epidural infusion of morphine sulfate (Astramorph). The patient's respiratory rate declines to 8 breaths/min. Which medication would the nurse anticipate administering?
A) Naloxone (Narcan)
B) Acetylcysteine (Mucomyst)
C) Me

A) Naloxone (Narcan)
Naloxone is a narcotic antagonist that can reverse the effects, both adverse and therapeutic, of opioid narcotic analgesics.

The nurse is planning care for a patient receiving morphine sulfate (Duramorph) by means of a patient-controlled analgesia (PCA) pump. Which intervention may be required because of a potential adverse effect of this drug?
A) Administer cough suppressant.

B) Insert Foley catheter.
Morphine can cause urinary hesitancy and urinary retention. If bladder distention or the inability to void is noted, the prescriber should be notified. Urinary catheterization may be required. Morphine acts as a cough suppressant

A patient admitted to the hospital with a diagnosis of pneumonia asks the nurse, "Why am I receiving codeine? I don't have any pain." The nurse's response is based on the knowledge that codeine also has which effect?
A) Immunostimulant
B) Antitussive
C) E

B) Antitussive
Codeine provides both analgesic and antitussive therapeutic effects.

A patient takes oxycodone (OxyContin), 40 mg PO twice daily, for the management of chronic pain. Which intervention should be added to the plan of care to minimize the gastrointestinal adverse effects?
A) Take an antacid with each dose.
B) Eat foods high

D) Increase fluid and fiber in the diet.
Narcotic analgesics reduce intestinal motility, leading to constipation. Increasing fluid and fiber in the diet can help manage this adverse effect.

OxyContin

Oxycodone, Narcotic pain reliever, analgesic

Which agent below is most likely to cause serious respiratory depression as a potential adverse reaction?
A) Morphine (Duramorph)
B) Pentazocine (Talwin)
C) Hydrocodone (Lortab)
D) Nalmefene (Revex)

A) Morphine (Duramorph)
Morphine is a strong opioid agonist and as such has the highest likelihood of respiratory depression. Pentazocine, a partial agonist, and hydrocodone, a moderate to strong agonist, may cause respiratory depression but not as often

The nurse is working on a postoperative unit where pain management is part of routine care. Which statement below is the most helpful in guiding clinical practice in this setting?
A) At least 30% of the U.S. population is prone to drug addiction and abuse

B) The development of opioid dependence is rare when opioids are used for acute pain.

A nurse administers naloxone (Narcan) to a postoperative patient experiencing respiratory sedation. What undesirable effect would the nurse anticipate after giving this medication?
A) Drowsiness
B) Tics and tremors
C) Increased pain
D) Nausea and vomiting

C) Increased pain
Naloxone is a medication that reverses the effects of narcotics. Although the patient's respiratory status will improve after the administration of naloxone, pain will be more acute.

The client informs the nurse that he has experienced pain in the lower extremities for the past eight months. The nurse recognizes that this pain is classified as:
a.) Moderate.
b.) Severe.
c.) Acute.
d.) Chronic.

d.) Chronic.
Chronic pain persists longer than six months.

A client who incurred an arm injury describes his pain as "sharp and localized to the lower arm." The nurse recognizes that this type of pain would be relieved best by administration of which type of medication?
a.) Muscle relaxant
b.) Acetaminophen
c.) N

c.) Narcotic analgesics
Injury to tissues produces nociceptor pain, which usually responds to conventional analgesic pain medications such as opiates or NSAIDS.

The nurse teaches the client relaxation techniques and guided imagery as an adjunct to medication for treatment of pain. The nurse explains that the major benefit of these techniques is that they:
a.) Are less costly.
b.) Allow lower doses of drugs with f

b.) Allow lower doses of drugs with fewer side effects.
When used concurrently with medication, non-pharmacologic techniques can allow for lower doses, and possibly fewer drug-related adverse effects. The other options also are advantages to guided imager

The nurse recognizes that opioid analgesics exert their action by interacting with a variety of opioid receptors. Drugs such as morphine act by activating

Mu and kappa

The client admitted with hepatitis B is prescribed Vicodin tabs 2 for treatment of pain. The appropriate nursing action is to:
a.) Administer the drug as ordered.
b.) Administer one tablet only.
c.) Question the physician about the order.
d.) Hold the dru

c.) Question the physician about the order.
Vicodin is a combination drug of hydrocodone and acetaminophen. Acetaminophen can be hepatotoxic, and is contraindicated in liver disease.

The nurse administers morphine sulfate 4 mg IV to a client for treatment of severe pain. Which of the following assessments requires immediate nursing interventions?
a.) Blood pressure 110/70
b.) The client is drowsy.
c.) Pain is unrelieved in 15 minutes.

d.) Respiratory rate 10/minute
Opioids activate mu and kappa receptors that can cause profound respiratory depression. Respiratory rate should remain above 12. The BP is not significantly low. Drowsiness is an expected effect of morphine. Unrelieved pain

Nursing intervention for a client receiving opioid analgesics over an extended period of time should include:
a.) Referring the client to a drug treatment center.
b.) Encouraging increased fluids and fiber in the diet.
c.) Monitoring for G.I. bleeding.
d.

b.) Encouraging increased fluids and fiber in the diet.
Opioids suppress intestinal contractility, increase anal sphincter tone, and inhibit fluids into the intestines, which can lead to constipation. There is nothing to indicate the drug is related to ad

Naloxone (Narcan) is administered to a client with severe respiratory depression and suspected drug overdose. After 20 minutes, the client remains unresponsive. The most likely explanation for this is:
a.) The client did not use an opioid drug.
b.) The do

a.) The client did not use an opioid drug.
If opioid antagonists (Naloxone) fail to reverse symptoms of respiratory depression quickly, the overdose was likely due to a non-opioid substance.

Celecoxib (Celebrex) is added to the treatment regimen of a client with arthritis. The nurse explains that the major advantage of this drug is:
a.) The drug is less expensive.
b.) The drug has no known side effects.
c.) The drug has anti-inflammatory prop

c.) The drug has anti-inflammatory properties.
Celecoxib (Celebrex) has anti-inflammatory properties. It is not less expensive, has many side effects, and is less potent than opioids.

The client is prescribed ketorolac tromethamine (Toradol) for treatment of pain following a surgical procedure. The nurse should question which of the following drug orders?
a.) Toradol 10 mg p.o. b.i.d.
b.) Toradol 20 mg p.o. b.i.d
c.) Toradol 5 mg p.o.

b.) Toradol 20 mg p.o. b.i.d
The maximum daily dose of Toradol is 40 mg.

Blood sugar is well controlled when Hemoglobin A1C is:
a. Below 7%
b. Between 12%-15%
c. Less than 180 mg/dL
d. Between 90 and 130 mg/dL

a. Below 7%
A1c measures the percentage of hemoglobin that is glycated and determines average blood glucose during the two to three months prior to testing. Used as a diagnostic tool, A1C levels of 6.5% or higher on two tests indicate diabetes. A1C of 6%

Untreated hyperglycemia may lead to all of the following complications except:
a. Hyperosmolar syndrome
b Vitiligo
c. Diabetic ketoacidosis
d. Coma

B.
Excessively high blood sugar or prolonged hyperglycemia can cause diabetic ketoacidosis, the condition in which the body breaks down fat for energy and ketones spill into the urine. Diabetic hyperosmolar syndrome occurs when blood sugar is excessively

Which of the following diabetes drugs acts by decreasing the amount of glucose produced by the liver?
a. Sulfonylureas
b. Meglitinides
c. Biguanides
d. Alpha-glucosidase inhibitors

c. Biguanides
Biguanides, such as metformin, lower blood glucose by reducing the amount of glucose produced by the liver.
Sulfonylureas and Meglitinides stimulate the beta cells of the pancreas to produce more insulin.
Alpha-glucosidase inhibitors block t

The benefits of using an insulin pump include all of the following except:
a. By continuously providing insulin they eliminate the need for injections of insulin
b. They simplify management of blood sugar and often improve A1C
c. They enable exercise with

d. They help with weight loss
Using an insulin pump has many advantages, including fewer dramatic swings in blood glucose levels, increased flexibility about diet, and improved accuracy of insulin doses and delivery; however, the use of an insulin pump ha

Which of the following regimens offers the best blood glucose control for persons with type 1 diabetes?
a. A single anti-diabetes drugs
b. Once daily insulin injections
c. A combination of oral anti-diabetic medications
d. Three or four injections per day

D. Three or four injections per day of different types of insulin.
Because persons with type 1 diabetes do not produce insulin, they require insulin and cannot be treated with oral anti-diabetic drugs.
Several injections of insulin per day, calibrated to

A patient has just been diagnosed with diabetes mellitus. His doctor has requested glucagon for emergency use at home. The nurse instructs the patient that the purpose of this drug is to treat:
A. Hyperglycemia from insufficient insulin injection.
B. Hype

C. Hypoglycemia from insulin overdose.
Glucagon is for emergency use for insulin overdose. The patient will usually arouse within 20 minutes if unconscious.
The family should also be instructed how to use the glucagon injection as well.

Which of the following statements from a newly diagnosed client with diabetes indicates more instruction is needed?
A.) i need to check my feet daily for sores
B.) i need to store my insulin in the refrigerator
C.) i can use my plastic insulin syringe mor

B.) i need to store my insulin in the refrigerator
Insulin only needs to be stored in the refrigerator if it wont be used within 6 weeks, after being opened. It should be at room temperature when given to decrease pain and prevent lipodystrophy.

Following heparin treatment for a pulmonary embolism, a client is being discharged with a prescription for warfarin (Coumadin). In conducting discharge teaching, the nurse advises the client to have which diagnostic test monitored regularly after discharg

B) Prothrombin Time (PT/INR)

The healthcare provider prescribes naproxen (Naproxen) twice daily for a client with osteoarthritis of the hands. The client tells the nurse that the drug does not seem to be effective after three weeks. Which is the best response for the nurse to provide

C) Another type of nonsteroidal antiinflammatory drug may be indicated.

The nurse instructs a patient about how insulin affects blood glucose. Arrange the events in sequence.
1. Beta cells are stimulated to release insulin.
2. Glucose enters the bloodstream.
3. Glycogen is converted to glucose by alpha cells (glycogenesis).
4

2
1
5
4
3

The teaching plan for a diabetic is focused on smoking cessation and control of hypertension for the avoidance of microvascular complications, such as (select all that apply):
1. macular degeneration.
2. end-stage renal disease (ESRD).
3. coronary artery

ANS: 1, 2
Macular degeneration and ESRD are both microvascular complications. CAD, PVD, and CVA are all macrovascular complications.

The patient has been admitted with hyperglycemic hyperosmolar nonketotic syndrome (HHNKS). Her blood glucose level is very high (880 mg/dL on admission). The physician believes that her condition is to the result of large amounts of glucose solutions admi

2. severe dehydration and hypernatremia caused by the hyperglycemia.
IV solutions containing glucose will bypass the digestive system, so there is no trigger for the pancreas to release insulin, but there is just enough insulin to prevent the breakdown of

The nurse giving Humulin R 20 U at 7 AM is aware that this drug will peak in:
1. 15 minutes.
2. 30 minutes.
3. 1 hour.
4. 2 hours.

Humulin R has its onset in about 15 minutes, but its peak is in 2 hours.

When the Type 1 diabetic patient asks why his 7 AM insulin has been changed from NPH insulin to 70/30 premixed insulin, the nurse explains that 70/30 insulin:
1. is absorbed more rapidly into the bloodstream.
2. has no peak action time and lasts all day.

4. give a bolus of rapid-acting insulin to prevent hyperglycemia after breakfast.
70/30 insulin is 30% rapid-acting and 70% intermediate-acting insulin. The rapid action of the 7 AM premixed insulin prevents hyperglycemia after the morning meal.

The type 1 diabetic patient has an insulin order for NPH insulin, 35 U, to be given at 7 AM. The patient is also NPO for laboratory work that will not be drawn until 10 AM. The nurse
should:
1. give the insulin as ordered.
2. give the insulin with a small

4. hold the insulin until after the blood
Holding the insulin for the NPO order is appropriate. The patient will not be getting food until after the blood draw, so will not need the insulin until then. Giving the insulin as ordered will create a possibili

A patient has come to the doctor's office after finding out that her blood glucose level was 135 mg/dL. She states that she had not eaten before the test and was told to come and see her doctor. She asks you if she has
diabetes. The nurse responds:
1. "Ha

2. "That test indicates that we need to do more tests that are specific for diabetes
The nurse needs to answer the patient's question in a way that gives information and is not misleading. Although 135 is high, there may be a nonpathologic explanation. Mo

A patient has come into the emergency room with her friend. Her friend states that she had been acting very strangely and confused. The friend states that the patient has diabetes and takes insulin. The nurse knows that signs and symptoms of hypoglycemia

2. irritability, anxiety, confusion, and dizziness
When blood sugar levels fall, hormones are activated to increase serum glucose. One of the hormones is epinephrine, which causes these symptoms.

The nurse is drawing up a teaching plan for a patient who has type 1 diabetes. The doctor has ordered two types of insulin, 10 U of regular insulin and 35 U of NPH insulin. The proper procedure is to:
1. draw up the insulins in two separate syringes so th

4. inject 35 U air into the NPH insulin, inject 10 U air into the regular insulin, withdraw 10 U of the regular insulin, and withdraw 35 U of the NPH insulin.
When drawing up two insulins, the vials are injected with air and the regular insulin is drawn f

The home health nurse is assessing a type 1 diabetic patient who has been controlled for 6 months. The nurse is surprised and concerned about a blood glucose reading of 52. This episode of hypoglycemia is probably caused by the patient's having:
1. taken

ANS: 3
Excessive exercise used up the glucose that was made available by the insulin taken by the patient. The patient now has too much insulin for the available glucose and has become hypoglycemic.

The patient with type 2 diabetes shows a blood sugar reading of 72 at 6 AM. Based on the reading of 72, the nurse should:
1. notify the charge nurse of the reading.
2. give regular insulin per sliding scale.
3. give him cup of milk.
4. administer the oral

ANS: 3 milk
The patient is hypoglycemic and needs an immediate source of glucose, such as milk or orange juice. The oral hyperglycemic agent will not work quickly enough. Notifying the charge nurse can be done later. Giving insulin per sliding scale would

When the type 2 diabetic patient says, "Why in the world are they looking at my hemoglobin? I thought my problem was with my blood sugar." The nurse responds that the level of hemoglobin A1c:
1. shows how a high glucose level can cause a significant drop

ANS: 2
By analyzing the amount of glucose bound to the hemoglobin, the level of blood glucose can be evaluated for the last 3 months, because the glucose stays bound to the hemoglobin for the life of the red blood cell (RBC).

When a newly diagnosed type 2 diabetes mellitus patient asks the nurse why she has to take a pill instead of insulin, you reply that in type 2 diabetes, the body makes insulin but:
1. overweight and underactive people simply cannot use the insulin produce

ANS: 4
Type 2 diabetes mellitus is a disease in which the cells become resistant to the action of insulin and the blood glucose level rises. Oral hyperglycemic agents make the cells more sensitive.

The nurse is caring for a patient whose seizures are characterized by a 10- to 30-second loss of consciousness with mild symmetric eye blinking. Which seizure type does this most closely illustrate?
A) Tonic-clonic
B) Absence
C) Atonic
D) Myoclonic

B) Absence
This scenario accurately describes absence seizures.
Tonic-clonic seizures present with convulsions and muscle rigidity followed by muscle jerks. Patients may experience urinary incontinence and loss of consciousness.
Atonic seizures cause sudd

The nurse is teaching a patient who is newly diagnosed with epilepsy about her disease. Which statement made by the nurse best describes the goals of
antiepilepsy medication therapy?
A) "With proper treatment we can completely eliminate your seizures."
B)

B) "Our goal is to reduce your seizures to an extent that helps you live a normal life."
Epilepsy is treated successfully with medication in a majority of patients. However, the dosages needed to completely eliminate seizures may cause intolerable side ef

The nurse is assessing a patient receiving valproic acid (Depakene) for potential adverse effects associated with this drug. Which item represents the most common problem with this drug?
A) Increased risk for infection
B) Reddened, swollen gums
C) Nausea,

D) Central nervous system depression
Valproic acid is generally well tolerated.
It does not cause hematologic effects resulting in increased risk for infection nor does it cause gingival hyperplasia.
It causes minimal sedation.
Gastrointestinal effects, w

The nurse is preparing to give ethosuximide (Zarontin). The nurse understands that this drug is only indicated for which seizure type?
A) Tonic-clonic
B) Absence
C) Simple partial
D) Complex partial

B) Absence
Absence seizures are the only indication for ethosuximide. The drug effectively eliminates absence seizures in approximately 60% of patients and effectively controls 80% to 90% of cases.

The nurse is conducting discharge teaching related to a new prescription for phenytoin (Dilantin). Which statements are appropriate to include in the teaching for this patient and his family? Select all that apply.
A) "Be sure to call the clinic if you or

A, B, D
Patients receiving an antiepileptic drug are at increased risk for suicidal thoughts and behavior beginning early in their treatment. The U.S. Food and Drug Administration (FDA) advises that patients, families, and caregivers be informed of the si

The nurse receives a lab report indicating that the phenytoin (Dilantin) level for the patient she saw in the clinic yesterday is 16 mcg/mL. Which intervention is most appropriate?
A) Continue as planned since the level is within normal limits.
B) Tell th

A) Continue as planned since the level is within normal limits

1. Which of the following statements made by a client taking phenytoin indicates understanding of the nurse's teaching?
A. "I will increase the dose if my seizures don't stop."
B. "I don't need to contact my health care provider before taking an over-the-

C. "I will take good care of my teeth and see my dentist regularly.

A 20-year-old client presents to the clinic with complaints of breast tenderness, nausea, vomiting, and absence of menses for 2 months. She has a history of a seizure disorder well controlled with carbamazepine (Tegretol). She tells the nurse that she has

B. "Carbamazepine can decrease the effectiveness of oral contraceptive drugs, so we need to do a pregnancy test.

A client in the trauma ICU is experiencing deep, throbbing pain. The nurse will provide medication for this pain because:
A.) The pain is being transmitted over C fibers and the enkephalins will not be
effective to control the pain.
B.) The pain is being

A.) The pain is being transmitted over C fibers and the enkephalins will not be
effective to control the pain.

A client in the ICU tells the nurse he is experiencing severe pain. Prior to administering a narcotic analgesic to this client, the nurse will conduct a pain assessment to include:
a.) Pain
b.) Nociception
c.) Pain behaviors
d.) Suffering

c.) pain behavior
There is a theory that addresses pain as having four facets: nociception, pain, suffering, and pain behaviors. Of these four facets, only the fourth, pain behavior, can be observed. This nurse will only be able to assess the client's pai

A client in the ICU who sustained a traumatic abdominal injury 1 week ago continues to complain of severe pain. The nurse notes his vital signs are normal. Which of the following would be appropriate for the nurse to do?
a.) Encourage early return to ambu

D.) Provide the client with pain medication.

The nurse is creating a pain management plan for a client with a previous history of substance abuse. Which of the following should be included in this plan?
a.) Ask the physician to prescribe short-acting analgesics.
b.) Ask the physician to prescribe a

C.) Ask the physician to prescribe all analgesics for the oral route.
Extended-release and long-acting analgesics are recommended for clients with a history of abuse.
Specific interventions should avoid analgesics similar to the abused drug, utilize long-

The safest narcotic choice for an elderly client with acute pain is:
a. Meperidine (Demerol).
b. Oxycodone.
c. Fentanyl transdermal patch.
d. Morphine sulfate.

d. Morphine sulfate.
Rationale: Morphine is the "gold standard" of narcotics for acute pain. The other choices are incorrect.

An elderly client had abdominal surgery six hours earlier. When the nurse asks the client about pain, the client responds that there is none. The best intervention on the part of the nurse is:
a. Administer a PRN dose of IV pain medication as ordered.
b.

c. Question the client further about discomfort to assess the meaning of pain.

A resident of the nursing home has quite severe arthritis. When administering an analgesic to this elderly resident, the nurse should:
a. Give the medication before the activity session in the day room.
b. Give the medication when the resident states the

a. Give the medication before the activity session in the day room.

Two days after surgery, an elderly client refuses a PRN dose of analgesic dose for fear of becoming "hooked." The nurse should respond by stating that:
a. It is impossible to become hooked on PRN narcotics.
b. Short-term use of narcotics is not likely to

b. Short-term use of narcotics is not likely to cause a person to become dependent on them.

When an elderly client with cancer experiences "breakthrough pain," the nurse should expect that pharmacological treatment will include:
a. Initiation of a placebo after every third dose of narcotic.
b. More aggressive chemotherapy.
c. Giving narcotics ev

d. Increasing the dose of the narcotic.

A patient with newly diagnosed type 2 diabetes mellitus asks the nurse what "type 2" means in relation to diabetes. The nurse explains to the patient that type 2 diabetes differs from type 1 diabetes primarily in that with type 2 diabetes
a. the patient i

B
Rationale: In type 2 diabetes, the pancreas produces insulin, but the insulin is insufficient for the body's needs or the cells do not respond to the insulin appropriately. The other information describes the physiology of type 1 diabetes.

A patient screened for diabetes at a clinic has a fasting plasma glucose level of 120 mg/dl (6.7 mmol/L). The nurse will plan to teach the patient about
a. use of low doses of regular insulin.
b. self-monitoring of blood glucose.
c. oral hypoglycemic medi

D
Rationale: The patient's impaired fasting glucose indicates prediabetes and the patient should be counseled about lifestyle changes to prevent the development of type 2 diabetes. The patient with prediabetes does not require insulin or the oral hypoglyc

When assessing the patient experiencing the onset of symptoms of type 1 diabetes, which question should the nurse ask?
a. "Have you lost any weight lately?"
b. "Do you crave fluids containing sugar?"
c. "How long have you felt anorexic?"
d. "Is your urine

A
Rationale: Weight loss occurs because the body is no longer able to absorb glucose and starts to break down protein and fat for energy. The patient is thirsty but does not necessarily crave sugar- containing fluids. Increased appetite is a classic sympt

During a clinic visit 3 months following a diagnosis of type 2 diabetes, the patient reports following a reduced-calorie diet. The patient has not lost any weight and did not bring the glucose-monitoring record. The nurse will plan to obtain a(n)
a. fasti

C
Rationale: The glycosylated hemoglobin (Hb A1C) test shows the overall control of glucose over 90 to 120 days.
A fasting blood level indicates only the glucose level at one time.
Urine glucose testing is not an accurate reflection of blood glucose level

A college student who has type 1 diabetes normally walks each evening as part of an exercise regimen. The student now plans to take a swimming class every day at 1:00 PM. The clinic nurse teaches the patient to
a. delay eating the noon meal until after th

D
Rationale: The change in exercise will affect blood glucose, and the patient will need to monitor glucose carefully to determine the need for changes in diet and insulin administration. Because exercise tends to decrease blood glucose, patients are advi

Alprazolam (Xanax)

Indication: anxiety, panic disorders, anxiety caused by depression
Class: benzodiazapien
Labs: hematocrit may decrease, renal function, liver function
Side effects: dizziness, drowsiness, depression, dependance
Precuations: monitor risk to fall, monitor d

Amitriptyline (Elvail)

Indication: depression
Class: antidepressant
Labs: liver, WBC, glucose
Side effects: suicide, arrythmias, hypotension, blurred vision, lethargy, dry mouth
Precuations: monitor BP and pulse before and after dose, monitor for suicide

Amoxicillin/clvulanate (Augmentin)

Indication: infection
Class: anti-infective
Labs: liver
Side effects: seizure, diarrhea, rash, C-Diff
Precuations: assess vital signs, appearance of any infection, monitor bowel function, monitor for signs of anaphlaxis

Betamethasone (Celestone)

Indication: asthma, neoplastic disease, renal disease, respiratory disease, pre-term labor at 36wk Class: gluccocorticoid
Labs: WBC, glucose (increase), Na (increase), potassium(decrease)
Side effects: depression, euphoria, decreased healing, hypertension

Bumetanide (Bumes)

Indication: fluid retention, high blood pressure, heart failure
Class: loop diuretic
Labs: Na, K, Mg, renal, liver
Side effects: dehydration, low K, low Na, metabolic alkalosis, muscle cramps
Precuations: fluid status, lung sounds, location of edema, moni

Bupropion (Wellbutrin)

Indication: depression, quit smoking aid Class: antidepressant
Labs: liver, renal
Side effects: seizure, suicide, agitation, headache, tremor, N/V
Precuations: mental status, mood changes

Calcitriol (Rocaltrol)

Indication: low blood calcium due to kidney dialysis, osteoporosis, hyperparathyroidism
Class: Vitamin D
Labs: Mg, Ca, phosphate
Side effects: pancreatitis, dizziness, malasia
Precuations: toxicity is hypocalcaemia assess for N/V, weakness, constipation,

Ceftazidime

Indication: pseudomonas, netropenia
Class: anti-infective broad spectrum
Labs: liver, WBC
Side effects: seizure, C-Diff, rash, diarrhea, N/V, anaphylaxis
Precuations: monitor for anaphylaxis, bowel function, assess infection, obtain culture

Cephalonia (Keflex)

Indication: UTI, sore throat, e-coli, pneumonia, wounds
Class: anti-infective
Labs: liver, WBC
Side effects: seizure, C-Diff, rash, diarrhea, N/V, anaphylaxis
Precuations: monitor for anaphylaxis, bowel function, assess infection, obtain culture

Ciprofloxacin

Indication: UTI, gonorrhea, sinus infection, e-coli
Class: anti-infective
Labs: liver, WBC, glucose, cholesterol, Ca
Side effects: seizure, C-Diff, rash, diarrhea, N/V, anaphylaxis, arrythmias, hyperglycemia
Precuations: monitor for anaphylaxis, bowel fun

Clonazepam

Indication: seizures, panic disorders, anxiety
Class: benzodiazepine
Labs: CBC, liver
Side effects: suicide, mood changes, lethargy, ataxia
Precuations: monitor levels, suicide, monitor and record seizure activity
Reversal: Flumanazil

Cyclobenzaprine (Flexrol)

Indication: acute painful muscle spasms Class: muscle relaxant
Labs: N/A
Side effects: dizziness, drowsiness
Precautions: monitor pain and ROM

Diazepam (Valium)

Indication: anxiety, conscious sedation, seizures, alcohol withdrawl
Class: benzodiazepines
Labs: hepatic, renal, CMB
Side effects: dizziness, lethargy, drowsiness
Precautions: monitor vitals, physical dependence for long term use, assess mental status, a

Enalapril (Vasotec)

Indication: CHF, lower blood pressure, improve CO
Class: ACE inhibitor
Labs: BUN, Cr, Na, hepatic, urine output
Side effects: hypotension, cough, dizziness
Precautions: monitor blood pressure and pulse, monitor weight, monitor for fluid overload, monitor

Erythromycin

Indication: infection, if penicillin can't be used
Class: anti-infective
Labs: hepatic
Side effects: prolonged QRS, arrythmias, N/V, itching
Precautions: monitor infection, culture and sensitivity

Famotidine (Pepcid)

Indication: ulcers, GERD
Class: H2 blockers
Labs: CBC
Side effects: confusion, arrythmias
Precautions: assess for epigastria or abdominal pain, frank occult blood, gastric aspirate, or emesis

Fluconazole

Indication: UTI, yeast infection
Class: antifungal
Labs: liver, renal
Side effects: hypokalemia, hepatotoxicity
Precautions: assess infection, renal dysfunction need dose adjustment

Fluoxetine (Prozac)

Indication: depression, panic disorder, OCD
Class: antidepressant
Labs: CBC, hepatic, renal
Side effects: seizures, suicidal thoughts, headache, insomnia, diarrhea, sweating
Precautions: suicide, mood changes, changes in mental status (serotonin syndrome)

Furosemide (Lasix)

Indication: edema caused by HF, liver disease, or renal failure, hypertension
Class: loop diuretic
Labs: Na, K, glucose, renal, Mg
Side effects: tinnitus, hypokalemia, low mag, hypovolemia, hyponatremia, dehydration, hyperglycemia
Precautions: monitor flu

Gentamicin

Indication: staph infections
Class: amino glycoside, anti-infective
Labs: renal, hepatic, calcium, gent levels Side effects: ototoxicity, nephrotoxicity
Precautions: infection monitor, test hearing, monitor I/O, monitor neurological function
Levels: Peak

Haloperidol (Haldol)

Indication: schizophrenia, chronic psychotic disorders, manic episodes Class: anti-psychotic
Labs: CBC, liver, WBC
Side effects: seizures, blurred vision, constipation, hypotension, respiratory depression, confusion
Precautions: monitor mental status, mon

Lisinopril

Indication: CHF, lower blood pressure, improve CO
Class: ACE inhibitor
Labs: BUN, Cr, Na, hepatic, urine output
Side effects: hypotension, cough, dizziness
Precautions: monitor blood pressure and pulse, monitor weight, monitor for fluid overload, monitor

Atrovant (Ipratropium Bromide)

Indication: COPD, cold, cough, allergies
Class: bronchodialator
Labs: N/A
Side Effects: headache, palpitations
Precautions: monitor respiratory status

Ativan (Lorazepam)

Indication: anxiety, preoperative sedation, seizures
Class: benzodiazepine
Labs: renal, hepatic, hematological
Side Effects: drowsiness, apnea, cardiac arrest, dependence, tolerance
Precautions: assess mental status, assess seizures, monitor for dependenc

Ketoalac

Indication: short term management not to exceed 5 days
Class: NSAID
Labs: hepatic, bleeding time, BUN, Cr, K
Side Effects: drowsiness, anaphylaxis, GI bleeding, asthma, renal toxicity
Precautions: rhinitis, asthma, and aspirin allergy increase risk for hy

Hesperidins (Dmerol)

Indication: moderate to severe pain
Class: opiod
Labs: N/A
Side Effects: seizure, confusion, hypotension, anaphylaxis, respiratory depression
Precautions: assess pain, monitor vitals, monitor bowel function
Reversal: Narcan

Metoprolol (Lopressor)

Indication: angina, hypertension, prevention and decreased mortality of MI
Class: beta blocker
Labs: glucose, hepatic, K, cholesterol, uric acid
Side Effects: fatigue, weakness, bradycardia, hypotension, pulmonary edema
Precautions: hold for pulse less th

Nystatin

Indication: skin fungus infections
Class: antifungal
Labs: N/A
Side Effects: burning, itching
Precautions: inspect affected area

Oxybutyn

Indication: Neurogenic bladder, increase bladder capacity
Class: anticholinergic
Labs: N/A
Side Effects: drowsiness, constipation, urinary retention
Precautions: monitor voiding pattern, assess for bladder distention

Oxymetholone

Indication: anemia
Class: androgen (increase production of erythropoietin)
Labs: hepatic
Side Effects: acne, allergic reaction, depression, mood changes, anxiety
Precautions: do not take if pregnant

Dilantin

Indication: prevention of seizures
Class: anti-arrhythmic
Labs: CBC, Ca, albumin, hepatic
Side Effects: ataxia, suicide, anemia, hypotension, rash
Precautions: monitor for mood changes, assess for hypersensitivity, oral hygiene, monitor seizures, monitor

Prazepam

Indication: anxiety, conscious sedation, seizures, alcohol withdrawl
Class: benzodiazepines
Labs: hepatic, renal, CMB
Side effects: dizziness, lethargy, drowsiness
Precautions: monitor vitals, physical dependence for long term use, assess mental status, a

Promethazine

Indication: vomiting and nausea
Class: anti-metic
Labs: glucose
Side Effects: confusion, sedation, bradycardia
Precautions: monitor vitals, monitor N/V

Propoxyphene

Indication: pain
Class: opiod
Labs: hepatic, amylase, lipase
Side Effects: dizziness, weakness, nausea, hypotension
Precautions: monitor pain, vitals, bowel function
Reversal: Narcan

Sudafed

Indication: nasal congestion
Class: adrenergic
Labs: N/A
Side Effects: seizure, anxiety, palpitations, anorexia, dizziness
Precautions: assess congestion, monitor pulse, adequate fluid intake

Temazepam

Indication: insomnia
Class: benzodiazepien
Labs: hepatic, renal, CBC
Side effects: dizziness, lethargy, drowsiness, suicidal thoughts, behavioral changes
Precautions: monitor vitals, physical dependence for long term use, assess mental status, assess pt w

Tolnaftate

Indication: fungal skin infection
Class: antifungal topical
Labs: N/A
Side Effects: rash
Precautions: N/A

Vancomycin

Indication: super bugs
Class: anti-infective
Labs: renal, CBC, levels in blood
Side Effects: nephrotoxicity, ototoxicity, anaphylaxis, hypotension
Precautions: monitor urinary output and color, monitor hearing, monitor vitals, monitor infection

Seroquel

Indication: schizophrenia, depressive episodes
Class: antipsychotic
Labs: anemia labs, WBC, hepatic, blood glucose, cholesterol
Side Effects: seizure, dizziness, weight gain, palpitations, hyperglycemia
Precautions: mental status monitor, monitor BMI/weig

Remeron

Indication: major depression
Class: anti-depressant
Labs: CBC, hepatic
Side Effects: drowsiness, constipation, weight gain
Precautions: assess mental status, monitor weight, monitor for seizures

Klonipin

Indication: seizures, panic disorders
Class: anticonvulsant, benzo
Labs: hepatic, renal, CBC
Side effects: dizziness, lethargy, drowsiness, suicidal thoughts, behavioral changes
Precautions: monitor vitals, physical dependence, monitor seizures, for long

Doxorubicin (Adiamycin)

Indication: in combination treatment for tumors
Class: cancer drug
Labs: CBC, platelets, WBC, renal, hepatic, uric acid
Side Effects: red color urine, diarrhea, alopecia, vomiting, anemia, leucopenia, thrombocytopenia
Precautions: monitor vitals, monitor

Albuterol

Indications: used as a bronchodialator to control and prevent reversible air way obstruction caused by asthma or COPDClass-Therapeutic: bronchodialators
Lab to monitor: Serum Potassium levels
Assessments: lung sounds, pulse, BP before administration and d

Ceftriazone (Rocephin)

Indications: treatment for skin infection, urinary infections, respiratory tract infections, meningitis, septicemia, intra abdominal infections, otitis media, lyme disease
Class-Therapeutic: anti-infectives
Lab to monitor: may increase serum AST, ALT, LDH

Levofloxacin (Levaquin)

Indications: treatment of the following bacterial infections: urinary tract, gonorrhea, gynecological infections, respiratory tract, skin, bone/joint, GI, post exposure treatment
Class-Therapeutic: anti-infectives
Lab to monitor: liver function tests
Moni

Heparin

Dilution: undiluted
Continuous: 25,000 units with 250-500 ml of 0.9% NaCl or D5W
Rate: over at least 1 min
Continuous: adjust to maintain therapeutic aPTT, infuse approx 1,000 units/hr
Concentration: varies
Continuous: 50-100 units/ml
Compatibility: y-sit

Heparin

Lab to monitor: monitor aPTT and hematocrit periodically during therapy
Monitor platelet count 2-3 days throughout therapy
Monitor potassium, AST, ALT
Assessments:
Assess for signs of bleeding and hemorrhage
Assess for evidence of additional thrombus
Moni

Heparin

Side effects: bleeding, anemia, thrombocytopenia, alopecia, hypersensitivity
Risk of bleeding increases with the use of drugs that affect platelet function like Aspirin, NSAID, and some penicillins
Concurrent use of thrombolytics increase risk of bleeding

Warfarin

Action: interferes with hepatic synthesis of vitamin K dependant clotting factors to prevent thromembolic events, extrinsic pathway
Indications: prophylaxis and treatment of venous thrombis, pulmonary embolism, a-fib, and management of MI.
Class-Therapeut

Warfarin

Lab to monitor:
Monitor PT, INR, and clotting factors want PT 1.3-1.5 and INR 2.5-3.0
Monitor hepatic function and CBC
Monitor stool and urine for occult blood
Assessments:
Assess for signs of bleeding and hemorrhage
Monitor Blood Pressure
Geri: patients

Lisinopril

Indication: management of hypertension and CHF
Class: ACE
Lab to monitor:
Monitor BUN, creatine, and electrolyte levels, (K)
Monitor Na levels
Monitor CBC periodically hematocrit or hemoglobin may decrease
Monitor AST, ALT, serum bilirubin, uric acid, glu

Atrovastatin (Lipitor)

Indication: primary prevention to decrease risk MI/Stroke, lowers cholesterol
Class: lipid lowering agent
Lab to monitor: serum cholesterol, triglycerides, liver function tests, if develop muscle tenderness monitor CK levels if > 10 D/C therapy
Specific p

Amiodarone

Indication: arrhythmias, tachycardia, ventricular fibrillation, cardiopulomanry resuscitation
Rate: Direct -- push, 300mg first does then 150mg next dose
Intermittent -- over 10 min
Continuous --- 1 mg/min for first 6 hr then decrease to 0.5mg/min
Concent

Amiodarone

Lab to monitor: liver and thyroid functions before and every 6 months during therapy
Monitor AST, ALT, and alkane phosphatase
Side effects: ARD, pulmonary fibrosis, CHF, bradycardia, dizziness, headache, neuropathy, tremor, ataxia, photosensitivity, hypot

Clopidogrel (Plavix)

Indication: reduction of atherosclerosis events
Class: anti-platelet
Lab to monitor:
Bleeding time
CBC
Platelet count
Bilirubin, hepatic enzymes, cholesterol, non protein nitrogen, uric acid (may increase)
Side effects: depression, cough, GI bleeding, Neu

Salicylism signs

tinnitus
dizziness
hyperpnea
psychological disturbances

Mafenide acetate (Sulfamylon) needs to be monitored for what systemic effect?

hyperventilation
-d/c med for 1-2 days

Isotretinoin (Accutane) needs to be monitored for____ levels bc it can elevate _______ levels.

triglyceride

Bleomycin is an antineoplastic med that can cause interstitial pneumonitis which can progress to_____ thus monitor ____ function studies as well as hematological, hepatic, & renal function tests.

pulmonary fibrosis; pulmonary

Busulfan is given to a pt w/ acute myelocytic leukemia; what needs to be monitored?

uric acid level

Hyperuricemia can produce

uric acid
nephropathy
renal stones
acute renal failure

main SE of etoposide (Toposar) given to pt w/ small cell lung cancer

orthostatic hypotension

Major SE of Vincristine (Oncovin, Vincasar) given to pt w/ ovarian cancer

peripheral neuropathy

Asparaginase (Elspar), an antineoplastic med is contraindicated for pts w/

hypersensitivity
pancreatitis
hx of pancreatitis

what tests need to be done w/ Asparaginase (Elspar), an antineoplastic med

pancreatic function tests

signs of pancreatitis

n/v
abdominal pain

during treatment of Tamoxifen given to breast cancer pt, monitior what

calcium
cholesterol
triglceride levels
CBC
platelet

during treatment of Tamoxifen given to breast cancer pt, watch for

hypercalcemia

signs of hypercalcemia

increased urine volume
excessive thirst
n/v
constipation
hypotonicity of muscles
deep bone pain
flank pain

Megestrol acetate (Megace), an antineoplastic med is used w/ caution in pts w/

thrombophlebitis

what is common following treatment for leukemias & lymphomas bc chemotherapy results in massive cell kill

hyperuricemia or increased uric acid level

Cyclophosphamide for breast cancer needs to be monitored for

hematuria

adverse effect of Cyclophosphamide for breast cancer

Hemorrhagic cystitis

Glimepiride (Amaryl) and alcohol can cause what reaction

disulfiram like reaction

disulfiram reaction

flushing
palpitations
nausea

Exenatide (Byetta) is an incretin mimetic used only for

DM type 2

Desmopressin acetate is prescribed for diabetes insipidus what is the therapeutic response of the med?

decreased urinary output

adverse result of Desmopressin acetate

water intoxication aka overhydration
hyponatremia

earl signs of water intoxification (overhydration, hyponatremia)

drowsiness
listelessness
headache

all signs of water intoxification (overhydration, hyponatremia)

drowsiness
listelessness
headache
decreased urination
rapid weight gain
confusion
seizures
coma

Levothyroxine (Synthroid) needs to be taken

on an empty stomach

Excessive doses of levothyroxine (Synthroid) produce signs of

hyperthyroidism

Hyperthyoidism signs

tachycardia
chest pain
tremors
nervousness
insomnia
hyperthermia
heat intolerance
sweating

Hypothyroidism signs

fatigue
cold intolerance
excessively dry skin

Propylthiouracil (PTU) is used to treat

hyperthyroidism or Grave's disease

Myexdema indicates

hypothyroidism

best time to take corticosteroids

before 9 am or early morning

glucocorticoids elevate

blood glucose levels

pts w/ DM taking insulin or oral hypoglycemics and taking glucocorticoid therapy will need

their insulin or oral hypoglycemic meds increased

common SE of Metformin is

diarrhea

Repaglinife (Prandin) common SE

hypoglycemia

pt w/ Crohn's disease takes infliximab (Remicade) how can the RN determine the effectiveness of treatment?

check frequency & consistency of bowel movements which inflammation in the colon will reduce thus reducing diarrhea

loperamide hydrochloride (Imodium) is used to treat

diarrhea

Cimetidine, histamine (H2) receptor antagonist, most common SE in older/elderly pt

confusion

Sucralfate (Carafate), gastric protectant, should be given

one hour before meals & at bedtime;
time is given to allow a protective coating over the ulcer before food intake stimulates gastric acid production & mechanical irritation

Misoprostol, gastric protectant, is given specifically to pts who use what drugs chronically?

NSAIDS

what is included in the triple therapy for Helicobacter pylori infection

2 antibacterial drugs & a proton pump inhibitor like Esomeprazole (Nexium)

Metoclopramide (Reglan) is an antiemetic as well as a GI stimulant; bc it is a GI stimulant , this med is not given to pts w/

GI obstruction
hemorrhage
perforation

Cholestyramine (Questran) is a bile acid sequestrant used to lower the cholestrol level and pt compliance is low due to

poor taste and patability

H2 receptor antagonists suppress secretion of GI acid, alleviate symptoms of heartburn and assist in preventing complications of peptic ulcer disease.
These meds have an ending of

tidine

Proton pump inhibitors have an ending of

prazole

Guaifenesin (Mucinex) needs to be monitored for _____ that lasts longer than 1 week or is accompanied by fever, rash, sore throat, persistent headache; notify MD

cough

The uses of Diphenhydramine (Benadryl)

antihistamine
antitussive (cough suppressant)
antidyskinetic
sedative-hypnotic

Cromolyn sodium (Intal) is an inhaled nonsteroidal antiallerg agent & mast cell stabilizer; what is the major adverse effect?

Bronchospasm
Cough
Nasal congestion
throat irritation
wheezing

Terbutaline is a bronchodilator and is used w/ caution in

impaired cardiac function
DM
HTN
Hyperthryroidism
Hx of seizures

Zafirlukast (Accolate) is a leukotriene receptor antagonists used in prophylaxis and long term treatment of bronchial asthma; what lab test is done before admin?

Liver function tests

common SE of INH

peripheral neuritis

peripheral neuritis signs

numbness
tingling
paresthesias in extremities

SE of peripheral neuritis from INH can be minimized w/

vitamin B6

INH is hepatotoxic; what are signs of hepatitis

yellow skin
yellow sclera

What is important to know about pt taking Rifampin (Ridadin) long term?

causes orange discoloration of sweat, tears, urine, feces.

Ethambutol (Myambutol) causes optic neuritis which does what

decreases visual acuity & the ability to discriminate between the colors red and green.

Streptomycin -major thing to look for?

impaired sense of hearing

before starting INH, the nurse gathers a baseline of what lab?

liver enzyme levels

Theophylline is a methylxanthine bronchodilator and needs to limit xanthine containing foods like

coffee
cola
chocolate

Omalizumab is an anti-inflammatory used for long term control of asthma;
what major reaction can occur w/ admin of this med?

anaphylactic reactions

therapeutic Digoxin (Lanoxin) level

0.5-2 ng/mL

Procainamide (Procanbid) given for cardiac dysrhythmia;
what are signs of toxicity of Procainamide (Procanbid)?

confusion
dizziness
drowsiness
decreased urination
n/v
tachydysrhythmias

Propranolol (Inderal) major adverse reaction?

Bronchospasm

What manifestation in a COPD or asthma pt taking Propanolol (Inderal), a beta blocker, show that they have bronchospasm?

audible expiratory wheezes

major complication of thrombolytic med?

hemorrhage

Before giving a thiazide diuretic like hydrochlorothiazide ask if they are allergic to?

sulfa

Pts taking a thiazide diuretic like hydrochlorothiazide are at risk for

hypokalemia
hyperglycemia
hypercalcemia
hyperlipidemia
hyperuricemia

Cholestyramine (Questran) & ________ (Niacin) need to be avoided

nicotinic acid

how can flushing that is caused by Nicotinic acid (Niacin) given for hyperlipidemia be prevented?

Aspirin or NSAID like Ibuprofen (Motrin) taken 30 min before med

early signs of Digoxin toxicity

double vision
loss of appetite or anorexia
nausea/ vomiting
diarrhea

all signs of digoxin toxicity, a cardiac glycoside

double vision
loss of appetite
nausea
bradycardia
difficulty reading
green or yellow vision
seeing spots
halos
confusion
vomiting
diarrhea
decreased libido
impotence

common SE of Bumetanide (Bumex)

hypotension; thus monitor BP

antidote for heparin

protamine sulfate

antidote for Coumadin

vitamin K

antidote for aminocaproic acid

thrombolytic therapy

what is given for potassium deficit

potassium chloride

Thrombolytic therapy is contraindicated in what conditions

risk of uncontrolled bleeding
severe uncontrolled HTN

before intiating thrombolytic therapy an important vital signs is

blood pressure

What is the effect of Nalidixic acid (Neg gram) and warfarin sodium (Coumadin)?

Nalidixic acid intensifies the effects of the oral anticoagulant.

W/ Nalidixic acid, Coumadin should be

decreased

Sulfisoxazole needs to be given w/

full glass of water

Trimethoprim- sulfamethoxazole (Bactrim) ; most important thing to look for

blood disorders

early signs of blood disorders

sore throat
fever
pallor

Bethanechol chloride (Urecholine) can be hazardous to pts w/

urinary tract obstruction
weakness of the bladder wall
bc elevation of pressure within the urinary tract could rupture the bladder in pts w/ these conditions

Overdose (toxicity) of bethanechol chloride manifestations

excessive muscarinic stimulation :
-salivation
- sweating
- involuntary urination
- defecation
- bradycardia
- severe hypotension

treatment of Bethanechol chloride (Urecholine)

atropine sulfate subc or IV

excessive muscarinic stimulation signs

-salivation
- sweating
- involuntary urination
- defecation
- bradycardia
- severe hypotension

Overdose (Toxicity) of oxybutnin chloride

Hypotension or Hypertension
Confusion
Tachycardia
Flushed or red face
Signs of Resp Depression
CNS excitation like
-nervousness
-hallucinations
-irritability

Cyclosporine (Sandimmune) given after kidney transplantation needs to be monitored for nephrotoxicity;
what labs are monitored for signs of nephrotoxicity?

Elevated blood urea nitrogen
Elevated creatinine levels

Tacrolimus (Prograf) is used in caution in pts w/

immunosuppressed
renal function impairment
hepatic function impairment
pancreatic function impairment

Therapeutic effect of Epoetin alfa (Epogen, Procrit) ??
Epoetin alfa is used to reverse anemia associated w/ chronic renal failure

hematocrit between 30-33%

With high Carbidopa-levodopa (Sinemet) levels what can occur?

Dyskinesia and impaired voluntary movement

Phenytoin and birth control pills: what do you need to know?

Phenytoin can decrease the effectiveness of birth control pills

antidote for acetaminophen (Tylenol)

acetylcysteine (Mucomyst)

normal acetaminophen (Tyelnol) therapeutic serum level

10-20 mcg/mL

What is the gold preparation used to treat rheumatoid arthritis?

Auranofin (Ridaura)

normal direct bilirubin level

0-0.3 mg/dL

normal platelet count

150,000 - 400,000 mm

normal prothrombin time

10-13 seconds

phenytoin therapeutic level

10-20 mcg/mL

phenytoin level of 20 mcg/mL or higher the pt will show signs of

nystagmus (involuntar movements of the eyeballs)

phenytoin level of 30 mcg/mL or higher the pt will show signs of

ataxia & slurred speech

Mild intoxication w/ acetylsalicylic acid (aspirin) is called salicylism and is experienced when daily dose is higher than

4 g or 4000 mg

what is the systemic manifestation in a pt w/ salicylism

hyperventilation

what is the major adverse effects of carbamazepine (Tegretol)

blood dyscrasias

what do blood dyscrasias include?

aplastic anemia
agranulocytosis
thrombocytopenia
thrombophlebitis
dysrhythmias
dermatological effects

antacids are given

after meals

oral steroids are given

with meals to decrease GI irritation

cholinergic crisis when there is

an overdose of Edrophoium (Tensilon) med

A pt w/ Myasthenia gravis becomes increasingly weaker and is given Edrophonium (Tensilon) and the symptoms improve.. what crisis is this?

Myasthenic crisis

Major SE of Etabercept (Enbrel) for rheumatoid arthritis

infection & pancytopenia; monitor WBC counts & platelets

most important nursing intervention for Allopurinol (Zyloprim)

drink 3000 mL of fluid daily

Colchicine given for gout is used w/ caution in pts w/

old age
debilitated pts
pts w/ cardiac disease
renal disease
GI disease

Major adverse effect of Alendronate (Fosamax)

esophageal irritation

Major teaching w/ Alendronate (Fosamax)

Do not eat or drink anything for 30 min after meds; do not lie down; take med w/ full glass of water after rising in the morning

Dantrolene sodium (Dantrium) major adverse effect

liver damage; thus monitor liver function tests

Cyclobenzaprine hydrochloride (Flexeril) for muscle spasms has anticholinergic effects and is used in caution w/

urinary retention
glaucoma
increased intraocular pressure

IV admin of Methocarbarnol (Robaxin) for multiple sclerosis can cause

hypotension
bradycardia

Major adverse effect of Clozapine (Clozaril)

agranulocytosis
monitor WBC count
treatment is stopped if WBC under 3000

Tardive dyskinesia signs

uncontrollable involuntar movements of the body & extremities, particulary the tongue

Parkinsonism signs

tremors
mask-like facies
rigidity
shuffling gait

hypertensive crisis signs

hypertension
occipital headache
readiating frontanlly
neck stiffness
soreness
n/v

MAOI major adverse effect

hypertensive crisis

neuroliptic malignant syndrome

Dyspnea or tachypnea
tachycardia
irregular pulse rate
fever
BP changes
increased sweating
loss of bladder control
skeletal muscle rigidty

neuroliptic malignant syndrome occurs in what meds

antipsychotic

SE of Fluoxetine (Prozac)

CNS and GI dysfunction

maintenance serum levels of lithium are

0.6- 1.2 mEq/L

Clomipramine (Anafranil) is a tricyclic antidepressant used for

OCD

Bupropion (Wellbutrin) causes toxicity when levels are greater than 450 mg/day; what do you need to look for when looking for toxicity

seizure activity

Phenelzine (Nardil) is a Monoamine oxidase inhibitor (MAOI) what type of foods need to be avoided bc it can cause hypertensive crisis?

Tyramine like yogurt, aged cheeses, smoked or processed meats, red wines, and fruits like avocados, raisins, or figs

Warfarin (Coumadin) & Levothyroxine (Synthroid) what dose needs to be changed when in use with the other?

Decrease the Warfarin (Coumadin) dose
bc Levothyroxine (Synthroid) enhances the effects of Warfarin ( Coumadin)

an insulin vile in current use can be kept at room temp for

1 month

most common SE of metformin (Glucophage) is

GI disturbances like
- decreased appetite or anorexia
-nausea
-diarrhea

Desmopressin is an ANTI-diuretic hormone used in treatment of diabetes insipidus; it causes

water reabsorption

Sulfonlureas

promote insulin secretion by the pancreas and may increase tissue response to insulin

Biguanides

decrease glucose production by the liver

Alpha glucosidease inhibitors

inhibit carbohydrate digestion

Thiazolidinediones

decrease insulin resistance

Isoproterenol hydrochloride is a adrenergic bronchodilator ; what are the SE

tachycardia
hypertension
chest pain
dysrhythmias
nervousness
restlessness
headache
increased pulse
increased BP

Bethanechol chloride (Urecholine ) can cause nausea & vomiting when should the pt take the med

2 hours after meals

Fludrocortisone acetate (Florinef Acetate) is given for Addison's disease; what is the primary action

enhances the REABSORPTION of sodium & chloride ions in the distal tubules of the kidney
** promotes water retention

Succimer (Chemet) given for lead poisoning
specifically for Chelatin therapy,what needs to be monitored and what lab?

renal function bc med is excreted by kidneys
monitor BUN level

Ribavirin given to a HOSPITALIZED child w/ respiratory syncytial virus (RSV) cannot be given orally, intramuscularly or subc, how is it given?

hood
face mask
oxygen tent

for child w/ HIV who have a positive Mantoux test result, what is the minimum months of treatment?

12 months

Corticosteroid therapy can cause

calcium depletion
potassium depletion
sodium retention
glucose intolerance
increased blood glucose levels

What should the nurse do innitially if there are crystal formations in the mannitol solution?

Place the vial in warm water
**bc it will quickly dissolve if the container is placed in warm water then cooled to body temp before admin

What should the nurse do if after placing the mannitol w/ crystal formation does not dissolve in warm water?

return to pharm; do not use

Ataxia from ototoxicity reflects damage from what cranial nerve

eighth

Lindane is prescribed for scabies; what preexisiting condition is high risk for adverse effect of seizures?

seizure hx

What med is used to treat known or suspected ethylene glycol (antifreeze) intoxication?

Fomepizole (Antizol)

Fomepizole (Antizol) given for treatment of known or suspected ethylene glycol (antifreeze) intoxication;
The RN recieves the med and it has solidified, what to do?

run the vial under warm water

Fomepizole (Antizol) given for treatment of known or suspected ethylene glycol (antifreeze) intoxication is given w/ what type of IV fluid over a 30 min period?

isotonic
0.9% Normal saline
5% dextrose in water

Irbesartan (Avapro) given for Hypertension is what type of med?

Angiotension II type 1 receptor antagonist

what is used to treat a bladder spasm after prostatectomy?

Antispasmodic meds like
Belladonna & opium (B&O) suppository
OR
propantheline bromide (Pro-Banthine)

Epoetin alfa (epogen, procrit) given to treat anemia in renal failure can be given IV or

subcutaneously

what gauge is used for heparin or enoxaparin subc?

25-27

what size needle is used for heparin or enoxaparin subc?

5/8 inch

pril" is the ending for drugs that are

ACE inhibitors

serious SE of ACE inhibitor drugs

hypotension
monitor BP

when are thrombolytics like streptokinase most effective?

within 4-6 hours after onset of symptoms

What is the antidote for Warfarin (Coumadin)?

Phytonadione a.k.a Vitamin K

Ticlopidine (Ticlid) is an antiplatelet med given to prevent thrombotic stroke
what is the major adverse effect of Ticlopidine (Ticlid)?

neutropenia

neutropenia

abnormally small number of MATURE white blood cells

A pt given Ticlopidine (Ticlid) needs to be monitored for neutropenia development thus a nurse should monitor

CBC count

Before giving Ticlopidine (Ticlid) what baseline labs does a RN need?

CBC count

Ticlopidine (Ticlid) can cause GI SE and should be given

with meals

neutropenia is aka?

agranulocytosis

pt taking Ticlopidine (Ticlid) will often develop neutropenia within

first 3 months of therapy

Phenobarbital sodium (Luminal) is both a

barbiturate & antiseizure med

Dantrolene sodium (Dantrium) given for spasticity, what is the oral maintenance dosage?

100 mg two to four times daily

a pt being changed from oral to inhalation glucocorticoids could experience signs of

adrenal insufficiency

adrenal insufficiency signs

anorexia
nausea
weakness
fatigue
hypotension
hypoglycemia

signs of opiod withdrawl

increased temp
increased BP
abdominal cramping
vomiting
restlessness

Food can hamper absorption w/ Norfloxacin (Noroxin) given for UTI, when should it be given?

1 hour before meals
OR 2 hours after meals

Nitrofurantoin (macrodantin) given for acute UTI , what is the adult dosage

50 mg every 6 hours

Nitrofurantoin (macrodantin) given for acute UTI , what is the adult dosage for prophylaxis or recurrent UTI?

50-100m mg at bedtime

Nalidixic acid (Neg Gram) given for UTI , what are normal adult dosage for this med?

1 g four times daily for a period of 1 week.

normal adult dosage for Bethanechol chloride (Urecholine) for post-op bladder spasm

10-50 mg three to four times a day

the injectable form of Bethanechol chloride (Urecholine) is given

subc

most significant adverse effect of Epoetin alfa (Epogen, Procrit) is?

Hypertension

Epoetin alfa (Epogen, Procrit) can show a therapeutic effect in

increase in hematocrit will show in 2 weeks after therapy

Aluminum hydroxide (Amphojel), an antacid, is given

w/ meals

Major SE of Aluminum hydroxide (Amphojel), an antacid, is

constipation

Ferrous sulfate (Feosol) is an iron supplement used to treat anemia what is the common SE?

constipation

what opioid analgesic is given to pts SPECIFICALLY after craniotomy & why?

Codeine sulfate
bc it does not alter respirations or mask neurological signs as do other opioids

what is given first: eye drops or eye ointment?

eye drops

Itraconazole (Sporanox) given for fungal infection of the hands needs to be monitored for what adverse effect?

Hepatitis

signs of hepatitis

urine becomes very dark in color
anorexia
abdominal pain
unusual tiredness
weakness
jaundice develops

Kanamycin sulfate (Kantrex) given for respiratory tract infection, what are the adverse reactions?

nephrotoxicity
irreversible ototoxicity
neurotoxicity

nephrotoxicity

increased BUN
increased creatinine

phentoin (Dilantin) is hung with what IV solution?

0.9% sodium chloride (NS)

SE of Meperidine hydrochloride are

respiratory depression
orthostatic hypotension
tachycardia
drowsiness
mental clouding
constipation
urinary retention

what diet is prescribed to prevent constipation?

high fiber diet & stool softners & increase fluid intake

Dantrolene (Dantrium) is a skeletal muscle relaxant and can cause

liver damage; thus monitor liver function studies

Dexamethasone (Decadron) is mixed with what IV solution?

50-100 mL of 0.9% sodium chloride (NS) OR
5% dextrose in water

most important teaching of tetracycline intake

stains teeth; use straw

important teaching of Allopuinol (Zyloprim) intake

encourage 3000 mL to prevent formations of crytsals in urine

Probenecid given for gout to inhibit reabsorption of uric acid by the kidney and promote excretion of uric acid needs to avoid what type of food intake?

High purine foods like
- organ meats
-sardines
-scallops
-anchovies
-broth
-mincemeat
-herring
-shrimp
-gravy
-yeast

Auranofin (Ridaura) is a gold preparation that is given orally for rheumatoid arthritis,
what are the early signs & symptoms of toxic reaction of Auranofin (Ridaura) ?

rash
purple blotches
pruritus
mouth lesions
metallic taste in mouth

Ibuprofen (Motrin) given to an older adult w/ rheumatoid arthritis or osteoarthritis , what is the normal oral dose?

400-800 mg three or four times a day
1200-3200 mg/day

Baclofen (Lioresal) is a skeletal muscle relaxant and decreases muscle spasms in pts w/ spinal cord

a skeletal muscle relaxant and decreases muscle spasms in pts w/ spinal cord

major SE of Baclofen (Lioresal)

urinary retention

Major adverse effect of Asparaginase (Elspar), an antineoplastic med

prolonged blood clotting times

How do you apply Crotamiton (Eurax) used for scabies?

-Massage the med into the skin from the chin downward.
-apply a second application in 24 hours
-followed by a cleansing bath 48 hours after the second applications
-if needed treatment can be repeated in 7 days

Teaching about Metronidazole (Flagl)

darkening of the urine may occur and is harmless

signs of developing pancreatitis

increased serum amylase
increased serum triglycerides
decreased serum calcium
nausea
vomiting
abdominal pain radiating to the back

what major disease can develop due to toxicity of Didanosine (Videx) ?

pancreatitis

Adverse effect w/ Zalcitabine

peripheral neuropathy

Amphotericin B (Fungizone) major adverse reaction

renal impairment
monitor kidney function; watch for elevated serum creatinine

If the serum creatinine level rises above ____ mg/dL, the dose of Amphotericin B (Fungizone) should be reduced

3.5 mg/dL

w/ Bleomycin, it is a priority to assess lung sound and pulmonary function studies bc

pneumonitis can manifest leasing to severe pulmonary fibrosis

Vincristine (Oncovin), anticancer med, major adverse effect?

peripheral neuropathy like weakness and sensory loss in the legs
-decreased refleces
-weakness
-paresthesia
-sensory loss

If GI systems occur Oral erythromycin can be given w/meals but it is BEST given

empty stomach w/ full glass of water

Azithromycin (Zithromax) given for chlamydial infection is given

1 hour before meals or 2 hours after

Bethanechol chloride (Urecholine) is a direct acting muscarinic agonists (cholinergic medication) and can cause

hypotension
vasodilation
bradycardia
excess salivation
increased secretion of gastric acid
abdominal cramps
diarrhea

what is the antidote for bethanechol chloride (Urecholine); a med that causes a cholinergic reaction

atropine sulfate

Atropine sulfate causes mydriasis and cycloplegia and is contraindicated in

narrow angle glaucoma

mydriasis

dilation of the pupils

cycloplegia

relaxation of the ciliary muscles

What is the antidote in a cholinergic crisis?

Atropine sulfate

When performing a Tensilon test, the RN obtains Edrophonium (Tensilon) and what antidote?

Atropine sulfate

Prazosin hydrochloride is an ANTI-hypertensive med , what manifestation determines a benefit of therapy

decreased BP

Clonidine is an ANTI-hypertensive med that is applied to a hairless intact skin area of the upper arm or torso and changed?

every 7 days

Phenytoin (Dilantin) therapeutic serum level

10-20 mcg/mL

IV admin of phenytoin (Dilantin) is performed slowly and no faster than

50 mg/min
**bc rapid admin can cause cardiovascular collapse

Before giving Valproic acid (Depakene), look at what results first?

liver function tests

Before giving Carbamazepine (Tegretol), look at what lab results first?

CBC count
**bc leukopenia, anemia, thrombocytopenia can occur

Dantrolene sodium (Dantrium) major adverse effect?

hepatotoxic

signs of liver dysfunction

jaundice
abdominal pain
malaise

Protamine sulfate is the antidote for

heparin

what is the antidote for heparin?

Protamine sulfate

Atropine sulfate is what type of med

anticholinergic

Ergotamine tartrate (Cafergot) is used

to stop an ongoing migraine attack
OR treat cluster headaches

Spironolactone (Aldactone) is what type of diuretic?

k sparing diuretic

main adverse effect of Spironolactone (Aldactone), a diuretic

Hyperkalemia

Moexipril, an ACE inhibitor, is given

1 hour before meals

What 2 ACE inhibitors are given 1 hour before meals?

Moexipril (Univasc) & Captopril (Capoten)

Major adverse effect w/ Potassium iodide solution (Lugol's solution)?

Iodism

signs of Iodism

brassy taste
burning sensation in the mouth
soreness of the gums & teeth

if signs of Iodism occur when taking potassium iodide solution (Lugol's solution), what should the nurse instruct the pt to do?

report symptom to MD

Abacavir is used to treat

HIV infection

A newly admitted client takes digoxin 0.25 mg/day. The nurse knows that which is the serum therapeutic range for digoxin?
a. 0.1 to 1.5 ng/mL
b. 0.5 to 2.0 ng/mL
c. 1.0 to 2.5 ng/mL
d. 2.0 to 4.0 ng/mL

b. 0.5 to 2.0 ng/mL

The client's serum digoxin level is 3.0 ng/mL. What does the nurse know about this serum digoxin level?
a. It is in the high (elevated) range.
b. It is in the low (decreased) range.
c. It is within the normal range.
d. It is in the low average range.

a. It is in the high (elevated) range.

The nurse is assessing the client for possible evidence of digitalis toxicity. The nurse acknowledges that which is included in the signs and symptoms for digitalis toxicity?
a. Pulse (heart) rate of 100 beats/min
b. Pulse of 72 with an irregular rate
c.

d. Pulse below 60 beats/min and irregular rate

The client is also taking a diuretic that decreases her potassium level. The nurse expects that a low potassium level (hypokalemia) could have what effect on the digoxin?
a. Increase the serum digoxin sensitivity level
b. Decrease the serum digoxin sensit

a. Increase the serum digoxin sensitivity level

When a client first takes a nitrate, the nurse expects which symptom that often occurs?
a. Nausea and vomiting
b. Headaches
c. Stomach cramps
d. Irregular pulse rate

b. Headaches

The nurse acknowledges that beta blockers are as effective as antianginals because they do what?
a. Increase oxygen to the systemic circulation.
b. Maintain heart rate and blood pressure.
c. Decrease heart rate and decrease myocardial contractility.
d. De

c. Decrease heart rate and decrease myocardial contractility.

The health care provider is planning to discontinue a client's beta blocker. What instruction should the nurse give the client regarding the beta blocker?
a. The beta blocker should be abruptly stopped when another cardiac drug is prescribed.
b. The beta

b. The beta blocker should NOT be abruptly stopped; the dose should be tapered down.

The beta blocker acebutolol (Sectral) is prescribed for dysrhythmias. The nurse knows that what is the primary purpose of the drug?
a. To increase the beta1 and beta2 receptors in the cardiac tissues
b. To increase the flow of oxygen to the cardiac tissue

c. To block the beta1-adrenergic receptors in the cardiac tissues

A client is taking hydrochlorothiazide 50 mg/day and digoxin 0.25 mg/day. What type of electrolyte imbalance does the nurse expect to occur?
a. Hypocalcemia
b. Hypokalemia
c. Hyperkalemia
d. Hypermagnesemia

b. Hypokalemia

What would cause the same client's electrolyte imbalance?
a. High dose of digoxin
b. Digoxin taken daily
c. Hydrochlorothiazide
d. Low dose of hydrochlorothiazide

c. Hydrochlorothiazide

A nurse is teaching a client who has diabetes mellitus and is taking hydrochlorothiazide 50 mg/day. The teaching should include the importance of monitoring which levels?
a. Hemoglobin and hematocrit
b. Blood urea nitrogen (BUN)
c. Arterial blood gases
d.

d. Serum glucose (sugar)

A client has heart failure and is prescribed Lasix. The nurse is aware that furosemide (Lasix) is what kind of drug?
a. Thiazide diuretic
b. Osmotic diuretic
c. High-ceiling (loop) diuretic
d. Potassium-sparing diuretic

c. High-ceiling (loop) diuretic

The nurse acknowledges that which condition could occur when taking furosemide?
a. Hypokalemia
b. Hyperkalemia
c. Hypoglycemia
d. Hypermagnesemia

a. Hypokalemia

For the client taking a diuretic, a combination such as triamterene and hydrochlorothiazide may be prescribed. The nurse realizes that this combination is ordered for which purpose?
a. To decrease the serum potassium level
b. To increase the serum potassi

b. To increase the serum potassium level

The client has been receiving spironolactone (Aldactone) 50 mg/day for heart failure. The nurse should closely monitor the client for which condition?
a. Hypokalemia
b. Hyperkalemia
c. Hypoglycemia
d. Hypermagnesemia

b. Hyperkalemia

A client who has angina is prescribed nitroglycerin. The nurse reviews which appropriate nursing interventions for nitroglycerin? (Select all that apply.)
a. Have client lie down when taking a nitroglycerin sublingual tablet.
b. Teach client to repeat tak

a. Have client lie down when taking a nitroglycerin sublingual tablet.
b. Teach client to repeat taking a tablet in 5 minutes if chest pain persists.
e. Warn client against ingesting alcohol while taking nitroglycerin.

A client's blood pressure (BP) is 145/90. According to the guidelines for determining hypertension, the nurse realizes that the client's BP is at which stage?
a. Normal
b. Prehypertension
c. Stage 1 hypertension
d. Stage 2 hypertension

c. Stage 1 hypertension

The nurse acknowledges that the first-line drug for treating this client's blood pressure might be which drug?
a. Diuretic
b. Alpha blocker
c. ACE inhibitor
d. Alpha/beta blocker

a. Diuretic

The nurse is aware that which group(s) of antihypertensive drugs are less effective in African-American clients?
a. Diuretics
b. Calcium channel blockers and vasodilators
c. Beta blockers and ACE inhibitors
d. Alpha blockers

c. Beta blockers and ACE inhibitors

The nurse knows that which diuretic is most frequently combined with an antihypertensive drug?
a. chlorthalidone
b. hydrochlorothiazide
c. bendroflumethiazide
d. potassium-sparing diuretic

b. hydrochlorothiazide

The nurse explains that which beta blocker category is preferred for treating hypertension?
a. Beta1 blocker
b. Beta2 blocker
c. Beta1 and beta2 blockers
d. Beta2 and beta3 blockers

a. Beta1 blocker

Captopril (Capoten) has been ordered for a client. The nurse teaches the client that ACE inhibitors have which common side effects?
a. Nausea and vomiting
b. Dizziness and headaches
c. Upset stomach
d. Constant, irritating cough

d. Constant, irritating cough

A client is prescribed losartan (Cozaar). The nurse teaches the client that an angiotensin II receptor blocker (ARB) acts by doing what?
a. Inhibiting angiotensin-converting enzyme
b. Blocking angiotensin II from AT1 receptors
c. Preventing the release of

b. Blocking angiotensin II from AT1 receptors

During an admission assessment, the client states that she takes amlodipine (Norvasc). The nurse wishes to determine whether or not the client has any common side effects of a calcium channel blocker. The nurse asks the client if she has which signs and s

b. Dizziness
c. Headache
e. Ankle edema

When a newly admitted client is placed on heparin, the nurse acknowledges that heparin is effective for preventing new clot formation in clients who have which disorder(s)? (Select all that apply.)
a. Coronary thrombosis
b. Acute myocardial infarction
c.

a. Coronary thrombosis
b. Acute myocardial infarction
c. Deep vein thrombosis (DVT)
d. Cerebrovascular accident (CVA) (stroke)
e. Venous disorders

A client who received heparin begins to bleed, and the physician calls for the antidote. The nurse knows that which is the antidote for heparin?
a. protamine sulfate
b. vitamin K
c. aminocaproic acid
d. vitamin C

a. protamine sulfate

A client is prescribed enoxaparin (Lovenox). The nurse knows that low-molecular-weight heparin (LMWH) has what kind of half-life?
a. A longer half-life than heparin
b. A shorter half-life than heparin
c. The same half-life as heparin
d. A four-times short

a. A longer half-life than heparin

The nurse is teaching a client about clopidogrel (Plavix). What is important information to include?
a. Constipation may occur.
b. Hypotension may occur.
c. Bleeding may increase when taken with aspirin.
d. Normal dose is 25 mg tablet per day.

c. Bleeding may increase when taken with aspirin.

A client is prescribed dalteparin (Fragmin). LMWH is administered via which route?
a. Intravenously
b. Intramuscularly
c. Intradermally
d. Subcutaneously

d. Subcutaneously

A client is being changed from an injectable anticoagulant to an oral anticoagulant. Which anticoagulant does the nurse realize is administered orally?
a. enoxaparin sodium (Lovenox)
b. warfarin (Coumadin)
c. bivalirudin (Angiomax)
d. lepirudin (Refludan)

b. warfarin (Coumadin)

A client is taking warfarin 5 mg/day for atrial fibrillation. The client's international normalized ratio (INR) is 3.8. The nurse would consider the INR to be what?
a. Within normal range
b. Elevated INR range
c. Low INR range
d. Low average INR range

b. Elevated INR range

Cilostazol (Pletal) is being prescribed for a client with coronary artery disease. The nurse knows that which is the major purpose for antiplatelet drug therapy?
a. To dissolve the blood clot
b. To decrease tissue necrosis
c. To inhibit hepatic synthesis

d. To suppress platelet aggregation

A client is to undergo a coronary angioplasty. The nurse acknowledges that which drug is used primarily for preventing reocclusion of coronary arteries following a coronary angioplasty?
a. clopidogrel (Plavix)
b. abciximab (ReoPro)
c. warfarin (Coumadin)

b. abciximab (ReoPro)

A client is admitted to the emergency department with an acute myocardial infarction. Which drug category does the nurse expect to be given to the client early for the prevention of tissue necrosis following blood clot blockage in a coronary or cerebral a

c. Thrombolytic agent

A client has a serum cholesterol level of 265 mg/dL, triglyceride level of 235 mg/dL, and LDL of 180 mg/dL. What do these serum levels indicate?
a. Hypolipidemia
b. Normolipidemia
c. Hyperlipidemia
d. Alipidemia

c. Hyperlipidemia

The nurse knows that the client's cholesterol level should be within which range?
a. 150 to 200 mg/dL
b. 200 to 225 mg/dL
c. 225 to 250 mg/dL
d. Greater than 250 mg/dL

a. 150 to 200 mg/dL

A client's high-density lipoprotein (HDL) is 60 mg/dL. What does the nurse acknowledge concerning this level?
a. It is lower than the desired level of HDL.
b. It is the desired level of HDL.
c. It is higher than the desired level of HDL.
d. It is a much l

b. It is the desired level of HDL.

The nurse realizes that which is the laboratory test ordered to determine the presence of the amino acid that can contribute to cardiovascular disease and stroke?
a. antidiuretic hormone
b. homocysteine
c. ceruloplasmin
d. cryoglobulin

b. homocysteine

A client is taking lovastatin (Mevacor). Which serum level is most important for the nurse to monitor?
a. Blood urea nitrogen
b. Complete blood count
c. Cardiac enzymes
d. Liver enzymes

d. Liver enzymes

The client is taking rosuvastatin (Crestor). What severe skeletal muscle adverse reaction should the nurse observe for?
a. Myasthenia gravis
b. Rhabdomyolysis
c. Dyskinesia
d. Agranulocytosis

b. Rhabdomyolysis

When a client is taking ezetimibe (Zetia), she asks the nurse how it works. The nurse should explain that Zetia does what?
a. Inhibits absorption of dietary cholesterol in the intestines.
b. Binds with bile acids in the intestines to reduce LDL levels.
c.

a. Inhibits absorption of dietary cholesterol in the intestines.

A client is diagnosed with peripheral arterial disease (PAD). He is prescribed isoxsuprine (Vasodilan). The nurse acknowledges that isoxsuprine does what? (Select all that apply.)
a. Relaxes the arterial walls within the skeletal muscles
b. May cause hypo

a. Relaxes the arterial walls within the skeletal muscles
b. May cause hypotension, chest pain, and palpitations

What are the symptoms of a panic attack?

Crying, flushed face, trembling hands, diaphoretic, trouble breathing, heart is fluttering, feels as if going crazy, increased HR, BP, and respirations.

What should be done immediately for someone experiencing a panic attack?

Reassure them that they are safe and that you'll stay with them until their symptoms resolve (most last for 10 mins although they can last for than an hour). Reassure them that symptoms won't harm them. COACH them to take slow deep breaths, inhaling throu

Should an LPN be assigned to a person experiencing a panic attack?

NO!!!

What drug might be given to a person experiencing a panic attack? Teaching?

Paxil. The pt should be taught that they shouldnt drive the 1st time they take it, it takes 2 weeks to work, and stress reduction techniques to use until medicine kicks in.

What is behavior modification? What should you reward with? What should you not reward with?

It is changing behaviors by reward the good and ignoring the bad. You should reward with your time (story,play). You should not reward with food.

What is milieu therapy?

Use the environment/peer pressure to change a person's behaviors. Patients develop the rules/punishments. (A controlling nurse for this type of therapy isn't appropriate).

What is agoraphobia?

A fear of open spaces or leaving their home. They fail to interact with others and are never w/o symptoms.

How do you know therapy for agoraphobia is working?

They are out shopping or initiating conversation!

What should you do with someone who is admitted with obsessive compulsive disorder?

Ask what makes them anxious. Don't interfere with rituals unless harming themselves or others. and interact in a positive way if need to influence.

What should you do with an obsessive compulsive patient who is wringing their hands for hours and they are beginning to bleed?

Give him lotion instead of asking him to stop the ritual.

What is an obsession? EX?

It is a thought. EX: "I'm so bad, I made a mistake", feeling like someone close to you might die, or you might be harmed.

What is a compulsion? EX?

It is an action in response to an obsession. EX: Washing hands repetitively, counting things, rechecking things 20 times.

What is characteristic of a patient with PTSD?

Tense, HYPERVIGILANT, depressed, or emotionally distant. They may also suddenly believe that a traumatic event is recurring (a flashback) or experience survival guilt, intrusive memories, or psychic numbing.

What should you do immediately for someone experience a PTSD episode?

Reassure the pt that they are safe before he injures himself, you, or others. First try to orient them to their surroundings. Explain who you are, and tell them you're going to turn the lights on so that they can see they are in a hospital and not a jungl

What would be appropriate treatment/coping for PTSD?

Valerian root can help with symptoms (takes 1 1/2 weeks to kick in) and avoiding triggers (stress) can help prevent episodes (not watching army movies).

What characteristics are present with anorexia nervosa?

INTENSE FEAR OF BECOMING FAT, WEIGHT DECREASE BY AT LEAST 25% FROM ORIGINAL BODY WT, body image disturbance (draw fat person even though skinny), no known physical illness, absence of periods, fine hair (lanugo), excessive exercising, cook a lot but don't

What is the main concern with anorexia nervosa?

F&E imbalance and muscle weakness.

What should you do on admission for a person with anorexia nervosa?

Have them sign a contract to eat and retain fluid. Weigh each morning after void.

When would you be extremely concerned about a person with anorexia nervosa?

IF THEY BEGIN TO PURGE ALSO!

What is characteristic of bulimia?

RECURRENT BINGE EATING, awareness of abnormal eating pattern, fear of not being able to stop eating voluntarily, depressed mood following binges, laxative misuse, calloused knuckles, and rotten teeth.

What is characteristic of PICA?

PERSISTENT EATING OF NON-NUTRITIVE SUBSTANCES X 1 MONTH, infants (paint, plaster, cloth), older kids (bugs, rocks, sand), adults (chalk, starch, paper). STASH= Stuff is hidden, Things (hair, clay, starch), All non-food items, Sick-don't digest (iron defic

What is a concern for pregnant women with PICA?

Iron deficiency anemia (ALL AGE GROUPS). Tx= foods high in iron (legumes, veggies, raisins). Give with vitamin C to help absorb.

What is Dysthmia?

Chronic depression that is less severe than major depression and can occur at all ages.

What is the medical treatment for depression?

Meds (MAOIs, Tricyclics) and ECT

What is ECT? What are interventions that are carried out for someone undergoing ECT?

Electroconvulsive therapy: planned controlled seizures to assist depression. The pt should be NPO, atropine to dry secretions (drooling) but not for a glaucoma patient, sleep 5-6 hours afterwards, and may have some memory loss. A Float nurse would be assi

Who is at risk for suicide as a result of depression? Evaluate risk?

SUBSTANCE ABUSERS, PEOPLE WHO LIVE ALONE, adolescents and elderly, recent crisis, chronic illness, previous attempts, identity crisis, hallucinating (voices telling them to harm themselves). (Use Sad Persons Scale to Evaluate).

What interventions should be implemented for someone at risk for suicide?

Safety Precautions: Remove harmful objects, Close observation, 1 on 1, and Written contract.

What types of meaningful socialization would be good for a depressed patient?

NO GROUP ACTIVITIES bc they hate to be in groups. NO competitive activities bc may decrease self esteem more. They will do better with puzzles, music, or activities/hobbies of their choice. These should help increase their self esteem.

What are the two drugs to know for depression?

MAOIs and Tricyclics

What's important to know about MAOIs?

NO Foods with tyramine should be consumed during therapy and for 2 weeks after treatment is discontinued. May cause hypotension.

What is important to know about Tricyclics?

It takes 2-4 weeks before the drug effects kick in so teach relaxation techniques to utilize in the meantime. SE= Dry mouth, constipation (give med before bed, increase fluids, and high fiber diet (NO COLACE!!)). May also cause hypotension.

What are the two components of bipolar disorder?

Manic and Depressive

What is characteristic of the manic phase of bipolar disorder?

Onset is prior to age 30, Mood (ELEVATED, expansive, and irritable), Speech (loud, rapid, running, rhyming, clanging, vulgar), ? WEIGHT LOSS, GRANDIOSE (think they are GOD or can fly), DISTRACTED, HYPERACTIVE, delusions, decreased need for sleep, inapprop

What interventions would be appropriate for a manic patient?

Lithium (.6-1.2), NO GROUPS, high protein/calorie diet to help increase wt (mobile food is impt and NO Caffeine), Treadmill=good activity bc helps with energy (NO PUZZLES), place the patient in a private room or with another manic pt, and make sure to mee

What is characteristic of the depressive part of bipolar disorder?

Previous manic episodes, Mood: dysphoric, depressive, and despairing), decrease interest in pleasure, negative views, fatigue, decreased appetite, constipation, insomnia, libido, suicidal preoccupation, and agitation or movement retardation.

What is the normal lithium level?

0.6-1.2

What factors increase retained lithium?

Diarrhea, diuretics or dehydration, low salt diets, or high fevers/strenuous exercise.

What factors decrease the amount of lithium through enhanced excretion?

High salt intake, high intake of sodium bicarbonate, and pregnancy.

What is important to remember with patients on lithium?

HIKE= Hydration. If have diarrhea could be from elevated lithium level from dehydration as a result of hiking.

What is an appropriate snack for someone on lithium who is going hiking?

Lemonade is good bc they need hydration but can't have caffeine. (No tea, coffee, etc.)

What are the 5 stages of grieving?

Denial, Anger, Bargaining, Depression, and Acceptance.

If a patient in the hospital is talking about good memories from their life time what should be done?

Hospice should be called at this time.

If a psych nurse floated to a cancer unit what type of patient should she receive?

A person who is in the acceptance stage of their disease.

What is juvenile schizophrenia/autism?

When a person has ritualistic behaviors so everyone must follow the plan of care. They self stimulate and don't like loud noises. To relax an autistic child try to de-pressure the situation.

Who is a good roommate for an autistic child?

A person in a coma.

What is characteristic of schizophrenia?

Loss of boundaries, inability to trust, withdrawn or peculiar behavior, indifferent/aloof, love/hate feelings, personality changes, confused chaotic thoughts, retreat to fantasy world, auditory hallucinations and delusions, hypersensitivity to sound & sme

How would someone describe a schizophrenic?

Weird, strange, and different.

What is an important thing that needs to be done with schizophrenics and why?

SET LIMITS with them because otherwise they won't manage their hygiene properly. (Poor hygiene skills). They may also break rules or be unintentionally rude (inform them of the mistake and help them regain control).

What are hallucinations? Ex?

Sensory impressions without external stimuli. Ex: Detox= seeing pink elephants (absolutely not there but seeing).

What are illusions? Ex?

Real stimuli that has been misinterpreted. Ex: Thinking that someone waving at you is trying to stab you (see person just misinterpret).

What is a delusion? Ex?

A false fixed belief. Ex: I am dating Madonna and she's waiting for me in a hotel room!

What are paranoid delusions?

The person thinks that someone is out to harm him/her.

What is the response to someone experiencing a paranoid delusion?

Acknowledge what the person says but don't agree. Hone in on their feelings ("it must be very frightening"= EMPATHY)

What are typical types of delusions that are less frightening than paranoid delusions?

Grandiose and religious

What are the guidelines for maintaining a therapeutic relationship with a schizophrenic patient?

Accept them as they are, monitor of hallucination/ delusions, assess their safety, give appropriate feedback on how interpret communication, SET LIMITS, and encourage them to learn about their mental illness.

What is associative looseness? Ex?

It means a persons one thought isn't connected to the next thought. Ex: Blue lights, I go round and round, the grass is green.

What is a neologism? Ex?

A word or phrase the patient constructs that's meaningless to everyone else. Ex: I bessaton the coaglese and vergified the rest.

What is clang association? Ex?

A meaningless use of rhyming words. Ex: I read the bed and said the head then led Ted to the dead.

What is word salad? Ex?

A mix of words or phrases that have no meaning to the listener. Ex: Animals, fast, food, family, working, birds, loving, the net.

What is the best way to respond to a patient who is using neologisms, clang association, or word salad?

Honesty is the best response. If you don't understand then tell them you don't understand but you'd like to help.

What is echolalia? Ex?

Repeating exactly what someone else said to you. Ex: "It's time for your shower", "It's time for your shower".

If a person has a delusion of being GOD, should you challenge them and tell them that he is not GOD he is Joe, etc.?

NO. Challenging a firmly fixed delusion serves no therapeutic purpose, but it could antagonize the patient.

What are 6 common side effects of antipsychotics?

Extrapyramidal sx, orthostatic hypotension, constipation, urinary retention, confusion, sexual dysfunction.

What are extrapyramidal sx?

Parkinson's like symptoms (tremors, drooling, etc.)

What is dystonia?

Face spasms, or awkward twisting of the head/body.

What is tardive dyskinesia?

Irreversible/permanent sticking out of the tongue, lip smacking, or leaning tree syndrome.

What are antipsychotics that commonly cause the 6 side effects listed previously?

LoxaPINE, clozaPINE/ Risperidone, Haldol/Haloperidol, Phenothiazine (DRUG CLASS)- ThroaZINE, Prolixin, ChlorpromaZINE, CompaZINE.

What are the S&S of neuroleptic malignant syndrome?

Diaphoresis, incoherent speech, disorientation, muscle rigidity, fever (101), hypertension, tachycardia, incontinence, tremor, increased salivation, and increased respirations. (present after receiving an antipsychotic medication).

What is neuroleptic malignant syndrome?

It is a rare but potentially fatal adverse reaction to antipsychotic medications that leads to autonomic dysfunction.

What must you do immediately if someone is presenting with neuroleptic malignant syndrome?

Page the house physician and hold all antipsychotic meds. Provide 1 on 1 observation. Closely monitor vital signs, establish IV access for fluids or give oral fluids. Adminsiter an antipyretic because he/she is at risk for SEIZURES d/t hyperthermia (TEMPE

When should you take off a cooling blanket on someone with a fever?

When their temperature reaches 101 bc they will continue to decrease after removal.

What is characteristic of antisocial behavior?

They are like the troll under the bridge. They are unreliable, self-centered, poor judgement, conflicts with society, LACK OF REMORSE (love em and leave em), MANIPULATIVE, blames others, and splits the staff.

What interventions are used for someone with antisocial behavior?

ASSIGN THE MOST SKILLED NURSE TO THIS MANIPULATIVE PATIENT (no float nurse), set firm consistent limits, explain consequences of breaking rules, work consistently with staff to prevent splitting, and give positive feedback for appropriate behavior.

How do you know when the treatment for an antisocial person has been effective?

They show remorse for taking advantage of others.

What is the difference between borderline personality disorder and antisocial behavior?

Borderline PD patients hurt/abuse themselves.

What are the S&S of alcohol withdrawal and when do they usually begin?

They usually begin 3 to 36 hours after the last drink. The patient may present with tremulousness, motor hyperactivity, anxiety, and hyperalertness, irritability, agitation, diaphoresis, tachycardia, hypertension, pounding HA, abdominal cramping, anorexia

What are the S&S of delerium tremens?

Hyperthermia (105), disorientation, delusions, paranoia, and frightening visual and tactile hallucinations. (tachycardia, tachypnea, hypertension, dilated pupils, and diaphoresis).

What are alcohol dependent patients with a thiamine deficiency at risk for?

WERNICKE-KORSAKOFF SYNDROME (WERNIKE'S ENCEPHALOPATHY AND KORSAKOFF PSYCHOSIS) AN IRREVERSIBLE DEGENERATIVE BRAIN DISORDER.

What should you give to a patient experiencing alcohol withdrawal to prevent seizures?

Magnesium sulfate IV

After a patient is discharged following an alcohol withdrawal episode and wants to stop drinking what is a good drug for him/her?

Antabuse- must be committed to not drinking or get severely sick. After the drug is d/c it is in the body for another 2 weeks.

What treatment should you implement for delirium tremens?

Administer D5 1/2 NS at 100cc/hr, now adding thiamine 100 mg, folate 1 mg, and multivitamins to the IV fluids once daily. (the B vitamins help with red blood cell formation and thiamine helps prevent wernicke-korsakoff syndrome). Place them in a quiet/pri

What are the signs and symptoms of a cocaine/crack user?

DILATED PUPILS, TACHYCARDIA, ELEVATED BP (HEART BLOWS UP!), nose bleeds, nasal congestion, sniffing, impaired movement, seizures, insomnia, and anorexia. (OVERDOSE= TACHYCARDIA AND INCREASED BP).

What are the signs of opiate overdose?

RESPIRATORY DEPRESSION --> respiratory arrest, circulatory depression, and unconsciousness-->death

What are the S&S of amphetamine overdose?

RAPID RESPIRATIONS, restlessness, tremor, confusion, hallucinations, and panic.

What is the difference between onset and duration of delirium and dementia?

Delirium is acute onset and duration is days to weeks. Dementia is an insidious onset and the duration is months to years.

1. Walter, teenage patient is admitted to the hospital because of acetaminophen (Tylenol) overdose. Overdoses of acetaminophen can precipitate life-threatening abnormalities in which of the following organs?
a. Lungs
b. Liver
c. Kidney
d. Adrenal Glands

1.Answer B. Acetaminophen is extensively metabolized by pathways in the liver. Toxic doses of acetaminophen deplete hepatic glutathione, resulting in accumulation of the intermediate agent, quinine, which leads to hepatic necrosis. Prolonged use of acetam

2. A contraindication for topical corticosteroid usage in a male patient with atopic dermatitis (eczema) is:
a. Parasite infection.
b. Viral infection.
c. Bacterial infection.
d. Spirochete infection.

2.Answer B. Topical agents produce a localized, rather than systemic effect. When treating atopic dermatitis with a steroidal preparation, the site is vulnerable to invasion by organisms. Viruses, such as herpes simplex or varicella-zoster, present a risk

3. In infants and children, the side effects of first generation over-the-counter (OTC) antihistamines, such as diphenhydramine (Benadryl) and hydroxyzine (Atarax) include:
a. Reye's syndrome.
b. Cholinergic effects.
c. Paradoxical CNS stimulation.
d. Nau

3.Answer C. Typically, first generation OTC antihistamines have a sedating effect because of passage into the CNS. However, in some individuals, especially infants and children, paradoxical CNS stimulation occurs and is manifested by excitement, euphoria,

4. Reye's syndrome, a potentially fatal illness associated with liver failure and encephalopathy is associated with the administration of which over-the-counter (OTC) medication?
a. acetaminophen (Tylenol)
b. ibuprofen (Motrin)
c. aspirin
d. bromphenirami

4.Answer C. Virus-infected children who are given aspirin to manage pain, fever, and inflammation are at an increased risk of developing Reye's syndrome. Use of acetaminophen has not been associated with Reye's syndrome and can be safely given to patients

5. The nurse is aware that the patients who are allergic to intravenous contrast media are usually also allergic to which of the following products?
a. Eggs
b. Shellfish
c. Soy
d. acidic fruits

5.Answer B. Some types of contrast media contain iodine as an ingredient. Shellfish also contain significant amounts of iodine. Therefore, a patient who is allergic to iodine will exhibit an allergic response to both iodine containing contrast media and s

6. A 13-month-old child recently arrived in the United States from a foreign country with his parents and needs childhood immunizations. His mother reports that he is allergic to eggs. Upon further questioning, you determine that the allergy to eggs is an

6.Answer D. The measles portion of the MMR vaccine is grown in chick embryo cells. The current MMR vaccine does not contain a significant amount of egg proteins, and even children with dramatic egg allergies are extremely unlikely to have an anaphylactic

7. The cell and Coombs classification system categorizes allergic reactions and is useful in describing and classifying patient reactions to drugs. Type I reactions are immediate hypersensitivity reactions and are mediated by:
a. immunoglobulin E (IgE).
b

7.Answer A. IgE, the least common serum immunoglobulin (Ig) binds very tightly to receptors on basophils and mast cells and is involved in allergic reactions. Binding of the allergen to the IgE on the cells results in the release of various pharmacologica

8. Drugs can cause adverse events in a patient. Bone marrow toxicity is one of the most frequent types of drug-induced toxicity. The most serious form of bone marrow toxicity is:
a. aplastic anemia.
b. thrombocytosis.
c. leukocytosis.
d. granulocytosis.

8.Answer A. Aplastic anemia is the result of a hypersensitivity reaction and is often irreversible. It leads to pancytopenia, a severe decrease in all cell types: red blood cells, white blood cells, and platelets. A reduced number of red blood cells cause

9. Serious adverse effects of oral contraceptives include:
a. Increase in skin oil followed by acne.
b. Headache and dizziness.
c. Early or mid-cycle bleeding.
d. Thromboembolic complications.

9.Answer D. Oral contraceptives have been associated with an increased risk of stroke, myocardial infarction, and deep vein thrombosis. These risks are increased in women who smoke. Increased skin oil and acne are effects of progestin excess. Headache and

10. The most serious adverse effect of Alprostadil (Prostin VR pediatric injection) administration in neonates is:
a. Apnea.
b. Bleeding tendencies.
c. Hypotension.
d. Pyrexia.

10.Answer A. All items are adverse reactions of the drug. However, apnea appearing during the first hour of drug infusion occurs in 10-12 percent of neonates with congenital heart defects. Clinicians deciding to utilize alprostadil must be prepared to int

11. Mandy, a patient calls the clinic today because he is taking atrovastatin (Lipitor) to treat his high cholesterol and is having pain in both of his legs. You instruct him to:
a. Stop taking the drug and make an appointment to be seen next week.
b. Con

11.Answer C. Muscle aches, soreness, and weakness may be early signs of myopathy such as rhabdomyolysis associated with the HMG-CoA reducatase class of antilipemic agents. This patient will need an immediate evaluation to rule out myopathy. Additional dos

12. Which of the following adverse effects is associated with levothyroxine (Synthroid) therapy?
a. Tachycardia
b. Bradycardia
c. Hypotension
d. Constipation

12.Answer A. Levothyroxine, especially in higher doses, can induce hyperthyroid-like symptoms including tachycardia. An agent that increases the basal metabolic rate would not be expected to induce a slow heart rate. Hypotension would be a side effect of

13. Which of the following adverse effects is specific to the biguanide diabetic drug metformin (Glucophage) therapy?
a. Hypoglycemia
b. GI distress
c. Lactic acidosis
d. Somulence

13.Answer C. Lactic acidosis is the most dangerous adverse effect of metformin administration with death resulting in approximately 50 percent of individuals who develop lactic acidosis while on this drug. Metformin does not induce insulin production; thu

14. The most serious adverse effect of tricyclic antidepressant (TCA) overdose is:
a. Seizures.
b. Hyperpyrexia.
c. Metabolic acidosis.
d. Cardiac arrhythmias.

14.Answer D. Excessive ingestion of TCAs result in life-threatening wide QRS complex tachycardia. TCA overdose can induce seizures, but they are typically not life-threatening. TCAs do not cause an elevation in body temperature. TCAs do not cause metaboli

15. The nurse is aware that the following solutions is routinely used to flush an IV device before and after the administration of blood to a patient is:
a. 0.9 percent sodium chloride
b. 5 percent dextrose in water solution
c. Sterile water
d. Heparin so

15.Answer A. 0.9 percent sodium chloride is normal saline. This solution has the same osmolarity as blood. Its use prevents red cell lysis. The solutions given in options 2 and 3 are hypotonic solutions and can cause red cell lysis. The solution in option

16. Cris asks the nurse whether all donor blood products are cross-matched with the recipient to prevent a transfusion reaction. Which of the following always require cross-matching?
a. packed red blood cells
b. platelets
c. plasma
d. granulocytes

16.Answer A. Red blood cells contain antigens and antibodies that must be matched between donor and recipient. The blood products in options 2-4 do not contain red cells. Thus, they require no cross-match.

17. A month after receiving a blood transfusion an immunocompromised male patient develops fever, liver abnormalities, a rash, and diarrhea. The nurse would suspect this patient has:
a. Nothing related to the blood transfusion.
b. Graft-versus-host diseas

17.Answer B. GVHD occurs when white blood cells in donor blood attack the tissues of an immunocompromised recipient. This process can occur within a month of the transfusion. Options 1 and 4 may be a thought, but the nurse must remember that immunocomprom

18. Jonas comes into the local blood donation center. He says he is here to donate platelets only today. The nurse knows this process is called:
a. Directed donation.
b. Autologous donation.
c. Allogenic donation.
d. Apheresis.

18.Answer D. The process of apheresis involves removal of whole blood from a donor. Within an instrument that is essentially designed as a centrifuge, the components of whole blood are separated. One of the separated portions is then withdrawn, and the re

19. Nurse Bryan knows that the age group that uses the most units of blood and blood products is:
a. Premature infants.
b. Children ages 1-20 years.
c. Adults ages 21-64 years.
d. The elderly above age 65 years.

19.Answer D. People older than 65 years use 43 percent of donated blood. This number is expected to increase as the population ages.

20. A child is admitted with a serious infection. After two days of antibiotics, he is severely neutropenic. The physician orders granulocyte transfusions for the next four days. The mother asks the nurse why? The nurse responds:
a. "This is the only trea

20.Answer D. Granulocyte (neutrophil) replacement therapy is given until the patient's blood values are normal and he is able to fight the infection himself. Options 1 and 3 are not therapeutic responses. The treatment in option 2 takes days and is not al

21. A neighbor tells nurse Maureen he has to have surgery and is reluctant to have any blood product transfusions because of a fear of contracting an infection. He asks the nurse what are his options. The nurse teaches the person that the safest blood pro

21.Answer C. This process is the collection and reinfusion of the patient's own blood. It is recommended by the American Medical Association's Council on Scientific Affairs as the safest product since it eliminates recipient incompatibility and infection.

22. A severely immunocompromised female patient requires a blood transfusion. To prevent GVHD, the physician will order:
a. Diphenhydramine hydrochloride (Benadryl).
b. The transfusion to be administered slowly over several hours.
c. Irradiation of the do

22.Answer C. This process eliminates white blood cell functioning, thus, preventing GVHD. Diphenhydramine HCl is an antihistamine. It's use prior to a blood transfusion decreases the likelihood of a transfusion reaction. Option 2 will not prevent GVHD. Us

23. Louie who is to receive a blood transfusion asks the nurse what is the most common type of infection he could receive from the transfusion. The nurse teaches him that approximately 1 in 250,000 patients contract:
a. Human immunodeficiency disease (HIV

23.Answer C. Hepatitis B is the most common infection spread via blood transfusion. Donors are screened by a questionnaire that includes symptoms. The donated blood is also tested for infection. The risk of infection with the agents in options 2 and 3 has

24. A male patient with blood type AB, Rh factor positive needs a blood transfusion. The Transfusion Service (blood bank) sends type O, Rh factor negative blood to the unit for the nurse to infuse into this patient. The nurse knows that:
a. This donor blo

24.Answer C. Type O, Rh negative blood has none of the major antigens and is safely administered to patients of all blood types. It is also known as the universal donor. Premedicating with these agents will not prevent a major transfusion reaction if the

25. Dr. Rodriguez orders 250 milliliters of packed red blood cells (RBC) for a patient. This therapy is administered for treatment of:
a. Thrombocytopenia.
b. Anemia.
c. Leukopenia.
d. Hypoalbuminemia.

25.Answer B. A red blood cell transfusion is used to correct anemia in patients in which the low red blood cell count must be rapidly corrected. RBC transfusion will not correct a low platelet count. RBC transfusion will not correct a low white blood cell

26. A female patient needs a whole blood transfusion. In order for transfusion services (the blood bank) to prepare the correct product a sample of the patient's blood must be obtained for:
a. A complete blood count and differential.
b. A blood type and c

26.Answer B. This is needed to utilize the correct type of donor blood and to match the donor product with the patient. Incompatible matches would result in severe adverse events and possible death. The tests in options 1 and 3 are unnecessary. The test i

27. A male patient needs to receive a unit of whole blood. What type of intravenous (IV) device should the nurse consider starting?
a. A small catheter to decrease patient discomfort
b. The type of IV device the patient has had in the past, which worked w

27.Answer C. Large bore catheters prevent damage to blood components and are less likely to develop clotting problems than a small bore catheter. The nurse should determine the correct device without asking the patient what type has been used before or as

28. Dr. Smith orders a gram of human salt poor albumin product for a patient. The product is available in a 50 milliliter vial with a concentration of 25 percent. What dosage will the nurse administer?
a. The nurse should use the entire 50 milliliter vial

28.Answer D. A 25 percent solution contains one quarter of a gram per milliliter. Thus, the nurse will administer 4 milliliters to provide a complete gram of albumin. The volume in option 1 would provide 12.5 grams of albumin. The nurse should determine t

29. Central venous access devices (CVADs) are frequently utilized to administer chemotherapy. What is a distinct advantage of using the CVAD for chemotherapeutic agent administration?
a. CVADs are less expensive than a peripheral IV.
b. Once a week admini

29.Answer C. Many chemotherapeutic drugs are vesicants (highly active corrosive materials that can produce tissue damage even in low concentrations). Extravasations of a vesicant can result in significant tissue necrosis. Administration into a large vein

30. A female patient's central venous access device (CVAD) becomes infected. Why would the physician order antibiotics to be given through the line rather than through a peripheral IV line?
a. To prevent infiltration of the peripheral line
b. To reduce th

30.Answer D. Microorganisms that infect CVADs are often coagulase-negative staphylococci, which can be eliminated by antibiotic administration through the catheter. If unsuccessful in eliminating the microorganism, the CVAD must be removed. CVAD use lesse

1. An infection in a central venous access device is not eliminated by giving antibiotics through the catheter. How would bacterial glycocalyx contribute to this?
a. It protects the bacteria from antibiotic and immunologic destruction.
b. Glycocalyx neutr

1.Answer C. Glycocalyx is a viscous polysaccharide or polypeptide slime that covers microbes. It enhances adherence to surfaces, resists phagocytic engulfment by the white blood cells, and prevents antibiotics from contacting the microbe. Glycocalyx does

2. Central venous access devices are beneficial in pediatric therapy because:
a. They don't frighten children.
b. Use of the arms is not restricted.
c. They cannot be dislodged.
d. They are difficult to see.

2.Answer B. The child can move his extremities and function in a normal fashion. This lessens stress associated with position restriction and promotes normal activity. Fear may not be eliminated. All lines can be dislodged. Even small catheters can be rea

3. How can central venous access devices (CVADs) be of value in a patient receiving chemotherapy who has stomatitis and severe diarrhea?
a. The chemotherapy can be rapidly completed allowing the stomatitis and diarrhea to resolve.
b. Crystalloid can be ad

3.Answer C. In patients unable to take oral nutrition, parenteral hyperalimentation is an option for providing nutritional support. High concentrations of dextrose, protein, minerals, vitamins, and trace elements can be provided. Dosing is not affected wi

4. Some central venous access devices (CVAD) have more than one lumen. These multi lumen catheters:
a. Have an increased risk of infiltration.
b. Only work a short while because the small bore clots off.
c. Are beneficial to patient care but are prohibiti

4.Answer D. A multilumen catheter contains separate ports and means to administer agents. An agent infusing in one port cannot mix with an agent infusing into another port. Thus, agents that would be incompatible if given together can be given in separate

5. Some institutions will not infuse a fat emulsion, such as Intralipid, into central venous access devices (CVAD) because:
a. Lipid residue may accumulate in the CVAD and occlude the catheter.
b. If the catheter clogs, there is no treatment other than re

5.Answer A. Occlusion occurs with slow infusion rates and concurrent administration of some medications. Lipid occlusions may be treated with 70 percent ethanol or with 0.1 mmol/mL NaOH. Lipids provide essential fatty acids. It is recommended that approxi

6. A male patient needs a percutaneously inserted central catheter (PICC) for prolonged IV therapy. He knows it can be inserted without going to the operating room. He mentions that, "at least the doctor won't be wearing surgical garb, will he?" How will

6.Answer C. Strict aseptic technique including the use of cap, mask, and sterile gown and gloves is require when placing a central venous line including a PICC. Options A, B, and D are incorrect statements. They increase the risk of infection.

7. A male patient is to receive a percutaneously inserted central catheter (PICC). He asks the nurse whether the insertion will hurt. How will the nurse reply?
a. "You will have general anesthesia so you won't feel anything."
b. "It will be inserted rapid

7.Answer C. Pain related to PICC insertion occurs with puncture of the skin. When inserting PICC lines, the insertion site is anesthetized so no pain is felt. The patient will not receive general anesthesia or sedation. Statement 2 is false. Unnecessary p

8. What volume of air can safely be infused into a patient with a central venous access device (CVAD)?
a. It is dependent on the patient's weight and height.
b. Air entering the patient through a CVAD will follow circulation to the lungs where it will be

8.Answer B. Any air entering the right heart can lead to a pulmonary embolus. All air should be purged from central venous lines none should enter the patient.

9. Kent a new staff nurse asks her preceptor nurse how to obtain a blood sample from a patient with a portacath device. The preceptor nurse teaches the new staff nurse:
a. The sample will be withdrawn into a syringe attached to the portacath needle and th

9.Answer A. A special portacath needle is used to access the portacath device. A syringe is attached and the sample is obtained. One of the primary reasons for insertion of a portacath device is the need for frequent or long-term blood sampling. A vacutai

10. What is the purpose of "tunneling" (inserting the catheter 2-4 inches under the skin) when the surgeon inserts a Hickman central catheter device? Tunneling:
a. Increases the patient's comfort level.
b. Decreases the risk of infection.
c. Prevents the

10.Answer B. The actual access to the subclavian vein is still just under the clavicle, but by tunneling the distal portion of the catheter several inches under the skin the risk of migratory infection is reduces compared to a catheter that enters the sub

11. The primary complication of a central venous access device (CVAD) is:
a. Thrombus formation in the vein.
b. Pain and discomfort.
c. Infection.
d. Occlusion of the catheter as the result of an intra-lumen clot.

11.Answer C. A foreign body in a blood vessel increases the risk of infection. Catheters that come outside the body have an even higher risk of infection. Most infections are caused by skin bacteria. Other infective organisms include yeasts and fungi. Opt

12. Nurse Blessy is doing some patient education related to a patient's central venous access device. Which of the following statements will the nurse make to the patient?
a. "These type of devices are essentially risk free."
b. "These devices seldom work

12.Answer D. A solution containing heparin is used to reduce catheter clotting and maintain patency. The concentration of heparin used depends on the patient's age, comorbidities, and the frequency of catheter access/flushing. Although patients have few c

13. The chemotherapeutic DNA alkylating agents such as nitrogen mustards are effective because they:
a. Cross-link DNA strands with covalent bonds between alkyl groups on the drug and guanine bases on DNA.
b. Have few, if any, side effects.
c. Are used to

13.Answer A. Alkylating agents are highly reactive chemicals that introduce alkyl radicals into biologically active molecules and thereby prevent their proper functioning, replication, and transcription. Alkylating agents have numerous side effects includ

14. Hormonal agents are used to treat some cancers. An example would be:
a. Thyroxine to treat thyroid cancer.
b. ACTH to treat adrenal carcinoma.
c. Estrogen antagonists to treat breast cancer.
d. Glucagon to treat pancreatic carcinoma.

14.Answer C. Estrogen antagonists are used to treat estrogen hormone-dependent cancer, such as breast carcinoma. A well-known estrogen antagonist used in breast cancer therapy is tamoxifen (Nolvadex). This drug, in combination with surgery and other chemo

15. Chemotherapeutic agents often produce a certain degree of myelosuppression including leukopenia. Leukopenia does not present immediately but is delayed several days to weeks because:
a. The patient's hemoglobin and hematocrit are normal.
b. Red blood

15.Answer D. The time required to clear circulating cells before the effect that chemotherapeutic drugs have on precursor cell maturation in the bone marrow becomes evident. Leukopenia is an abnormally low white blood cell count. Answers A-C pertain to re

16. Currently, there is no way to prevent myelosuppression. However, there are medications available to elicit a more rapid bone marrow recovery. An example is:
a. Epoetin alfa (Epogen, Procrit).
b. Glucagon.
c. Fenofibrate (Tricor).
d. Lamotrigine (Lamic

16.Answer A. Epoetin alfa (Epogen, Procrit) is a recombinant form of endogenous erythropoietin, a hematopoietic growth factor normally produced by the kidney that is used to induce red blood cell production in the bone marrow and reduce the need for blood

17. Estrogen antagonists are used to treat estrogen hormone-dependent cancer, such as breast carcinoma. Androgen antagonists block testosterone stimulation of androgen-dependent cancers. An example of an androgen-dependent cancer would be:
a. Prostate can

17.Answer A. Prostate tissue is stimulated by androgens and suppressed by estrogens. Androgen antagonists will block testosterone stimulation of prostate carcinoma cells. The types of cancer in options 2-4 are not androgen dependent.

18. Serotonin release stimulates vomiting following chemotherapy. Therefore, serotonin antagonists are effective in preventing and treating nausea and vomiting related to chemotherapy. An example of an effective serotonin antagonist antiemetic is:
a. onda

18.Answer A. Chemotherapy often induces vomiting centrally by stimulating the chemoreceptor trigger zone (CTZ) and peripherally by stimulating visceral afferent nerves in the GI tract. Ondansetron (Zofran) is a serotonin antagonist that bocks the effects

19. Methotrexate, the most widely used antimetabolite in cancer chemotherapy does not penetrate the central nervous system (CNS). To treat CNS disease this drug must be administered:
a. Intravenously.
b. Subcutaneously.
c. Intrathecally.
d. By inhalation.

19.Answer C. With intrathecal administration chemotherapy is injected through the theca of the spinal cord and into the subarachnoid space entering into the cerebrospinal fluid surrounding the brain and spinal cord. The methods in options A, B, and D are

20. Methotrexate is a folate antagonist. It inhibits enzymes required for DNA base synthesis. To prevent harm to normal cells, a fully activated form of folic acid known as leucovorin (folinic acid citrovorum factor) can be administered. Administration of

20.Answer B. Leucovorin is used to save or "rescue" normal cells from the damaging effects of chemotherapy allowing them to survive while the cancer cells die. Therapy to rapidly reduce the number of cancerous cells is the induction phase. Consolidation t

21. A male Patient is undergoing chemotherapy may also be given the drug allopurinol (Zyloprim, Aloprim). Allopurinol inhibits the synthesis of uric acid. Concomitant administration of allopurinol prevents:
a. Myelosuppression.
b. Gout and hyperuricemia.

21.Answer B. Prevent uric acid nephropathy, uric acid lithiasis, and gout during cancer therapy since chemotherapy causes the rapid destruction of cancer cells leading to excessive purine catabolism and uric acid formation. Allopurinol can induce myelosup

22. Superficial bladder cancer can be treated by direct instillation of the antineoplastic antibiotic agent mitomycin (Mutamycin). This process is termed:
a. Intraventricular administration.
b. Intravesical administration.
c. Intravascular administration.

22.Answer B. Medications administered intravesically are instilled into the bladder. Intraventricular administration involves the ventricles of the brain. Intravascular administration involves blood vessels. Intrathecal administration involves the fluid s

23. The most common dose-limiting toxicity of chemotherapy is:
a. Nausea and vomiting.
b. Bloody stools.
c. Myelosuppression.
d. Inability to ingest food orally due to stomatitis and mucositis.

23.Answer C. The overall goal of cancer chemotherapy is to give a dose large enough to be lethal to the cancer cells, but small enough to be tolerable for normal cells. Unfortunately, some normal cells are affected including the bone marrow. Myelosuppress

24. Chemotherapy induces vomiting by:
a. Stimulating neuroreceptors in the medulla.
b. Inhibiting the release of catecholamines.
c. Autonomic instability.
d. Irritating the gastric mucosa.

24.Answer A. Vomiting (emesis) is initiated by a nucleus of cells located in the medulla called the vomiting center. This center coordinates a complex series of events involving pharyngeal, gastrointestinal, and abdominal wall contractions that lead to ex

25. Myeloablation using chemotherapeutic agents is useful in cancer treatment because:
a. It destroys the myelocytes (muscle cells).
b. It reduces the size of the cancer tumor.
c. After surgery, it reduces the amount of chemotherapy needed.
d. It destroys

25.Answer A. Myelo comes from the Greek word myelos, which means marrow. Ablation comes from the Latin word ablatio, which means removal. Thus, myeloablative chemotherapeurtic agents destroy the bone marrow. This procedure destroys normal bone marrow as w

26. Anticipatory nausea and vomiting associated with chemotherapy occurs:
a. Within the first 24 hours after chemotherapy.
b. 1-5 days after chemotherapy.
c. Before chemotherapy administration.
d. While chemotherapy is being administered.

26.Answer C. Nausea and vomiting (N&V) are common side effects of chemotherapy. Some patients are able to trigger these events prior to actually receiving chemotherapy by anticipating, or expecting, to have these effects. N&V occurring post-chemotherapeut

27. Medications bound to protein have the following effect:
a. Enhancement of drug availability.
b. Rapid distribution of the drug to receptor sites.
c. The more drug bound to protein, the less available for desired effect.
d. Increased metabolism of the

27.Answer C. Only an unbound drug can be distributed to active receptor sites. Therefore, the more of a drug that is bound to protein, the less it is available for the desired drug effect. Less drug is available if bound to protein. Distribution to recept

28. Some drugs are excreted into bile and delivered to the intestines. Prior to elimination from the body, the drug may be absorbed. This process is known as:
a. Hepatic clearance.
b. Total clearance.
c. Enterohepatic cycling.
d. First-pass effect.

28.Answer C. Drugs and drug metabolites with molecular weights higher than 300 may be excreted via the bile, stored in the gallbladder, delivered to the intestines by the bile duct, and then reabsorbed into the circulation. This process reduces the elimin

29. An adult patient has been taking a drug (Drug A) that is highly metabolized by the cytochrome p-450 system. He has been on this medication for 6 months. At this time, he is started on a second medication (Drug B) that is an inducer of the cytochrome p

29.Answer C. Drug B will induce the cytochrome p-450 enzyme system of the liver thus, increasing the metabolism of Drug A. Therefore, Drug A will be broken down faster and exert decreased therapeutic effects. Drug A will be metabolized faster, thus reduci

30. Epinephrine is administered to a female patient. The nurse should expect this agent to rapidly affect:
a. Adrenergic receptors.
b. Muscarinic receptors.
c. Cholinergic receptors.
d. Nicotinic receptors.

30.Answer A. Epinephrine (adrenaline) rapidly affects both alpha and beta adrenergic receptors eliciting a sympathetic (fight or flight) response. Muscarinic receptors are cholinergic receptors and are primarily located at parasympathetic junctions. Choli

1. The nursery nurse is putting erythromycin ointment in the newborn's eyes to prevent infection. She places it in the following area of the eye:
a. under the eyelid
b. on the cornea.
c. in the lower conjunctival sac
d. by the optic disc.

1. C. The ointment is placed in the lower conjunctival sac so it will not scratch the eye itself and will get well distributed.

2. The physician orders penicillin for a patient with streptococcal pharyngitis. The nurse administers the drug as ordered, and the patient has an allergic reaction. The nurse checks the medication order sheet and finds that the patient is allergic to pen

2. D. The physician, nurse, and pharmacist all are licensed professionals and share responsibility for errors.

3. James Perez, a nurse on a geriatric floor, is administering a dose of digoxin to one of his patients. The woman asks why she takes a different pill than her niece, who also has heart trouble. James replies that as people get older, liver and kidney fun

3. B. The decreased circulation to the kidney and reduced liver function tend to allow drugs to accumulate and have toxic effects.

4. The nurse is administering Augmentin to her patient with a sinus infection. Which is the best way for her to insure that she is giving it to the right patient?
a. Call the patient by name
b. Read the name of the patient on the patient's door
c. Check t

4. C. The correct way to identify a patient before giving a medication is to check the name on the medication administration record with the patient's identification band. The nurse should also ask the patient to state their name. The name on the door or

5. The most important instructions a nurse can give a patient regarding the use of the antibiotic Ampicillin prescribed for her are to
a. call the physician if she has any breathing difficulties.
b. take it with meals so it doesn't cause an upset stomach.

5. C. Frequently patients do not complete an entire course of antibiotic therapy, and the bacteria are not destroyed.

6. Mr. Jessie Ray, a newly admitted patient, has a seizure disorder which is being treated with medication. Which of the following drugs would the nurse question if ordered for him?
a. Phenobarbitol, 150 mg hs
b. Amitriptylene (Elavil), 10 mg QID.
c. Valp

6. B. Elavil is an antidepressant that lowers the seizure threshold, so would not be appropriate for this patient. The other medications are anti-seizure drugs.

7. Mrs. Jane Gately has been dealing with uterine cancer for several months. Pain management is the primary focus of her current admission to your oncology unit. Her vital signs on admission are BP 110/64, pulse 78, respirations 18, and temperature 99.2 F

7. C. Morphine sulfate depresses the respiratory center. When the rate is less than 10, the MD should be notified.

8. When counseling a patient who is starting to take MAO (monoamine oxidase) inhibitors such as Nardil for depression, it is essential that they be warned not to eat foods containing tyramine, such as:
a. Roquefort, cheddar, or Camembert cheese.
b. grape

8. A. Monoamine oxidase inhibitors react with foods high in the amino acid tyramine to cause dangerously high blood pressure. Aged cheeses are all high in this amino acid the other foods are not.

9. The physician orders an intramuscular injection of Demerol for the postoperativepatient's pain. When preparing to draw up the medication, the nurse is careful to remove the correct vial from the narcotics cabinet. It is labeled
a. simethicone.
b. albut

9. C. The generic name for Demerol is meperidine.

10. The nurse is administering an antibiotic to her pediatric patient. She checks the patient's armband and verifies the correct medication by checking the physician's order, medication kardex, and vial. Which of the following is not considered one of the

10. C. The five rights of medication administration are right drug, right dose, right route, right time, right patient. Frequency is not included.

11. A nurse is preparing the client's morning NPH insulin dose and notices a clumpy precipitate inside the insulin vial. The nurse should:
a. draw up and administer the dose
b. shake the vial in an attempt to disperse the clumps
c. draw the dose from a ne

11. C. The nurse should always inspect the vial of insulin before use for solution changes that may signify loss of potency. NPH insulin is normally uniformly cloudy. Clumping, frosting, and precipitates are signs of insulin damage. In this situation, bec

12. A client with histoplasmosis has an order for ketoconazole (Nizoral). The nurse teaches the client to do which of the following while taking this medication?
a. take the medication on an empty stomach
b. take the medication with an antacid
c. avoid ex

12. C. The client should be taught that ketoconazole is an antifungal medication. It should be taken with food or milk. Antacids should be avoided for 2 hours after it is taken because gastric acid is needed to activate the medication. The client should a

13. A nurse has taught a client taking a xanthine bronchodilator about beverages to avoid. The nurse determines that the client understands the information if the client chooses which of the following beverages from the dietary menu?
a. chocolate milk
b.

13. B. Cola, coffee, and chocolate contain xanthine and should be avoided by the client taking a xanthine bronchodilator. This could lead to an increased incidence of cardiovascular and central nervous system side effects that can occur with the use of th

14. A client is taking famotidine (Pepcid) asks the home care nurse what would be the best medication to take for a headache. The nurse tells the client that it would be best to take:
a. aspirin (acetylsalicylic acid, ASA)
b. ibuprofen (Motrin)
c. acetami

14. C. The client is taking famotidine, a histamine receptor antagonist. This implies that the client has a disorder characterized by gastrointestinal (GI) irritation. The only medication of the ones listed in the options that is not irritating to the GI

15. A nurse is planning dietary counseling for the client taking triamterene (Dyrenium). The nurse plans to include which of the following in a list of foods that are acceptable?
a. baked potato
b. bananas
c. oranges
d. pears canned in water

15. D. Triamterene is a potassium-sparing diuretic, and clients taking this medication should be cautioned against eating foods that are high in potassium, including many vegetables, fruits, and fresh meats. Because potassium is very water-soluble, foods

16. A client with advanced cirrhosis of the liver is not tolerating protein well, as eveidenced by abnormal laboratory values. The nurse anticipates that which of the following medications will be prescribed for the client?
a. lactulose (Chronulac)
b. eth

16. A. The client with cirrhosis has impaired ability to metabolize protein because of liver dysfunction. Administration of lactulose aids in the clearance of ammonia via the gastrointestinal (GI) tract. Ethacrynic acid is a diuretic. Folic acid and thiam

17. A female client tells the clinic nurse that her skin is very dry and irritated. Which product would the nurse suggest that the client apply to the dry skin?
a. glycerin emollient
b. aspercreame
c. myoflex
d. acetic acid solution

17. A. Glycerin is an emollient that is used for dry, cracked, and irritated skin. Aspercreame and Myoflex are used to treat muscular aches. Acetic acid solution is used for irrigating, cleansing, and packing wounds infected by Pseudomonas aeruginosa.

18. A nurse is providing instructions to a client regarding quinapril hydrochloride (Accupril). The nurse tells the client:
a. to take the medication with food only
b. to rise slowly from a lying to a sitting position
c. to discontinue the medication if n

18. B. Accupril is an angiotensin-converting enzyme (ACE) inhibitor. It is used in the treatment of hypertension. The client should be instructed to rise slowly from a lying to sitting position and to permit the legs to dangle from the bed momentarily bef

19. Auranofin (Ridaura) is prescribed for a client with rheumatoid arthritis, and the nurse monitors the client for signs of an adverse effect related to the medication. Which of the following indicates an adverse effect?
a. nausea
b. diarrhea
c. anorexia

19. D. Auranofin (Ridaura) is a gold preparation that is used as an antirheumatic. Gold toxicity is an adverse effect and is evidenced by decreased hemoglobin, leukopenia, reduced granulocyte counts, proteinuria, hematuria, stomatitis, glomerulonephritis,

20. A client has been taking benzonatate (Tessalon) as ordered. The nurse tells the client that this medication should do which of the following?
a. take away nausea and vomiting
b. calm the persistent cough
c. decrease anxiety level
d. increase comfort l

20. B. Benzonatate is a locally acting antitussive. Its effectiveness is measured by the degree to which it decreases the intensity and frequency of cough, without eliminating the cough reflex

1. The physician orders penicillin for a patient with streptococcal pharyngitis. The nurse administers the drug as ordered, and the patient has an allergic reaction. The nurse checks the medication order sheet and finds that the patient is allergic to pen

The correct answer is (4).
The physician, nurse, and pharmacist all are licensed professionals and share responsibility for errors.

2. The nurse is administering Augmentin to her patient with a sinus infection. Which is the best way for her to insure that she is giving it to the right patient?
1) Call the patient by name
2) Read the name of the patient on the patient's door
3) Check t

The correct answer is (3).
The correct way to identify a patient before giving a medication is to check the name on the medication administration record with the patient's identification band. The nurse should also ask the patient to state their name. The

3. Tom R., a newly admitted patient, has a seizure disorder which is being treated with medication. Which of the following drugs would the nurse question if ordered for him?
1) Phenobarbitol, 150 mg hs
2) Amitriptylene (Elavil), 10 mg QID.
3) Valproic aci

The correct answer is (2).
Elavil is an antidepressant that lowers the seizure threshold, so would not be appropriate for this patient. The other medications are anti-seizure drugs.

4. When counseling a patient who is starting to take MAO (monoamineoxidase) inhibitors such as Nardil for depression, it is essential that they be warned not to eat foods containing tyramine, such as
1) Roquefort, cheddar, or Camembert cheese.
2) grape ju

The correct answer is (1).
Monoamine oxidase inhibitors react with foods high in the amino acid tyramine to cause dangerously high blood pressure. Aged cheeses are all high in this amino acid the other foods are not.

5. The nurse is administering an antibiotic to her pediatric patient. She checks the patient's armband and verifies the correct medication by checking the physician's order, medication kardex, and vial. Which of the following is not considered one of the

The correct answer is (3).
The five rights of medication administration are right drug, right dose, right route, right time, right patient. Frequency is not included.

6. The physician orders an intramuscular injection of Demerol for the post-operative patient's pain. When preparing to draw up the medication, the nurse is careful to remove the correct vial from the narcotics cabinet. It is labeled
1) simethicone.
2) alb

The correct answer is (3).
The generic name for Demerol is meperidine.

7. Mrs. Garvey has been dealing with uterine cancer for several months. Pain management is the primary focus of her current admission to your oncology unit. Her vital signs on admission are BP 110/64, pulse 78, respirations 18,and temperature 99.2 F. Morp

The correct answer is (3).
Morphine sulfate depresses the respiratory center. When the rate is less than 10, the MD should be notified.

8. The most important instructions a nurse can give a patient regarding the use of the antibiotic Ampicillian prescribed for her are to
1) call the physician if she has any breathing difficulties.
2) take it with meals so it doesn't cause an upset stomach

The correct answer is (3).
Frequently patients do not complete an entire course of antibiotic therapy, and the bacteria are not destroyed.

9. Frank Jones, a nurse on a geriatric floor, is administering a dose of digoxin to one of his patients. The woman asks why she takes a different pill than her niece, who also has heart trouble. Frank replies that as people get older, liver and kidney fun

The correct answer is (2).
The decreased circulation to the kidney and reduced liver function tend to allow drugs to accumulate and have toxic effects.

10. The nursery nurse is putting erythromycin ointment in the newborn's eyes to prevent infection. She places it in the following area of the eye:
1) under the eyelid.
2) on the cornea.
3) in the lower conjunctival sac.
4) by the optic disc.

The correct answer is (3).
The ointment is placed in the lower conjunctival sac so it will not scratch the eye itself and will get well distributed.

The nurse's understanding of the clotting mechanism is important in administering anticoagulant drugs. the nurse understands that which of the following clotting factors are formed after injury to the vessels?
1. Fibrin, vitamin K
2. Thromboplastin, fibri

4. Thrombin, fibrin

The patient receiving heparin therapy asks how the "blood thinner" works. The best response by the nurse would be:
1. "heparin makes the blood less thick"
2. "heparin does not thin the blood but prevents clots from forming as easily in the blood vessels

2. "heparin does not thin the blood but prevents clots form forming as easily in the blood vessels

Nursing interventions for a patient receiving enoxaparin (Lovenox) may include: (Select all that apply)
1. teaching the patient or family to give subcutaneous injections at home
2. teaching the patient or family not take any OTC drugs without first consul

1. teaching the patient or family to give subcutaneous injections at home
2. teaching the patient or family not take any OTC drugs without first consulting with the health care provider
3. teaching the patient to observe for unexplained bleeding such as p

The nurse receives the patient's lab values throughout warfarin drug therapy. The expected therapeutic level is:
1. aPTT of three to four times the normal control value
2. aPTT one to two times the patient's baseline level
3. aPT one to two times the pati

4. aPT one and half to two and half times the control values.
Rationale: aPT is the coagulation study that monitors oral anticoagulant use, such as warfarin. As a result of one and half to two and half times the control value indicates adquate anticoagula

A patient is receiving a thrombolytic agent, alteplase (Activase), follwing an acute myocardial infraction. Which condition is most likely attributed to thrombolytic therapy with this agent?
1. Skin rash with urticaria
2. Wheezing with labored respiration

3. Bruising and epistaxis
Rationale: Thrombolytic agents dissolve existing clots rapidly and continue to have effects for 2 to 4 days. All forms of bleeding must be monitored and reported immediately. Skin rash with urticara, wheezing with labored respira

A patient has started clopidogrel (Plavix) after experiencing a TIA (transient ischemic attack). The desired therapeutic effects of this drug will be:
1. anti-inflammatory and antipyretic effects
2. to reduce the risk of a stroke from a blood clot
3. anal

2. to reduce the risk of a stroke from a blood clot
Rationale: Antiplatelet drugs such as clopidogerel are given to inhibit platelet aggreagtion and thus reduce the risk of thrombus formation. Antiplatelet drugs do no exert antiinflammatory, antipyretic o

1) A nurse is caring for a client with hyperparathyroidism and notes that the client's serum calcium level is 13 mg/dL. Which medication should the nurse prepare to administer as prescribed to the client?
1. Calcium chloride
2. Calcium gluconate
3. Calcit

3. Calcitonin (Miacalcin)
Rationale:
The normal serum calcium level is 8.6 to 10.0 mg/dL. This client is experiencing hypercalcemia. Calcium gluconate and calcium chloride are medications used for the treatment of tetany, which occurs as a result of acute

2.) Oral iron supplements are prescribed for a 6-year-old child with iron deficiency anemia. The nurse instructs the mother to administer the iron with which best food item?
1. Milk
2. Water
3. Apple juice
4. Orange juice

4. Orange juice
Rationale:
Vitamin C increases the absorption of iron by the body. The mother should be instructed to administer the medication with a citrus fruit or a juice that is high in vitamin C. Milk may affect absorption of the iron. Water will no

3.) Salicylic acid is prescribed for a client with a diagnosis of psoriasis. The nurse monitors the client, knowing that which of the following would indicate the presence of systemic toxicity from this medication?
1. Tinnitus
2. Diarrhea
3. Constipation

1. Tinnitus
Rationale:
Salicylic acid is absorbed readily through the skin, and systemic toxicity (salicylism) can result. Symptoms include tinnitus, dizziness, hyperpnea, and psychological disturbances. Constipation and diarrhea are not associated with s

4.) The camp nurse asks the children preparing to swim in the lake if they have applied sunscreen. The nurse reminds the children that chemical sunscreens are most effective when applied:
1. Immediately before swimming
2. 15 minutes before exposure to the

4. At least 30 minutes before exposure to the sun
Rationale:
Sunscreens are most effective when applied at least 30 minutes before exposure to the sun so that they can penetrate the skin. All sunscreens should be reapplied after swimming or sweating.

5.) Mafenide acetate (Sulfamylon) is prescribed for the client with a burn injury. When applying the medication, the client complains of local discomfort and burning. Which of the following is the most appropriate nursing action?
1. Notifying the register

3. Informing the client that this is normal
Rationale:
Mafenide acetate is bacteriostatic for gram-negative and gram-positive organisms and is used to treat burns to reduce bacteria present in avascular tissues. The client should be informed that the medi

6.) The burn client is receiving treatments of topical mafenide acetate (Sulfamylon) to the site of injury. The nurse monitors the client, knowing that which of the following indicates that a systemic effect has occurred?
1.Hyperventilation
2.Elevated blo

1.Hyperventilation
Rationale:
Mafenide acetate is a carbonic anhydrase inhibitor and can suppress renal excretion of acid, thereby causing acidosis. Clients receiving this treatment should be monitored for signs of an acid-base imbalance (hyperventilation

7.) Isotretinoin is prescribed for a client with severe acne. Before the administration of this medication, the nurse anticipates that which laboratory test will be prescribed?
1. Platelet count
2. Triglyceride level
3. Complete blood count
4. White blood

2. Triglyceride level
Rationale:
Isotretinoin can elevate triglyceride levels. Blood triglyceride levels should be measured before treatment and periodically thereafter until the effect on the triglycerides has been evaluated. Options 1, 3, and 4 do not n

8.) A client with severe acne is seen in the clinic and the health care provider (HCP) prescribes isotretinoin. The nurse reviews the client's medication record and would contact the (HCP) if the client is taking which medication?
1. Vitamin A
2. Digoxin

1. Vitamin A
Rationale:
Isotretinoin is a metabolite of vitamin A and can produce generalized intensification of isotretinoin toxicity. Because of the potential for increased toxicity, vitamin A supplements should be discontinued before isotretinoin thera

9.) The nurse is applying a topical corticosteroid to a client with eczema. The nurse would monitor for the potential for increased systemic absorption of the medication if the medication were being applied to which of the following body areas?
1. Back
2.

2. Axilla
Rationale:
Topical corticosteroids can be absorbed into the systemic circulation. Absorption is higher from regions where the skin is especially permeable (scalp, axilla, face, eyelids, neck, perineum, genitalia), and lower from regions in which

10.) The clinic nurse is performing an admission assessment on a client. The nurse notes that the client is taking azelaic acid (Azelex). Because of the medication prescription, the nurse would suspect that the client is being treated for:
1. Acne
2. Ecze

1. Acne
Rationale:
Azelaic acid is a topical medication used to treat mild to moderate acne. The acid appears to work by suppressing the growth of Propionibacterium acnes and decreasing the proliferation of keratinocytes. Options 2, 3, and 4 are incorrect

11.) The health care provider has prescribed silver sulfadiazine (Silvadene) for the client with a partial-thickness burn, which has cultured positive for gram-negative bacteria. The nurse is reinforcing information to the client about the medication. Whi

3. "The medication will permanently stain my skin."
Rationale:
Silver sulfadiazine (Silvadene) is an antibacterial that has a broad spectrum of activity against gram-negative bacteria, gram-positive bacteria, and yeast. It is applied directly to the wound

12.) A nurse is caring for a client who is receiving an intravenous (IV) infusion of an antineoplastic medication. During the infusion, the client complains of pain at the insertion site. During an inspection of the site, the nurse notes redness and swell

1. Notify the registered nurse.
Rationale:
When antineoplastic medications (Chemotheraputic Agents) are administered via IV, great care must be taken to prevent the medication from escaping into the tissues surrounding the injection site, because pain, ti

13.) The client with squamous cell carcinoma of the larynx is receiving bleomycin intravenously. The nurse caring for the client anticipates that which diagnostic study will be prescribed?
1. Echocardiography
2. Electrocardiography
3. Cervical radiography

4. Pulmonary function studies
Rationale:
Bleomycin is an antineoplastic medication (Chemotheraputic Agents) that can cause interstitial pneumonitis, which can progress to pulmonary fibrosis. Pulmonary function studies along with hematological, hepatic, an

14.) The client with acute myelocytic leukemia is being treated with busulfan (Myleran). Which laboratory value would the nurse specifically monitor during treatment with this medication?
1. Clotting time
2. Uric acid level
3. Potassium level
4. Blood glu

2. Uric acid level
Rationale:
Busulfan (Myleran) can cause an increase in the uric acid level. Hyperuricemia can produce uric acid nephropathy, renal stones, and acute renal failure. Options 1, 3, and 4 are not specifically related to this medication.

15.) The client with small cell lung cancer is being treated with etoposide (VePesid). The nurse who is assisting in caring for the client during its administration understands that which side effect is specifically associated with this medication?
1. Alo

4. Orthostatic hypotension
Rationale:
A side effect specific to etoposide is orthostatic hypotension. The client's blood pressure is monitored during the infusion. Hair loss occurs with nearly all the antineoplastic medications. Chest pain and pulmonary f

16.) The clinic nurse is reviewing a teaching plan for the client receiving an antineoplastic medication. When implementing the plan, the nurse tells the client:
1. To take aspirin (acetylsalicylic acid) as needed for headache
2. Drink beverages containin

3. Consult with health care providers (HCPs) before receiving immunizations
Rationale:
Because antineoplastic medications lower the resistance of the body, clients must be informed not to receive immunizations without a HCP's approval. Clients also need t

17.) The client with ovarian cancer is being treated with vincristine (Oncovin). The nurse monitors the client, knowing that which of the following indicates a side effect specific to this medication?
1. Diarrhea
2. Hair loss
3. Chest pain
4. Numbness and

4. Numbness and tingling in the fingers and toes
Rationale:
A side effect specific to vincristine is peripheral neuropathy, which occurs in almost every client. Peripheral neuropathy can be manifested as numbness and tingling in the fingers and toes. Depr

18.) The nurse is reviewing the history and physical examination of a client who will be receiving asparaginase (Elspar), an antineoplastic agent. The nurse consults with the registered nurse regarding the administration of the medication if which of the

1. Pancreatitis
Rationale:
Asparaginase (Elspar) is contraindicated if hypersensitivity exists, in pancreatitis, or if the client has a history of pancreatitis. The medication impairs pancreatic function and pancreatic function tests should be performed b

19.) Tamoxifen is prescribed for the client with metastatic breast carcinoma. The nurse understands that the primary action of this medication is to:
1. Increase DNA and RNA synthesis.
2. Promote the biosynthesis of nucleic acids.
3. Increase estrogen con

4. Compete with estradiol for binding to estrogen in tissues containing high concentrations of receptors.
Rationale:
Tamoxifen is an antineoplastic medication that competes with estradiol for binding to estrogen in tissues containing high concentrations o

20.) The client with metastatic breast cancer is receiving tamoxifen. The nurse specifically monitors which laboratory value while the client is taking this medication?
1. Glucose level
2. Calcium level
3. Potassium level
4. Prothrombin time

2. Calcium level
Rationale:
Tamoxifen may increase calcium, cholesterol, and triglyceride levels. Before the initiation of therapy, a complete blood count, platelet count, and serum calcium levels should be assessed. These blood levels, along with cholest

21.) A nurse is assisting with caring for a client with cancer who is receiving cisplatin. Select the adverse effects that the nurse monitors for that are associated with this medication. Select all that apply.
1. Tinnitus
2. Ototoxicity
3. Hyperkalemia
4

1. Tinnitus
2. Ototoxicity
5. Nephrotoxicity
6. Hypomagnesemia
Rationale:
Cisplatin is an alkylating medication. Alkylating medications are cell cycle phase-nonspecific medications that affect the synthesis of DNA by causing the cross-linking of DNA to in

22.) A nurse is caring for a client after thyroidectomy and notes that calcium gluconate is prescribed for the client. The nurse determines that this medication has been prescribed to:
1. Treat thyroid storm.
2. Prevent cardiac irritability.
3. Treat hypo

3. Treat hypocalcemic tetany.
Rationale:
Hypocalcemia can develop after thyroidectomy if the parathyroid glands are accidentally removed or injured during surgery. Manifestations develop 1 to 7 days after surgery. If the client develops numbness and tingl

23.) A client who has been newly diagnosed with diabetes mellitus has been stabilized with daily insulin injections. Which information should the nurse teach when carrying out plans for discharge?
1. Keep insulin vials refrigerated at all times.
2. Rotate

2. Rotate the insulin injection sites systematically.
Rationale:
Insulin dosages should not be adjusted or increased before unusual exercise. If acetone is found in the urine, it may possibly indicate the need for additional insulin. To minimize the disco

24.) A nurse is reinforcing teaching for a client regarding how to mix regular insulin and NPH insulin in the same syringe. Which of the following actions, if performed by the client, indicates the need for further teaching?
1. Withdraws the NPH insulin f

1. Withdraws the NPH insulin first
Rationale:
When preparing a mixture of regular insulin with another insulin preparation, the regular insulin is drawn into the syringe first. This sequence will avoid contaminating the vial of regular insulin with insuli

25.) A home care nurse visits a client recently diagnosed with diabetes mellitus who is taking Humulin NPH insulin daily. The client asks the nurse how to store the unopened vials of insulin. The nurse tells the client to:
1. Freeze the insulin.
2. Refrig

2. Refrigerate the insulin.
Rationale:
Insulin in unopened vials should be stored under refrigeration until needed. Vials should not be frozen. When stored unopened under refrigeration, insulin can be used up to the expiration date on the vial. Options 1,

26.) Glimepiride (Amaryl) is prescribed for a client with diabetes mellitus. A nurse reinforces instructions for the client and tells the client to avoid which of the following while taking this medication?
1. Alcohol
2. Organ meats
3. Whole-grain cereals

1. Alcohol
Rationale:
When alcohol is combined with glimepiride (Amaryl), a disulfiram-like reaction may occur. This syndrome includes flushing, palpitations, and nausea. Alcohol can also potentiate the hypoglycemic effects of the medication. Clients need

27.) Sildenafil (Viagra) is prescribed to treat a client with erectile dysfunction. A nurse reviews the client's medical record and would question the prescription if which of the following is noted in the client's history?
1. Neuralgia
2. Insomnia
3. Use

3. Use of nitroglycerin
Rationale:
Sildenafil (Viagra) enhances the vasodilating effect of nitric oxide in the corpus cavernosum of the penis, thus sustaining an erection. Because of the effect of the medication, it is contraindicated with concurrent use

28.) The health care provider (HCP) prescribes exenatide (Byetta) for a client with type 1 diabetes mellitus who takes insulin. The nurse knows that which of the following is the appropriate intervention?
1. The medication is administered within 60 minute

2. The medication is withheld and the HCP is called to question the prescription for the client.
Rationale:
Exenatide (Byetta) is an incretin mimetic used for type 2 diabetes mellitus only. It is not recommended for clients taking insulin. Hence, the nurs

29.) A client is taking Humulin NPH insulin daily every morning. The nurse reinforces instructions for the client and tells the client that the most likely time for a hypoglycemic reaction to occur is:
1. 2 to 4 hours after administration
2. 4 to 12 hours

2. 4 to 12 hours after administration
Rationale:
Humulin NPH is an intermediate-acting insulin. The onset of action is 1.5 hours, it peaks in 4 to 12 hours, and its duration of action is 24 hours. Hypoglycemic reactions most likely occur during peak time.

30.) A client with diabetes mellitus visits a health care clinic. The client's diabetes mellitus previously had been well controlled with glyburide (DiaBeta) daily, but recently the fasting blood glucose level has been 180 to 200 mg/dL. Which medication,

1. Prednisone
Rationale:
Prednisone may decrease the effect of oral hypoglycemics, insulin, diuretics, and potassium supplements. Option 2, a monoamine oxidase inhibitor, and option 3, a ?-blocker, have their own intrinsic hypoglycemic activity. Option 4

31.) A community health nurse visits a client at home. Prednisone 10 mg orally daily has been prescribed for the client and the nurse reinforces teaching for the client about the medication. Which statement, if made by the client, indicates that further t

1. "I can take aspirin or my antihistamine if I need it."
Rationale:
Aspirin and other over-the-counter medications should not be taken unless the client consults with the health care provider (HCP). The client needs to take the medication at the same tim

32.) Desmopressin acetate (DDAVP) is prescribed for the treatment of diabetes insipidus. The nurse monitors the client after medication administration for which therapeutic response?
1. Decreased urinary output
2. Decreased blood pressure
3. Decreased per

1. Decreased urinary output
Rationale:
Desmopressin promotes renal conservation of water. The hormone carries out this action by acting on the collecting ducts of the kidney to increase their permeability to water, which results in increased water reabsor

33.) The home health care nurse is visiting a client who was recently diagnosed with type 2 diabetes mellitus. The client is prescribed repaglinide (Prandin) and metformin (Glucophage) and asks the nurse to explain these medications. The nurse should rein

1. Diarrhea can occur secondary to the metformin.
2. The repaglinide is not taken if a meal is skipped.
3. The repaglinide is taken 30 minutes before eating.
4. Candy or another simple sugar is carried and used to treat mild hypoglycemia episodes.
Rationa

34.) A client with Crohn's disease is scheduled to receive an infusion of infliximab (Remicade). The nurse assisting in caring for the client should take which action to monitor the effectiveness of treatment?
1. Monitoring the leukocyte count for 2 days

2. Checking the frequency and consistency of bowel movements
Rationale:
The principal manifestations of Crohn's disease are diarrhea and abdominal pain. Infliximab (Remicade) is an immunomodulator that reduces the degree of inflammation in the colon, ther

35.) The client has a PRN prescription for loperamide hydrochloride (Imodium). The nurse understands that this medication is used for which condition?
1. Constipation
2. Abdominal pain
3. An episode of diarrhea
4. Hematest-positive nasogastric tube draina

3. An episode of diarrhea
Rationale:
Loperamide is an antidiarrheal agent. It is used to manage acute and also chronic diarrhea in conditions such as inflammatory bowel disease. Loperamide also can be used to reduce the volume of drainage from an ileostom

36.) The client has a PRN prescription for ondansetron (Zofran). For which condition should this medication be administered to the postoperative client?
1. Paralytic ileus
2. Incisional pain
3. Urinary retention
4. Nausea and vomiting

4. Nausea and vomiting
Rationale:
Ondansetron is an antiemetic used to treat postoperative nausea and vomiting, as well as nausea and vomiting associated with chemotherapy. The other options are incorrect.

37.) The client has begun medication therapy with pancrelipase (Pancrease MT). The nurse evaluates that the medication is having the optimal intended benefit if which effect is observed?
1. Weight loss
2. Relief of heartburn
3. Reduction of steatorrhea
4.

3. Reduction of steatorrhea
Rationale:
Pancrelipase (Pancrease MT) is a pancreatic enzyme used in clients with pancreatitis as a digestive aid. The medication should reduce the amount of fatty stools (steatorrhea). Another intended effect could be improve

38.) An older client recently has been taking cimetidine (Tagamet). The nurse monitors the client for which most frequent central nervous system side effect of this medication?
1. Tremors
2. Dizziness
3. Confusion
4. Hallucinations

3. Confusion
Rationale:
Cimetidine is a histamine 2 (H2)-receptor antagonist. Older clients are especially susceptible to central nervous system side effects of cimetidine. The most frequent of these is confusion. Less common central nervous system side e

39.) The client with a gastric ulcer has a prescription for sucralfate (Carafate), 1 g by mouth four times daily. The nurse schedules the medication for which times?
1. With meals and at bedtime
2. Every 6 hours around the clock
3. One hour after meals an

4. One hour before meals and at bedtime
Rationale:
Sucralfate is a gastric protectant. The medication should be scheduled for administration 1 hour before meals and at bedtime. The medication is timed to allow it to form a protective coating over the ulce

40.) The client who chronically uses nonsteroidal anti-inflammatory drugs has been taking misoprostol (Cytotec). The nurse determines that the medication is having the intended therapeutic effect if which of the following is noted?
1. Resolved diarrhea
2.

2. Relief of epigastric pain
Rationale:
The client who chronically uses nonsteroidal anti-inflammatory drugs (NSAIDs) is prone to gastric mucosal injury. Misoprostol is a gastric protectant and is given specifically to prevent this occurrence. Diarrhea ca

41.) The client has been taking omeprazole (Prilosec) for 4 weeks. The ambulatory care nurse evaluates that the client is receiving optimal intended effect of the medication if the client reports the absence of which symptom?
1. Diarrhea
2. Heartburn
3. F

2. Heartburn
Rationale:
Omeprazole is a proton pump inhibitor classified as an antiulcer agent. The intended effect of the medication is relief of pain from gastric irritation, often called heartburn by clients. Omeprazole is not used to treat the conditi

42.) A client with a peptic ulcer is diagnosed with a Helicobacter pylori infection. The nurse is reinforcing teaching for the client about the medications prescribed, including clarithromycin (Biaxin), esomeprazole (Nexium), and amoxicillin (Amoxil). Whi

3. "The medications will kill the bacteria and stop the acid production."
Rationale:
Triple therapy for Helicobacter pylori infection usually includes two antibacterial drugs and a proton pump inhibitor. Clarithromycin and amoxicillin are antibacterials.

43.) A histamine (H2)-receptor antagonist will be prescribed for a client. The nurse understands that which medications are H2-receptor antagonists? Select all that apply.
1. Nizatidine (Axid)
2. Ranitidine (Zantac)
3. Famotidine (Pepcid)
4. Cimetidine (T

1. Nizatidine (Axid)
2. Ranitidine (Zantac)
3. Famotidine (Pepcid)
4. Cimetidine (Tagamet)
Rationale:
H2-receptor antagonists suppress secretion of gastric acid, alleviate symptoms of heartburn, and assist in preventing complications of peptic ulcer disea

44.) A client is receiving acetylcysteine (Mucomyst), 20% solution diluted in 0.9% normal saline by nebulizer. The nurse should have which item available for possible use after giving this medication?
1. Ambu bag
2. Intubation tray
3. Nasogastric tube
4.

4. Suction equipment
Rationale:
Acetylcysteine can be given orally or by nasogastric tube to treat acetaminophen overdose, or it may be given by inhalation for use as a mucolytic. The nurse administering this medication as a mucolytic should have suction

45.) A client has a prescription to take guaifenesin (Humibid) every 4 hours, as needed. The nurse determines that the client understands the most effective use of this medication if the client states that he or she will:
1. Watch for irritability as a si

2. Take the tablet with a full glass of water.
Rationale:
Guaifenesin is an expectorant. It should be taken with a full glass of water to decrease viscosity of secretions. Sustained-release preparations should not be broken open, crushed, or chewed. The m

46.) A postoperative client has received a dose of naloxone hydrochloride for respiratory depression shortly after transfer to the nursing unit from the postanesthesia care unit. After administration of the medication, the nurse checks the client for:
1.

3. Sudden increase in pain
Rationale:
Naloxone hydrochloride is an antidote to opioids and may also be given to the postoperative client to treat respiratory depression. When given to the postoperative client for respiratory depression, it may also revers

47.) A client has been taking isoniazid (INH) for 2 months. The client complains to a nurse about numbness, paresthesias, and tingling in the extremities. The nurse interprets that the client is experiencing:
1. Hypercalcemia
2. Peripheral neuritis
3. Sma

2. Peripheral neuritis
Rationale:
A common side effect of the TB drug INH is peripheral neuritis. This is manifested by numbness, tingling, and paresthesias in the extremities. This side effect can be minimized by pyridoxine (vitamin B6) intake. Options 1

48.) A client is to begin a 6-month course of therapy with isoniazid (INH). A nurse plans to teach the client to:
1. Drink alcohol in small amounts only.
2. Report yellow eyes or skin immediately.
3. Increase intake of Swiss or aged cheeses.
4. Avoid vita

2. Report yellow eyes or skin immediately.
Rationale:
INH is hepatotoxic, and therefore the client is taught to report signs and symptoms of hepatitis immediately (which include yellow skin and sclera). For the same reason, alcohol should be avoided durin

49.) A client has been started on long-term therapy with rifampin (Rifadin). A nurse teaches the client that the medication:
1. Should always be taken with food or antacids
2. Should be double-dosed if one dose is forgotten
3. Causes orange discoloration

3. Causes orange discoloration of sweat, tears, urine, and feces
Rationale:
Rifampin should be taken exactly as directed as part of TB therapy. Doses should not be doubled or skipped. The client should not stop therapy until directed to do so by a health

50.) A nurse has given a client taking ethambutol (Myambutol) information about the medication. The nurse determines that the client understands the instructions if the client states that he or she will immediately report:
1. Impaired sense of hearing
2.

2. Problems with visual acuity
Rationale:
Ethambutol causes optic neuritis, which decreases visual acuity and the ability to discriminate between the colors red and green. This poses a potential safety hazard when a client is driving a motor vehicle. The

51.) Cycloserine (Seromycin) is added to the medication regimen for a client with tuberculosis. Which of the following would the nurse include in the client-teaching plan regarding this medication?
1. To take the medication before meals
2. To return to th

2. To return to the clinic weekly for serum drug-level testing
Rationale:
Cycloserine (Seromycin) is an antitubercular medication that requires weekly serum drug level determinations to monitor for the potential of neurotoxicity. Serum drug levels lower t

52.) A client with tuberculosis is being started on antituberculosis therapy with isoniazid (INH). Before giving the client the first dose, a nurse ensures that which of the following baseline studies has been completed?
1. Electrolyte levels
2. Coagulati

3. Liver enzyme levels
Rationale:
INH therapy can cause an elevation of hepatic enzyme levels and hepatitis. Therefore, liver enzyme levels are monitored when therapy is initiated and during the first 3 months of therapy. They may be monitored longer in t

53.) Rifabutin (Mycobutin) is prescribed for a client with active Mycobacterium avium complex (MAC) disease and tuberculosis. The nurse monitors for which side effects of the medication? Select all that apply.
1. Signs of hepatitis
2. Flu-like syndrome
3.

1. Signs of hepatitis
2. Flu-like syndrome
3. Low neutrophil count
5. Ocular pain or blurred vision
Rationale:
Rifabutin (Mycobutin) may be prescribed for a client with active MAC disease and tuberculosis. It inhibits mycobacterial DNA-dependent RNA polym

54.) A nurse reinforces discharge instructions to a postoperative client who is taking warfarin sodium (Coumadin). Which statement, if made by the client, reflects the need for further teaching?
1. "I will take my pills every day at the same time."
2. "I

4. "I will take Ecotrin (enteric-coated aspirin) for my headaches because it is coated."
Rationale:
Ecotrin is an aspirin-containing product and should be avoided. Alcohol consumption should be avoided by a client taking warfarin sodium. Taking prescribed

55.) A client who is receiving digoxin (Lanoxin) daily has a serum potassium level of 3.0 mEq/L and is complaining of anorexia. A health care provider prescribes a digoxin level to rule out digoxin toxicity. A nurse checks the results, knowing that which

2.) 0.5 to 2 ng/mL
Rationale:
Therapeutic levels for digoxin range from 0.5 to 2 ng/mL. Therefore, options 1, 3, and 4 are incorrect.

56.) Heparin sodium is prescribed for the client. The nurse expects that the health care provider will prescribe which of the following to monitor for a therapeutic effect of the medication?
1. Hematocrit level
2. Hemoglobin level
3. Prothrombin time (PT)

4. Activated partial thromboplastin time (aPTT)
Rationale:
The PT will assess for the therapeutic effect of warfarin sodium (Coumadin) and the aPTT will assess the therapeutic effect of heparin sodium. Heparin sodium doses are determined based on these la

57.) A nurse is monitoring a client who is taking propranolol (Inderal LA). Which data collection finding would indicate a potential serious complication associated with propranolol?
1. The development of complaints of insomnia
2. The development of audib

2. The development of audible expiratory wheezes
Rationale:
Audible expiratory wheezes may indicate a serious adverse reaction, bronchospasm. ?-Blockers may induce this reaction, particularly in clients with chronic obstructive pulmonary disease or asthma

58.) Isosorbide mononitrate (Imdur) is prescribed for a client with angina pectoris. The client tells the nurse that the medication is causing a chronic headache. The nurse appropriately suggests that the client:
1. Cut the dose in half.
2. Discontinue th

3. Take the medication with food.
Rationale:
Isosorbide mononitrate is an antianginal medication. Headache is a frequent side effect of isosorbide mononitrate and usually disappears during continued therapy. If a headache occurs during therapy, the client

59.) A client is diagnosed with an acute myocardial infarction and is receiving tissue plasminogen activator, alteplase (Activase, tPA). Which action is a priority nursing intervention?
1. Monitor for renal failure.
2. Monitor psychosocial status.
3. Moni

3. Monitor for signs of bleeding.
Rationale:
Tissue plasminogen activator is a thrombolytic. Hemorrhage is a complication of any type of thrombolytic medication. The client is monitored for bleeding. Monitoring for renal failure and monitoring the client'

60.) A nurse is planning to administer hydrochlorothiazide (HydroDIURIL) to a client. The nurse understands that which of the following are concerns related to the administration of this medication?
1. Hypouricemia, hyperkalemia
2. Increased risk of osteo

3. Hypokalemia, hyperglycemia, sulfa allergy
Rationale:
Thiazide diuretics such as hydrochlorothiazide are sulfa-based medications, and a client with a sulfa allergy is at risk for an allergic reaction. Also, clients are at risk for hypokalemia, hyperglyc

61.) A home health care nurse is visiting a client with elevated triglyceride levels and a serum cholesterol level of 398 mg/dL. The client is taking cholestyramine (Questran). Which of the following statements, if made by the client, indicates the need f

4. "I'll continue my nicotinic acid from the health food store."
Rationale:
Nicotinic acid, even an over-the-counter form, should be avoided because it may lead to liver abnormalities. All lipid-lowering medications also can cause liver abnormalities, so

62.) A client is on nicotinic acid (niacin) for hyperlipidemia and the nurse provides instructions to the client about the medication. Which statement by the client would indicate an understanding of the instructions?
1. "It is not necessary to avoid the

4. "Ibuprofen (Motrin) taken 30 minutes before the nicotinic acid should decrease the flushing."
Rationale:
Flushing is a side effect of this medication. Aspirin or a nonsteroidal anti-inflammatory drug can be taken 30 minutes before taking the medication

63.) A client with coronary artery disease complains of substernal chest pain. After checking the client's heart rate and blood pressure, a nurse administers nitroglycerin, 0.4 mg, sublingually. After 5 minutes, the client states, "My chest still hurts.

2. Contact the registered nurse.
4. Assess the client's pain level.
5. Check the client's blood pressure.
6. Administer a second nitroglycerin, 0.4 mg, sublingually.
Rationale:
The usual guideline for administering nitroglycerin tablets for a hospitalized

64.) Nalidixic acid (NegGram) is prescribed for a client with a urinary tract infection. On review of the client's record, the nurse notes that the client is taking warfarin sodium (Coumadin) daily. Which prescription should the nurse anticipate for this

2. A decrease in the warfarin sodium (Coumadin) dosage
Rationale:
Nalidixic acid can intensify the effects of oral anticoagulants by displacing these agents from binding sites on plasma protein. When an oral anticoagulant is combined with nalidixic acid,

65.) A nurse is reinforcing discharge instructions to a client receiving sulfisoxazole. Which of the following should be included in the list of instructions?
1. Restrict fluid intake.
2. Maintain a high fluid intake.
3. If the urine turns dark brown, cal

2. Maintain a high fluid intake.
Rationale:
Each dose of sulfisoxazole should be administered with a full glass of water, and the client should maintain a high fluid intake. The medication is more soluble in alkaline urine. The client should not be instru

66.) Trimethoprim-sulfamethoxazole (TMP-SMZ) is prescribed for a client. A nurse should instruct the client to report which symptom if it developed during the course of this medication therapy?
1. Nausea
2. Diarrhea
3. Headache
4. Sore throat

4. Sore throat
Rationale:
Clients taking trimethoprim-sulfamethoxazole (TMP-SMZ) should be informed about early signs of blood disorders that can occur from this medication. These include sore throat, fever, and pallor, and the client should be instructed

67.) Phenazopyridine hydrochloride (Pyridium) is prescribed for a client for symptomatic relief of pain resulting from a lower urinary tract infection. The nurse reinforces to the client:
1. To take the medication at bedtime
2. To take the medication befo

4. That a reddish orange discoloration of the urine may occur
Rationale:
The nurse should instruct the client that a reddish-orange discoloration of urine may occur. The nurse also should instruct the client that this discoloration can stain fabric. The m

68.) Bethanechol chloride (Urecholine) is prescribed for a client with urinary retention. Which disorder would be a contraindication to the administration of this medication?
1. Gastric atony
2. Urinary strictures
3. Neurogenic atony
4. Gastroesophageal r

2. Urinary strictures
Rationale:
Bethanechol chloride (Urecholine) can be harmful to clients with urinary tract obstruction or weakness of the bladder wall. The medication has the ability to contract the bladder and thereby increase pressure within the ur

69.) A nurse who is administering bethanechol chloride (Urecholine) is monitoring for acute toxicity associated with the medication. The nurse checks the client for which sign of toxicity?
1. Dry skin
2. Dry mouth
3. Bradycardia
4. Signs of dehydration

3. Bradycardia
Rationale:
Toxicity (overdose) produces manifestations of excessive muscarinic stimulation such as salivation, sweating, involuntary urination and defecation, bradycardia, and severe hypotension. Treatment includes supportive measures and t

70.) Oxybutynin chloride (Ditropan XL) is prescribed for a client with neurogenic bladder. Which sign would indicate a possible toxic effect related to this medication?
1. Pallor
2. Drowsiness
3. Bradycardia
4. Restlessness

4. Restlessness
Rationale:
Toxicity (overdosage) of this medication produces central nervous system excitation, such as nervousness, restlessness, hallucinations, and irritability. Other signs of toxicity include hypotension or hypertension, confusion, ta

71.) After kidney transplantation, cyclosporine (Sand immune) is prescribed for a client. Which laboratory result would indicate an adverse effect from the use of this medication?
1. Decreased creatinine level
2. Decreased hemoglobin level
3. Elevated blo

3. Elevated blood urea nitrogen level
Rationale:
Nephrotoxicity can occur from the use of cyclosporine (Sandimmune). Nephrotoxicity is evaluated by monitoring for elevated blood urea nitrogen (BUN) and serum creatinine levels. Cyclosporine is an immunosup

72.) Cinoxacin (Cinobac), a urinary antiseptic, is prescribed for the client. The nurse reviews the client's medical record and should contact the health care provider (HCP) regarding which documented finding to verify the prescription? Refer to chart.
1.

1. Renal insufficiency
Rationale:
Cinoxacin should be administered with caution in clients with renal impairment. The dosage should be reduced, and failure to do so could result in accumulation of cinoxacin to toxic levels. Therefore the nurse would verif

73.) A client with myasthenia gravis is suspected of having cholinergic crisis. Which of the following indicate that this crisis exists?
1. Ataxia
2. Mouth sores
3. Hypotension
4. Hypertension

4. Hypertension
Rationale:
Cholinergic crisis occurs as a result of an overdose of medication. Indications of cholinergic crisis include gastrointestinal disturbances, nausea, vomiting, diarrhea, abdominal cramps, increased salivation and tearing, miosis,

74.) A client with myasthenia gravis is receiving pyridostigmine (Mestinon). The nurse monitors for signs and symptoms of cholinergic crisis caused by overdose of the medication. The nurse checks the medication supply to ensure that which medication is av

2. Atropine sulfate
Rationale:
The antidote for cholinergic crisis is atropine sulfate. Vitamin K is the antidote for warfarin (Coumadin). Protamine sulfate is the antidote for heparin, and acetylcysteine (Mucomyst) is the antidote for acetaminophen (Tyle

75.) A client with myasthenia gravis becomes increasingly weak. The health care provider prepares to identify whether the client is reacting to an overdose of the medication (cholinergic crisis) or increasing severity of the disease (myasthenic crisis). A

4. A temporary worsening of the condition
Rationale:
An edrophonium (Enlon) injection, a cholinergic drug, makes the client in cholinergic crisis temporarily worse. This is known as a negative test. An improvement of weakness would occur if the client wer

76.) Carbidopa-levodopa (Sinemet) is prescribed for a client with Parkinson's disease, and the nurse monitors the client for adverse reactions to the medication. Which of the following indicates that the client is experiencing an adverse reaction?
1. Prur

4. Impaired voluntary movements
Rationale:
Dyskinesia and impaired voluntary movement may occur with high levodopa dosages. Nausea, anorexia, dizziness, orthostatic hypotension, bradycardia, and akinesia (the temporary muscle weakness that lasts 1 minute

77.) Phenytoin (Dilantin), 100 mg orally three times daily, has been prescribed for a client for seizure control. The nurse reinforces instructions regarding the medication to the client. Which statement by the client indicates an understanding of the ins

1. "I will use a soft toothbrush to brush my teeth."
Rationale:
Phenytoin (Dilantin) is an anticonvulsant. Gingival hyperplasia, bleeding, swelling, and tenderness of the gums can occur with the use of this medication. The client needs to be taught good o

78.) A client is taking phenytoin (Dilantin) for seizure control and a sample for a serum drug level is drawn. Which of the following indicates a therapeutic serum drug range?
1. 5 to 10 mcg/mL
2. 10 to 20 mcg/mL
3. 20 to 30 mcg/mL
4. 30 to 40 mcg/mL

2. 10 to 20 mcg/mL
Rationale:
The therapeutic serum drug level range for phenytoin (Dilantin) is 10 to 20 mcg/mL.
*
A helpful hint may be to remember that the theophylline therapeutic range and the acetaminophen (Tylenol) therapeutic range are the same as

79.) Ibuprofen (Advil) is prescribed for a client. The nurse tells the client to take the medication:
1. With 8 oz of milk
2. In the morning after arising
3. 60 minutes before breakfast
4. At bedtime on an empty stomach

1. With 8 oz of milk
Rationale:
Ibuprofen is a nonsteroidal anti-inflammatory drug (NSAID). NSAIDs should be given with milk or food to prevent gastrointestinal irritation. Options 2, 3, and 4 are incorrect.

80.) A nurse is caring for a client who is taking phenytoin (Dilantin) for control of seizures. During data collection, the nurse notes that the client is taking birth control pills. Which of the following information should the nurse provide to the clien

3. The potential for decreased effectiveness of the birth control pills exists while taking phenytoin (Dilantin).
Rationale:
Phenytoin (Dilantin) enhances the rate of estrogen metabolism, which can decrease the effectiveness of some birth control pills. O

81.) A client with trigeminal neuralgia is being treated with carbamazepine (Tegretol). Which laboratory result would indicate that the client is experiencing an adverse reaction to the medication?
1. Sodium level, 140 mEq/L
2. Uric acid level, 5.0 mg/dL

3. White blood cell count, 3000 cells/mm3
Rationale:
Adverse effects of carbamazepine (Tegretol) appear as blood dyscrasias, including aplastic anemia, agranulocytosis, thrombocytopenia, leukopenia, cardiovascular disturbances, thrombophlebitis, dysrhythm

82.) A client is receiving meperidine hydrochloride (Demerol) for pain. Which of the following are side effects of this medication. Select all that apply.
1. Diarrhea
2. Tremors
3. Drowsiness
4. Hypotension
5. Urinary frequency
6. Increased respiratory ra

2. Tremors
3. Drowsiness
4. Hypotension
Rationale:
Meperidine hydrochloride is an opioid analgesic. Side effects include respiratory depression, drowsiness, hypotension, constipation, urinary retention, nausea, vomiting, and tremors.

83.) The client has been on treatment for rheumatoid arthritis for 3 weeks. During the administration of etanercept (Enbrel), it is most important for the nurse to check:
1. The injection site for itching and edema
2. The white blood cell counts and plate

2. The white blood cell counts and platelet counts
Rationale:
Infection and pancytopenia are side effects of etanercept (Enbrel). Laboratory studies are performed before and during drug treatment. The appearance of abnormal white blood cell counts and abn

84.) Baclofen (Lioresal) is prescribed for the client with multiple sclerosis. The nurse assists in planning care, knowing that the primary therapeutic effect of this medication is which of the following?
1. Increased muscle tone
2. Decreased muscle spasm

2. Decreased muscle spasms
Rationale:
Baclofen is a skeletal muscle relaxant and central nervous system depressant and acts at the spinal cord level to decrease the frequency and amplitude of muscle spasms in clients with spinal cord injuries or diseases

85.) A nurse is monitoring a client receiving baclofen (Lioresal) for side effects related to the medication. Which of the following would indicate that the client is experiencing a side effect?
1. Polyuria
2. Diarrhea
3. Drowsiness
4. Muscular excitabili

3. Drowsiness
Rationale:
Baclofen is a central nervous system (CNS) depressant and frequently causes drowsiness, dizziness, weakness, and fatigue. It can also cause nausea, constipation, and urinary retention. Clients should be warned about the possible r

86.) A nurse is reinforcing discharge instructions to a client receiving baclofen (Lioresal). Which of the following would the nurse include in the instructions?
1. Restrict fluid intake.
2. Avoid the use of alcohol.
3. Stop the medication if diarrhea occ

2. Avoid the use of alcohol.
Rationale:
Baclofen is a central nervous system (CNS) depressant. The client should be cautioned against the use of alcohol and other CNS depressants, because baclofen potentiates the depressant activity of these agents. Const

87.) A client with acute muscle spasms has been taking baclofen (Lioresal). The client calls the clinic nurse because of continuous feelings of weakness and fatigue and asks the nurse about discontinuing the medication. The nurse should make which appropr

4. "Weakness and fatigue commonly occur and will diminish with continued medication use."
Rationale:
The client should be instructed that symptoms such as drowsiness, weakness, and fatigue are more intense in the early phase of therapy and diminish with c

88.) Dantrolene sodium (Dantrium) is prescribed for a client experiencing flexor spasms, and the client asks the nurse about the action of the medication. The nurse responds, knowing that the therapeutic action of this medication is which of the following

2. Acts directly on the skeletal muscle to relieve spasticity
Rationale:
Dantrium acts directly on skeletal muscle to relieve muscle spasticity. The primary action is the suppression of calcium release from the sarcoplasmic reticulum. This in turn decreas

89.) A nurse is reviewing the laboratory studies on a client receiving dantrolene sodium (Dantrium). Which laboratory test would identify an adverse effect associated with the administration of this medication?
1. Creatinine
2. Liver function tests
3. Blo

2. Liver function tests
Rationale:
Dose-related liver damage is the most serious adverse effect of dantrolene. To reduce the risk of liver damage, liver function tests should be performed before treatment and periodically throughout the treatment course.

90.) A nurse is reviewing the record of a client who has been prescribed baclofen (Lioresal). Which of the following disorders, if noted in the client's history, would alert the nurse to contact the health care provider?
1. Seizure disorders
2. Hyperthyro

1. Seizure disorders
Rationale:
Clients with seizure disorders may have a lowered seizure threshold when baclofen is administered. Concurrent therapy may require an increase in the anticonvulsive medication. The disorders in options 2, 3, and 4 are not a

91.) Cyclobenzaprine (Flexeril) is prescribed for a client to treat muscle spasms, and the nurse is reviewing the client's record. Which of the following disorders, if noted in the client's record, would indicate a need to contact the health care provider

1. Glaucoma
Rationale:
Because this medication has anticholinergic effects, it should be used with caution in clients with a history of urinary retention, angle-closure glaucoma, and increased intraocular pressure. Cyclobenzaprine hydrochloride should be

92.) In monitoring a client's response to disease-modifying antirheumatic drugs (DMARDs), which findings would the nurse interpret as acceptable responses? Select all that apply.
1. Symptom control during periods of emotional stress
2. Normal white blood

1. Symptom control during periods of emotional stress
2. Normal white blood cell counts, platelet, and neutrophil counts
3. Radiological findings that show nonprogression of joint degeneration
4. An increased range of motion in the affected joints 3 month

93.) The client who is human immunodeficiency virus seropositive has been taking stavudine (d4t, Zerit). The nurse monitors which of the following most closely while the client is taking this medication?
1. Gait
2. Appetite
3. Level of consciousness
4. He

1. Gait
Rationale:
Stavudine (d4t, Zerit) is an antiretroviral used to manage human immunodeficiency virus infection in clients who do not respond to or who cannot tolerate conventional therapy. The medication can cause peripheral neuropathy, and the nurs

94.) The client with acquired immunodeficiency syndrome has begun therapy with zidovudine (Retrovir, Azidothymidine, AZT, ZDV). The nurse carefully monitors which of the following laboratory results during treatment with this medication?
1. Blood culture

4. Complete blood count
Rationale:
A common side effect of therapy with zidovudine is leukopenia and anemia. The nurse monitors the complete blood count results for these changes. Options 1, 2, and 3 are unrelated to the use of this medication.

95.) The nurse is reviewing the results of serum laboratory studies drawn on a client with acquired immunodeficiency syndrome who is receiving didanosine (Videx). The nurse interprets that the client may have the medication discontinued by the health care

3. Serum amylase
Rationale:
Didanosine (Videx) can cause pancreatitis. A serum amylase level that is increased 1.5 to 2 times normal may signify pancreatitis in the client with acquired immunodeficiency syndrome and is potentially fatal. The medication ma

96.) The nurse is caring for a postrenal transplant client taking cyclosporine (Sandimmune, Gengraf, Neoral). The nurse notes an increase in one of the client's vital signs, and the client is complaining of a headache. What is the vital sign that is most

3. Blood pressure
Rationale:
Hypertension can occur in a client taking cyclosporine (Sandimmune, Gengraf, Neoral), and because this client is also complaining of a headache, the blood pressure is the vital sign to be monitoring most closely. Other adverse

97.) Amikacin (Amikin) is prescribed for a client with a bacterial infection. The client is instructed to contact the health care provider (HCP) immediately if which of the following occurs?
1. Nausea
2. Lethargy
3. Hearing loss
4. Muscle aches

3. Hearing loss
Rationale:
Amikacin (Amikin) is an aminoglycoside. Adverse effects of aminoglycosides include ototoxicity (hearing problems), confusion, disorientation, gastrointestinal irritation, palpitations, blood pressure changes, nephrotoxicity, and

98.) The nurse is assigned to care for a client with cytomegalovirus retinitis and acquired immunodeficiency syndrome who is receiving foscarnet. The nurse should check the latest results of which of the following laboratory studies while the client is ta

3. Serum creatinine
Rationale:
Foscarnet is toxic to the kidneys. Serum creatinine is monitored before therapy, two to three times per week during induction therapy, and at least weekly during maintenance therapy. Foscarnet may also cause decreased levels

99.) The client with acquired immunodeficiency syndrome and Pneumocystis jiroveci infection has been receiving pentamidine isethionate (Pentam 300). The client develops a temperature of 101� F. The nurse does further monitoring of the client, knowing that

4. The result of another infection caused by leukopenic effects of the medication.
Rationale:
Frequent side effects of this medication include leukopenia, thrombocytopenia, and anemia. The client should be monitored routinely for signs and symptoms of inf

100.) Saquinavir (Invirase) is prescribed for the client who is human immunodeficiency virus seropositive. The nurse reinforces medication instructions and tells the client to:
1. Avoid sun exposure.
2. Eat low-calorie foods.
3. Eat foods that are low in

1. Avoid sun exposure.
Rationale:
Saquinavir (Invirase) is an antiretroviral (protease inhibitor) used with other antiretroviral medications to manage human immunodeficiency virus infection. Saquinavir is administered with meals and is best absorbed if th

101.) Ketoconazole is prescribed for a client with a diagnosis of candidiasis. Select the interventions that the nurse includes when administering this medication. Select all that apply.
1. Restrict fluid intake.
2. Instruct the client to avoid alcohol.
3

2. Instruct the client to avoid alcohol.
3. Monitor hepatic and liver function studies.
5. Instruct the client to avoid exposure to the sun.
Rationale:
Ketoconazole is an antifungal medication. It is administered with food (not on an empty stomach) and an

102.) A client with human immunodeficiency virus is taking nevirapine (Viramune). The nurse should monitor for which adverse effects of the medication? Select all that apply.
1. Rash
2. Hepatotoxicity
3. Hyperglycemia
4. Peripheral neuropathy
5. Reduced b

1. Rash
2. Hepatotoxicity
Rationale:
Nevirapine (Viramune) is a non-nucleoside reverse transcriptase inhibitors (NRTI) that is used to treat HIV infection. It is used in combination with other antiretroviral medications to treat HIV. Adverse effects inclu

103.) A nurse is caring for a hospitalized client who has been taking clozapine (Clozaril) for the treatment of a schizophrenic disorder. Which laboratory study prescribed for the client will the nurse specifically review to monitor for an adverse effect

3. White blood cell count
Rationale:
Hematological reactions can occur in the client taking clozapine and include agranulocytosis and mild leukopenia. The white blood cell count should be checked before initiating treatment and should be monitored closely

104.) Disulfiram (Antabuse) is prescribed for a client who is seen in the psychiatric health care clinic. The nurse is collecting data on the client and is providing instructions regarding the use of this medication. Which is most important for the nurse

4. When the last alcoholic drink was consumed
Rationale:
Disulfiram is used as an adjunct treatment for selected clients with chronic alcoholism who want to remain in a state of enforced sobriety. Clients must abstain from alcohol intake for at least 12 h

105.) A nurse is collecting data from a client and the client's spouse reports that the client is taking donepezil hydrochloride (Aricept). Which disorder would the nurse suspect that this client may have based on the use of this medication?
1. Dementia
2

1. Dementia
Rationale:
Donepezil hydrochloride is a cholinergic agent used in the treatment of mild to moderate dementia of the Alzheimer type. It enhances cholinergic functions by increasing the concentration of acetylcholine. It slows the progression of

106.) Fluoxetine (Prozac) is prescribed for the client. The nurse reinforces instructions to the client regarding the administration of the medication. Which statement by the client indicates an understanding about administration of the medication?
1. "I

3. "I should take the medication in the morning when I first arise."
Rationale:
Fluoxetine hydrochloride is administered in the early morning without consideration to meals.
*
Eliminate options 1, 2, and 4 because they are comparable or alike and indicate

107.) A client receiving a tricyclic antidepressant arrives at the mental health clinic. Which observation indicates that the client is correctly following the medication plan?
1. Reports not going to work for this past week
2. Complains of not being able

3. Arrives at the clinic neat and appropriate in appearance
Rationale:
Depressed individuals will sleep for long periods, are not able to go to work, and feel as if they cannot "do anything." Once they have had some therapeutic effect from their medicatio

108.) A nurse is performing a follow-up teaching session with a client discharged 1 month ago who is taking fluoxetine (Prozac). What information would be important for the nurse to gather regarding the adverse effects related to the medication?
1. Cardio

2. Gastrointestinal dysfunctions
Rationale:
The most common adverse effects related to fluoxetine include central nervous system (CNS) and gastrointestinal (GI) system dysfunction. This medication affects the GI system by causing nausea and vomiting, cram

109.) A client taking buspirone (BuSpar) for 1 month returns to the clinic for a follow-up visit. Which of the following would indicate medication effectiveness?
1. No rapid heartbeats or anxiety
2. No paranoid thought processes
3. No thought broadcasting

1. No rapid heartbeats or anxiety
Rationale:
Buspirone hydrochloride is not recommended for the treatment of drug or alcohol withdrawal, paranoid thought disorders, or schizophrenia (thought broadcasting or delusions). Buspirone hydrochloride is most ofte

110.) A client taking lithium carbonate (Lithobid) reports vomiting, abdominal pain, diarrhea, blurred vision, tinnitus, and tremors. The lithium level is checked as a part of the routine follow-up and the level is 3.0 mEq/L. The nurse knows that this lev

1. Toxic
Rationale:
The therapeutic serum level of lithium is 0.6 to 1.2 mEq/L. A level of 3 mEq/L indicates toxicity.

111.) A client arrives at the health care clinic and tells the nurse that he has been doubling his daily dosage of bupropion hydrochloride (Wellbutrin) to help him get better faster. The nurse understands that the client is now at risk for which of the fo

3. Seizure activity
Rationale:
Bupropion does not cause significant orthostatic blood pressure changes. Seizure activity is common in dosages greater than 450 mg daily. Bupropion frequently causes a drop in body weight. Insomnia is a side effect, but seiz

112.) A hospitalized client is started on phenelzine sulfate (Nardil) for the treatment of depression. The nurse instructs the client to avoid consuming which foods while taking this medication? Select all that apply.
1. Figs
2. Yogurt
3. Crackers
4. Aged

1. Figs
2. Yogurt
4. Aged cheese
Rationale:
Phenelzine sulfate (Nardil) is a monoamine oxidase inhibitor(MAOI). The client should avoid taking in foods that are high in tyramine. Use of these foods could trigger a potentially fatal hypertensive crisis. So

113.) A nurse is reinforcing discharge instructions to a client receiving sulfisoxazole. Which of the following would be included in the plan of care for instructions?
1. Maintain a high fluid intake.
2. Discontinue the medication when feeling better.
3.

1. Maintain a high fluid intake.
Rationale:
Each dose of sulfisoxazole should be administered with a full glass of water, and the client should maintain a high fluid intake. The medication is more soluble in alkaline urine. The client should not be instru

114.) A postoperative client requests medication for flatulence (gas pains). Which medication from the following PRN list should the nurse administer to this client?
1. Ondansetron (Zofran)
2. Simethicone (Mylicon)
3. Acetaminophen (Tylenol)
4. Magnesium

2. Simethicone (Mylicon)
Rationale:
Simethicone is an antiflatulent used in the relief of pain caused by excessive gas in the gastrointestinal tract. Ondansetron is used to treat postoperative nausea and vomiting. Acetaminophen is a nonopioid analgesic. M

115.) A client received 20 units of NPH insulin subcutaneously at 8:00 AM. The nurse should check the client for a potential hypoglycemic reaction at what time?
1. 5:00 PM
2. 10:00 AM
3. 11:00 AM
4. 11:00 PM

1. 5:00 PM
Rationale:
NPH is intermediate-acting insulin. Its onset of action is 1 to 2� hours, it peaks in 4 to 12 hours, and its duration of action is 24 hours. Hypoglycemic reactions most likely occur during peak time.

116.) A nurse administers a dose of scopolamine (Transderm-Scop) to a postoperative client. The nurse tells the client to expect which of the following side effects of this medication?
1. Dry mouth
2. Diaphoresis
3. Excessive urination
4. Pupillary constr

1. Dry mouth
Rationale:
Scopolamine is an anticholinergic medication for the prevention of nausea and vomiting that causes the frequent side effects of dry mouth, urinary retention, decreased sweating, and dilation of the pupils. The other options describ

117.) A nurse has given the client taking ethambutol (Myambutol) information about the medication. The nurse determines that the client understands the instructions if the client immediately reports:
1. Impaired sense of hearing
2. Distressing gastrointes

4. Difficulty discriminating the color red from green
Rationale:
Ethambutol causes optic neuritis, which decreases visual acuity and the ability to discriminate between the colors red and green. This poses a potential safety hazard when driving a motor ve

118.) A nurse is caring for an older client with a diagnosis of myasthenia gravis and has reinforced self-care instructions. Which statement by the client indicates that further teaching is necessary?
1. "I rest each afternoon after my walk."
2. "I cough

4. "I can change the time of my medication on the mornings that I feel strong."
Rationale:
The client with myasthenia gravis should be taught that timing of anticholinesterase medication is critical. It is important to instruct the client to administer th

119.) A client with diabetes mellitus who has been controlled with daily insulin has been placed on atenolol (Tenormin) for the control of angina pectoris. Because of the effects of atenolol, the nurse determines that which of the following is the most re

4. Low blood glucose level
Rationale:
?-Adrenergic blocking agents, such as atenolol, inhibit the appearance of signs and symptoms of acute hypoglycemia, which would include nervousness, increased heart rate, and sweating. Therefore, the client receiving

120.) A client is taking lansoprazole (Prevacid) for the chronic management of Zollinger-Ellison syndrome. The nurse advises the client to take which of the following products if needed for a headache?
1. Naprosyn (Aleve)
2. Ibuprofen (Advil)
3. Acetamino

3. Acetaminophen (Tylenol)
Rationale:
Zollinger-Ellison syndrome is a hypersecretory condition of the stomach. The client should avoid taking medications that are irritating to the stomach lining. Irritants would include aspirin and nonsteroidal antiinfla

121.) A client who is taking hydrochlorothiazide (HydroDIURIL, HCTZ) has been started on triamterene (Dyrenium) as well. The client asks the nurse why both medications are required. The nurse formulates a response, based on the understanding that:
1. Both

4. Triamterene is a potassium-sparing diuretic, whereas hydrochlorothiazide is a potassium-losing diuretic.
Rationale:
Potassium-sparing diuretics include amiloride (Midamor), spironolactone (Aldactone), and triamterene (Dyrenium). They are weak diuretics

122.) A client who has begun taking fosinopril (Monopril) is very distressed, telling the nurse that he cannot taste food normally since beginning the medication 2 weeks ago. The nurse provides the best support to the client by:
1. Telling the client not

3. Informing the client that impaired taste is expected and generally disappears in 2 to 3 months
Rationale:
ACE inhibitors, such as fosinopril, cause temporary impairment of taste (dysgeusia). The nurse can tell the client that this effect usually disapp

123.) A nurse is planning to administer amlodipine (Norvasc) to a client. The nurse plans to check which of the following before giving the medication?
1. Respiratory rate
2. Blood pressure and heart rate
3. Heart rate and respiratory rate
4. Level of con

2. Blood pressure and heart rate
Rationale:
Amlodipine is a calcium channel blocker. This medication decreases the rate and force of cardiac contraction. Before administering a calcium channel blocking agent, the nurse should check the blood pressure and

124.) A client with chronic renal failure is receiving ferrous sulfate (Feosol). The nurse monitors the client for which common side effect associated with this medication?
1. Diarrhea
2. Weakness
3. Headache
4. Constipation

4. Constipation
Rationale:
Feosol is an iron supplement used to treat anemia. Constipation is a frequent and uncomfortable side effect associated with the administration of oral iron supplements. Stool softeners are often prescribed to prevent constipatio

125.) A nurse is preparing to administer digoxin (Lanoxin), 0.125 mg orally, to a client with heart failure. Which vital sign is most important for the nurse to check before administering the medication?
1. Heart rate
2. Temperature
3. Respirations
4. Blo

1. Heart rate
Rationale:
Digoxin is a cardiac glycoside that is used to treat heart failure and acts by increasing the force of myocardial contraction. Because bradycardia may be a clinical sign of toxicity, the nurse counts the apical heart rate for 1 fu

126.) A nurse is caring for a client who has been prescribed furosemide (Lasix) and is monitoring for adverse effects associated with this medication. Which of the following should the nurse recognize as a potential adverse effect Select all that apply.
1

2. Tinnitus
3. Hypotension
4. Hypokalemia
Rationale:
Furosemide is a loop diuretic; therefore, an expected effect is increased urinary frequency. Nausea is a frequent side effect, not an adverse effect. Photosensitivity is an occasional side effect. Adver

127.) The nurse provides medication instructions to an older hypertensive client who is taking 20 mg of lisinopril (Prinivil, Zestril) orally daily. The nurse evaluates the need for further teaching when the client states which of the following?
1. "I can

1. "I can skip a dose once a week."
Rationale:
Lisinopril is an antihypertensive angiotensin-converting enzyme (ACE) inhibitor. The usual dosage range is 20 to 40 mg per day. Adverse effects include headache, dizziness, fatigue, orthostatic hypotension, t

128.) A nurse is providing instructions to an adolescent who has a history of seizures and is taking an anticonvulsant medication. Which of the following statements indicates that the client understands the instructions?
1. "I will never be able to drive

3. "I can't drink alcohol while I am taking my medication."
Rationale:
Alcohol will lower the seizure threshold and should be avoided. Adolescents can obtain a driver's license in most states when they have been seizure free for 1 year. Anticonvulsants ca

129.) Megestrol acetate (Megace), an antineoplastic medication, is prescribed for the client with metastatic endometrial carcinoma. The nurse reviews the client's history and contacts the registered nurse if which diagnosis is documented in the client's h

3. Thrombophlebitis
Rationale:
Megestrol acetate (Megace) suppresses the release of luteinizing hormone from the anterior pituitary by inhibiting pituitary function and regressing tumor size. Megestrol is used with caution if the client has a history of t

130.) The nurse is analyzing the laboratory results of a client with leukemia who has received a regimen of chemotherapy. Which laboratory value would the nurse specifically note as a result of the massive cell destruction that occurred from the chemother

3. Increased uric acid level
Rationale:
Hyperuricemia is especially common following treatment for leukemias and lymphomas because chemotherapy results in a massive cell kill. Although options 1, 2, and 4 also may be noted, an increased uric acid level is

131.) The nurse is reinforcing medication instructions to a client with breast cancer who is receiving cyclophosphamide (Neosar). The nurse tells the client to:
1. Take the medication with food.
2. Increase fluid intake to 2000 to 3000 mL daily.
3. Decrea

2. Increase fluid intake to 2000 to 3000 mL daily.
Rationale:
Hemorrhagic cystitis is a toxic effect that can occur with the use of cyclophosphamide. The client needs to be instructed to drink copious amounts of fluid during the administration of this med

132.) The client with non-Hodgkin's lymphoma is receiving daunorubicin (DaunoXome). Which of the following would indicate to the nurse that the client is experiencing a toxic effect related to the medication?
1. Fever
2. Diarrhea
3. Complaints of nausea a

4. Crackles on auscultation of the lungs
Rationale:
Cardiotoxicity noted by abnormal electrocardiographic findings or cardiomyopathy manifested as congestive heart failure is a toxic effect of daunorubicin. Bone marrow depression is also a toxic effect. N

133.) A nurse is monitoring a client receiving desmopressin acetate (DDAVP) for adverse effects to the medication. Which of the following indicates the presence of an adverse effect?
1. Insomnia
2. Drowsiness
3. Weight loss
4. Increased urination

2. Drowsiness
Rationale:
Water intoxication (overhydration) or hyponatremia is an adverse effect to desmopressin. Early signs include drowsiness, listlessness, and headache. Decreased urination, rapid weight gain, confusion, seizures, and coma also may oc

134.) A nurse reinforces instructions to a client who is taking levothyroxine (Synthroid). The nurse tells the client to take the medication:
1. With food
2. At lunchtime
3. On an empty stomach
4. At bedtime with a snack

Rationale:
Oral doses of levothyroxine (Synthroid) should be taken on an empty stomach to enhance absorption. Dosing should be done in the morning before breakfast.
*
Note that options 1, 2, and 4 are comparable or alike in that these options address admi

135.) A nurse reinforces medication instructions to a client who is taking levothyroxine (Synthroid). The nurse instructs the client to notify the health care provider (HCP) if which of the following occurs?
1. Fatigue
2. Tremors
3. Cold intolerance
4. Ex

2. Tremors
Rationale:
Excessive doses of levothyroxine (Synthroid) can produce signs and symptoms of hyperthyroidism. These include tachycardia, chest pain, tremors, nervousness, insomnia, hyperthermia, heat intolerance, and sweating. The client should be

136.) A nurse performs an admission assessment on a client who visits a health care clinic for the first time. The client tells the nurse that propylthiouracil (PTU) is taken daily. The nurse continues to collect data from the client, suspecting that the

2. Graves' disease
Rationale:
PTU inhibits thyroid hormone synthesis and is used to treat hyperthyroidism, or Graves' disease. Myxedema indicates hypothyroidism.
Cushing's syndrome and Addison's disease are disorders related to adrenal function.

137.) A nurse is reinforcing instructions for a client regarding intranasal desmopressin acetate (DDAVP). The nurse tells the client that which of the following is a side effect of the medication?
1. Headache
2. Vulval pain
3. Runny nose
4. Flushed skin

3. Runny nose
Rationale:
Desmopressin administered by the intranasal route can cause a runny or stuffy nose. Headache, vulval pain, and flushed skin are side effects if the medication is administered by the intravenous (IV) route.

138.) A daily dose of prednisone is prescribed for a client. A nurse reinforces instructions to the client regarding administration of the medication and instructs the client that the best time to take this medication is:
1. At noon
2. At bedtime
3. Early

3. Early morning
Rationale:
Corticosteroids (glucocorticoids) should be administered before 9:00 AM. Administration at this time helps minimize adrenal insufficiency and mimics the burst of glucocorticoids released naturally by the adrenal glands each mor

139.) Prednisone is prescribed for a client with diabetes mellitus who is taking Humulin neutral protamine Hagedorn (NPH) insulin daily. Which of the following prescription changes does the nurse anticipate during therapy with the prednisone?
1. An additi

3. An increased amount of daily Humulin NPH insulin
Rationale:
Glucocorticoids can elevate blood glucose levels. Clients with diabetes mellitus may need their dosages of insulin or oral hypoglycemic medications increased during glucocorticoid therapy. The

140.) The client has a new prescription for metoclopramide (Reglan). On review of the chart, the nurse identifies that this medication can be safely administered with which condition?
1. Intestinal obstruction
2. Peptic ulcer with melena
3. Diverticulitis

4. Vomiting following cancer chemotherapy
Rationale:
Metoclopramide is a gastrointestinal (GI) stimulant and antiemetic. Because it is a GI stimulant, it is contraindicated with GI obstruction, hemorrhage, or perforation. It is used in the treatment of em

141.) The nurse has reinforced instructions to a client who has been prescribed cholestyramine (Questran). Which statement by the client indicates a need for further instructions?
1. "I will continue taking vitamin supplements."
2. "This medication will h

3. "This medication should only be taken with water."
Rationale:
Cholestyramine (Questran) is a bile acid sequestrant used to lower the cholesterol level, and client compliance is a problem because of its taste and palatability. The use of flavored produc

142.) A health care provider has written a prescription for ranitidine (Zantac), once daily. The nurse should schedule the medication for which of the following times?
1. At bedtime
2. After lunch
3. With supper
4. Before breakfast

1. At bedtime
Rationale:
A single daily dose of ranitidine is usually scheduled to be given at bedtime. This allows for a prolonged effect, and the greatest protection of the gastric mucosa.
*
recall that ranitidine suppresses secretions of gastric acids

143.) A client has just taken a dose of trimethobenzamide (Tigan). The nurse plans to monitor this client for relief of:
1. Heartburn
2. Constipation
3. Abdominal pain
4. Nausea and vomiting

4. Nausea and vomiting
Rationale:
Trimethobenzamide is an antiemetic agent used in the treatment of nausea and vomiting. The other options are incorrect.

144.) A client is taking docusate sodium (Colace). The nurse monitors which of the following to determine whether the client is having a therapeutic effect from this medication?
1. Abdominal pain
2. Reduction in steatorrhea
3. Hematest-negative stools
4.

4. Regular bowel movements
Rationale:
Docusate sodium is a stool softener that promotes the absorption of water into the stool, producing a softer consistency of stool. The intended effect is relief or prevention of constipation. The medication does not r

145.) A nurse has a prescription to give a client albuterol (Proventil HFA) (two puffs) and beclomethasone dipropionate (Qvar) (nasal inhalation, two puffs), by metered-dose inhaler. The nurse administers the medication by giving the:
1. Albuterol first a

1. Albuterol first and then the beclomethasone dipropionate
Rationale:
Albuterol is a bronchodilator. Beclomethasone dipropionate is a glucocorticoid. Bronchodilators are always administered before glucocorticoids when both are to be given on the same tim

146.) A client has begun therapy with theophylline (Theo-24). The nurse tells the client to limit the intake of which of the following while taking this medication?
1. Oranges and pineapple
2. Coffee, cola, and chocolate
3. Oysters, lobster, and shrimp
4.

2. Coffee, cola, and chocolate
Rationale:
Theophylline is a xanthine bronchodilator. The nurse teaches the client to limit the intake of xanthine-containing foods while taking this medication. These include coffee, cola, and chocolate.

147.) A client with a prescription to take theophylline (Theo-24) daily has been given medication instructions by the nurse. The nurse determines that the client needs further information about the medication if the client states that he or she will:
1. D

2. Take the daily dose at bedtime.
Rationale:
The client taking a single daily dose of theophylline, a xanthine bronchodilator, should take the medication early in the morning. This enables the client to have maximal benefit from the medication during day

148.) A client is taking cetirizine hydrochloride (Zyrtec). The nurse checks for which of the following side effects of this medication?
1. Diarrhea
2. Excitability
3. Drowsiness
4. Excess salivation

3. Drowsiness
Rationale:
A frequent side effect of cetirizine hydrochloride (Zyrtec), an antihistamine, is drowsiness or sedation. Others include blurred vision, hypertension (and sometimes hypotension), dry mouth, constipation, urinary retention, and swe

149.) A client taking fexofenadine (Allegra) is scheduled for allergy skin testing and tells the nurse in the health care provider's office that a dose was taken this morning. The nurse determines that:
1. The client should reschedule the appointment.
2.

1. The client should reschedule the appointment.
Rationale:
Fexofenadine is an antihistamine, which provides relief of symptoms caused by allergy. Antihistamines should be discontinued for at least 3 days (72 hours) before allergy skin testing to avoid fa

150.) A client complaining of not feeling well is seen in a clinic. The client is taking several medications for the control of heart disease and hypertension. These medications include a ?-blocker, digoxin (Lanoxin), and a diuretic. A tentative diagnosis

3. Double vision, loss of appetite, and nausea
Rationale:
Double vision, loss of appetite, and nausea are signs of digoxin toxicity. Additional signs of digoxin toxicity include bradycardia, difficulty reading, visual alterations such as green and yellow

151.) A client is being treated for acute congestive heart failure with intravenously administered bumetanide. The vital signs are as follows: blood pressure, 100/60 mm Hg; pulse, 96 beats/min; and respirations, 24 breaths/min. After the initial dose, whi

3. Monitoring blood pressure
Rationale:
Bumetanide is a loop diuretic. Hypotension is a common side effect associated with the use of this medication. The other options also require assessment but are not the priority.
*
priority ABCs�airway, breathing, a

152.) Intravenous heparin therapy is prescribed for a client. While implementing this prescription, a nurse ensures that which of the following medications is available on the nursing unit?
1. Protamine sulfate
2. Potassium chloride
3. Phytonadione (vitam

1. Protamine sulfate
Rationale:
The antidote to heparin is protamine sulfate; it should be readily available for use if excessive bleeding or hemorrhage occurs. Potassium chloride is administered for a potassium deficit. Vitamin K is an antidote for warfa

153.) A client is diagnosed with pulmonary embolism and is to be treated with streptokinase (Streptase). A nurse would report which priority data collection finding to the registered nurse before initiating this therapy?
1. Adventitious breath sounds
2. T

3. Blood pressure of 198/110 mm Hg
Rationale:
Thrombolytic therapy is contraindicated in a number of preexisting conditions in which there is a risk of uncontrolled bleeding, similar to the case in anticoagulant therapy. Thrombolytic therapy also is contr

154.) A nurse is reinforcing dietary instructions to a client who has been prescribed cyclosporine (Sandimmune). Which food item would the nurse instruct the client to avoid?
1. Red meats
2. Orange juice
3. Grapefruit juice
4. Green, leafy vegetables

3. Grapefruit juice
Rationale:
A compound present in grapefruit juice inhibits metabolism of cyclosporine. As a result, the consumption of grapefruit juice can raise cyclosporine levels by 50% to 100%, thereby greatly increasing the risk of toxicity. Grap

155.) Mycophenolate mofetil (CellCept) is prescribed for a client as prophylaxis for organ rejection following an allogeneic renal transplant. Which of the following instructions does the nurse reinforce regarding administration of this medication?
1. Adm

4. Contact the health care provider (HCP) if a sore throat occurs.
Rationale:
Mycophenolate mofetil should be administered on an empty stomach. The capsules should not be opened or crushed. The client should contact the HCP if unusual bleeding or bruising

156.) A nurse is reviewing the laboratory results for a client receiving tacrolimus (Prograf). Which laboratory result would indicate to the nurse that the client is experiencing an adverse effect of the medication?
1. Blood glucose of 200 mg/dL
2. Potass

1. Blood glucose of 200 mg/dL
Rationale:
A blood glucose level of 200 mg/dL is elevated above the normal range of 70 to 110 mg/dL and suggests an adverse effect. Other adverse effects include neurotoxicity evidenced by headache, tremor, insomnia; gastroin

157.) A client receiving nitrofurantoin (Macrodantin) calls the health care provider's office complaining of side effects related to the medication. Which side effect indicates the need to stop treatment with this medication?
1. Nausea
2. Diarrhea
3. Anor

4. Cough and chest pain
Rationale:
Gastrointestinal (GI) effects are the most frequent adverse reactions to this medication and can be minimized by administering the medication with milk or meals. Pulmonary reactions, manifested as dyspnea, chest pain, ch

158.) A client with chronic renal failure is receiving epoetin alfa (Epogen, Procrit). Which laboratory result would indicate a therapeutic effect of the medication?
1. Hematocrit of 32%
2. Platelet count of 400,000 cells/mm3
3. White blood cell count of

1. Hematocrit of 32%
Rationale:
Epoetin alfa is used to reverse anemia associated with chronic renal failure. A therapeutic effect is seen when the hematocrit is between 30% and 33%. The laboratory tests noted in the other options are unrelated to the use

159.) A nurse is caring for a client receiving morphine sulfate subcutaneously for pain. Because morphine sulfate has been prescribed for this client, which nursing action would be included in the plan of care?
1. Encourage fluid intake.
2. Monitor the cl

4. Encourage the client to cough and deep breathe.
Rationale:
Morphine sulfate suppresses the cough reflex. Clients need to be encouraged to cough and deep breathe to prevent pneumonia.
*
ABCs�airway, breathing, and circulation
n**

160.) Meperidine hydrochloride (Demerol) is prescribed for the client with pain. Which of the following would the nurse monitor for as a side effect of this medication?
1. Diarrhea
2. Bradycardia
3. Hypertension
4. Urinary retention

4. Urinary retention
Rationale:
Meperidine hydrochloride (Demerol) is an opioid analgesic. Side effects of this medication include respiratory depression, orthostatic hypotension, tachycardia, drowsiness and mental clouding, constipation, and urinary rete

161.) A nurse is caring for a client with severe back pain, and codeine sulfate has been prescribed for the client. Which of the following would the nurse include in the plan of care while the client is taking this medication?
1. Restrict fluid intake.
2.

2. Monitor bowel activity.
Rationale:
While the client is taking codeine sulfate, an opioid analgesic, the nurse would monitor vital signs and monitor for hypotension. The nurse should also increase fluid intake, palpate the bladder for urinary retention,

162.) Carbamazepine (Tegretol) is prescribed for a client with a diagnosis of psychomotor seizures. The nurse reviews the client's health history, knowing that this medication is contraindicated if which of the following disorders is present?
1. Headaches

2. Liver disease
Rationale:
Carbamazepine (Tegretol) is contraindicated in liver disease, and liver function tests are routinely prescribed for baseline purposes and are monitored during therapy. It is also contraindicated if the client has a history of b

163.) A client with trigeminal neuralgia tells the nurse that acetaminophen (Tylenol) is taken on a frequent daily basis for relief of generalized discomfort. The nurse reviews the client's laboratory results and determines that which of the following ind

4. A direct bilirubin level of 2 mg/dL
Rationale:
In adults, overdose of acetaminophen (Tylenol) causes liver damage. Option 4 is an indicator of liver function and is the only option that indicates an abnormal laboratory value. The normal direct bilirubi

164.) A client receives a prescription for methocarbamol (Robaxin), and the nurse reinforces instructions to the client regarding the medication. Which client statement would indicate a need for further instructions?
1. "My urine may turn brown or green.

3. "If my vision becomes blurred, I don't need to be concerned about it."
Rationale:
The client needs to be told that the urine may turn brown, black, or green. Other adverse effects include blurred vision, nasal congestion, urticaria, and rash. The clien

165.) The client has been on treatment for rheumatoid arthritis for 3 weeks. During the administration of etanercept (Enbrel), it is most important for the nurse to assess:
1. The injection site for itching and edema
2. The white blood cell counts and pla

2. The white blood cell counts and platelet counts
Rationale:
Infection and pancytopenia are adverse effects of etanercept (Enbrel). Laboratory studies are performed before and during treatment. The appearance of abnormal white blood cell counts and abnor

166.) Alendronate (Fosamax) is prescribed for a client with osteoporosis. The client taking this medication is instructed to:
1. Take the medication at bedtime.
2. Take the medication in the morning with breakfast.
3. Lie down for 30 minutes after taking

4. Take the medication with a full glass of water after rising in the morning.
Rationale:
Precautions need to be taken with administration of alendronate to prevent gastrointestinal side effects (especially esophageal irritation) and to increase absorptio

167.) A nurse prepares to reinforce instructions to a client who is taking allopurinol (Zyloprim). The nurse plans to include which of the following in the instructions?
1. Instruct the client to drink 3000 mL of fluid per day.
2. Instruct the client to t

1. Instruct the client to drink 3000 mL of fluid per day.
Rationale:
Allopurinol (Zyloprim) is an antigout medication used to decrease uric acid levels. Clients taking allopurinol are encouraged to drink 3000 mL of fluid a day. A full therapeutic effect m

168.) Colcrys (colchicine) is prescribed for a client with a diagnosis of gout. The nurse reviews the client's medical history in the health record, knowing that the medication would be contraindicated in which disorder?
1. Myxedema
2. Renal failure
3. Hy

2. Renal failure
Rationale:
Colchicine is contraindicated in clients with severe gastrointestinal, renal, hepatic or cardiac disorders, or with blood dyscrasias. Clients with impaired renal function may exhibit myopathy and neuropathy manifested as genera

169.) Insulin glargine (Lantus) is prescribed for a client with diabetes mellitus. The nurse tells the client that it is best to take the insulin:
1. 1 hour after each meal
2. Once daily, at the same time each day
3. 15 minutes before breakfast, lunch, an

2. Once daily, at the same time each day
Rationale:
Insulin glargine is a long-acting recombinant DNA human insulin used to treat type 1 and type 2 diabetes mellitus. It has a 24-hour duration of action and is administered once a day, at the same time eac

170.) Atenolol hydrochloride (Tenormin) is prescribed for a hospitalized client. The nurse should perform which of the following as a priority action before administering the medication?
1. Listen to the client's lung sounds.
2. Check the client's blood p

2. Check the client's blood pressure.
Rationale:
Atenolol hydrochloride is a beta-blocker used to treat hypertension. Therefore the priority nursing action before administration of the medication is to check the client's blood pressure. The nurse also che

171.) A nurse is preparing to administer furosemide (Lasix) to a client with a diagnosis of heart failure. The most important laboratory test result for the nurse to check before administering this medication is:
1. Potassium level
2. Creatinine level
3.

1. Potassium level
Rationale:
Furosemide is a loop diuretic. The medication causes a decrease in the client's electrolytes, especially potassium, sodium, and chloride. Administering furosemide to a client with low electrolyte levels could precipitate vent

172.) A nurse provides dietary instructions to a client who will be taking warfarin sodium (Coumadin). The nurse tells the client to avoid which food item?
1. Grapes
2. Spinach
3. Watermelon
4. Cottage cheese

2. Spinach
Rationale:
Warfarin sodium is an anticoagulant. Anticoagulant medications act by antagonizing the action of vitamin K, which is needed for clotting. When a client is taking an anticoagulant, foods high in vitamin K often are omitted from the di

173.) A nurse reviews the medication history of a client admitted to the hospital and notes that the client is taking leflunomide (Arava). During data collection, the nurse asks which question to determine medication effectiveness?
1. "Do you have any joi

1. "Do you have any joint pain?"
Rationale:
Leflunomide is an immunosuppressive agent and has an anti-inflammatory action. The medication provides symptomatic relief of rheumatoid arthritis. Diarrhea can occur as a side effect of the medication. The other

174.) A client with portosystemic encephalopathy is receiving oral lactulose (Chronulac) daily. The nurse assesses which of the following to determine medication effectiveness?
1. Lung sounds
2. Blood pressure
3. Blood ammonia level
4. Serum potassium lev

3. Blood ammonia level
Rationale:
Lactulose is a hyperosmotic laxative and ammonia detoxicant. It is used to prevent or treat portosystemic encephalopathy, including hepatic precoma and coma. It also is used to treat constipation. The medication retains a

175.) A nurse notes that a client is receiving lamivudine (Epivir). The nurse determines that this medication has been prescribed to treat which of the following?
1. Pancreatitis
2. Pharyngitis
3. Tonic-clonic seizures
4. Human immunodeficiency virus (HIV

4. Human immunodeficiency virus (HIV) infection
Rationale:
Lamivudine is a nucleoside reverse transcriptase inhibitor and antiviral medication. It slows HIV replication and reduces the progression of HIV infection. It also is used to treat chronic hepatit

176.) A nurse notes that a client is taking lansoprazole (Prevacid). On data collection, the nurse asks which question to determine medication effectiveness?
1. "Has your appetite increased?"
2. "Are you experiencing any heartburn?"
3. "Do you have any pr

2. "Are you experiencing any heartburn?"
Rationale:
Lansoprazole is a gastric acid pump inhibitor used to treat gastric and duodenal ulcers, erosive esophagitis, and hypersecretory conditions. It also is used to treat gastroesophageal reflux disease (GERD

177.) A nurse is assisting in caring for a pregnant client who is receiving intravenous magnesium sulfate for the management of preeclampsia and notes that the client's deep tendon reflexes are absent. On the basis of this data, the nurse reports the find

4. The client is experiencing magnesium toxicity.
Rationale:
Magnesium toxicity can occur as a result of magnesium sulfate therapy. Signs of magnesium sulfate toxicity relate to the central nervous system depressant effects of the medication and include r

178.) Methylergonovine (Methergine) is prescribed for a client with postpartum hemorrhage caused by uterine atony. Before administering the medication, the nurse checks which of the following as the important client parameter?
1. Temperature
2. Lochial fl

4. Blood pressure
Rationale:
Methylergonovine is an ergot alkaloid used for postpartum hemorrhage. It stimulates contraction of the uterus and causes arterial vasoconstriction. Ergot alkaloids are avoided in clients with significant cardiovascular disease

179.) A nurse provides medication instructions to a client who had a kidney transplant about therapy with cyclosporine (Sandimmune). Which statement by the client indicates a need for further instruction?
1. "I need to obtain a yearly influenza vaccine.

1. "I need to obtain a yearly influenza vaccine."
Rationale:
Cyclosporine is an immunosuppressant medication. Because of the medication's effects, the client should not receive any vaccinations without first consulting the HCP. The client should report de

180.) A health care provider (HCP) writes a prescription for digoxin (Lanoxin), 0.25 mg daily. The nurse teaches the client about the medication and tells the client that it is important to:
1. Count the radial and carotid pulses every morning.
2. Check t

4. Withhold the medication and call the HCP if the pulse is less than 60 beats per minute.
Rationale:
An important component of taking this medication is monitoring the pulse rate; however, it is not necessary for the client to take both the radial and ca

181.) A client is taking ticlopidine hydrochloride (Ticlid). The nurse tells the client to avoid which of the following while taking this medication?
1. Vitamin C
2. Vitamin D
3. Acetaminophen (Tylenol)
4. Acetylsalicylic acid (aspirin)

4. Acetylsalicylic acid (aspirin)
Rationale:
Ticlopidine hydrochloride is a platelet aggregation inhibitor. It is used to decrease the risk of thrombotic strokes in clients with precursor symptoms. Because it is an antiplatelet agent, other medications th

182.) A client with angina pectoris is experiencing chest pain that radiates down the left arm. The nurse administers a sublingual nitroglycerin tablet to the client. The client's pain is unrelieved, and the nurse determines that the client needs another

Rationale:
Nitroglycerin acts directly on the smooth muscle of the blood vessels, causing relaxation and dilation. As a result, hypotension can occur. The nurse would check the client's blood pressure before administering the second nitroglycerin tablet.

183.) A client who received a kidney transplant is taking azathioprine (Imuran), and the nurse provides instructions about the medication. Which statement by the client indicates a need for further instructions?
1. "I need to watch for signs of infection.

2. "I need to discontinue the medication after 14 days of use."
Rationale:
Azathioprine is an immunosuppressant medication that is taken for life. Because of the effects of the medication, the client must watch for signs of infection, which are reported i

184.) A nurse preparing a client for surgery reviews the client's medication record. The client is to be nothing per mouth (NPO) after midnight. Which of the following medications, if noted on the client's record, should the nurse question?
1. Cyclobenzap

4. Prednisone
Rationale:
Prednisone is a corticosteroid that can cause adrenal atrophy, which reduces the body's ability to withstand stress. Before and during surgery, dosages may be temporarily increased. Cyclobenzaprine is a skeletal muscle relaxant. A

185.) Which of the following herbal therapies would be prescribed for its use as an antispasmodic? Select all that apply.
1.Aloe
2.Kava
3.Ginger
4.Chamomile
5.Peppermint oil

4.Chamomile
5.Peppermint oil
Rationale:
Chamomile has a mild sedative effect and acts as an antispasmodic and anti-inflammatory. Peppermint oil acts as an antispasmodic and is used for irritable bowel syndrome. Topical aloe promotes wound healing. Aloe ta

186.) A nurse prepares to administer sodium polystyrene sulfonate (Kayexalate) to a client. Before administering the medication, the nurse reviews the action of the medication and understands that it:
1. Releases bicarbonate in exchange for primarily sodi

2. Releases sodium ions in exchange for primarily potassium ions
Rationale:
Sodium polystyrene sulfonate is a cation exchange resin used in the treatment of hyperkalemia. The resin either passes through the intestine or is retained in the colon. It releas

187.) A clinic nurse prepares to administer an MMR (measles, mumps, rubella) vaccine to a child. How is this vaccine best administered?
1. Intramuscularly in the deltoid muscle
2. Subcutaneously in the gluteal muscle
3. Subcutaneously in the outer aspect

3. Subcutaneously in the outer aspect of the upper arm
Rationale:
The MMR vaccine is administered subcutaneously in the outer aspect of the upper arm. The gluteal muscle is most often used for intramuscular injections. The MMR vaccine is not administered

188.) The nurse should anticipate that the most likely medication to be prescribed prophylactically for a child with spina bifida (myelomeningocele) who has a neurogenic bladder would be:
1. Prednisone
2. Sulfisoxazole
3. Furosemide (Lasix)
4. Intravenous

2. Sulfisoxazole
Rationale:
A neurogenic bladder prevents the bladder from completely emptying because of the decrease in muscle tone. The most likely medication to be prescribed to prevent urinary tract infection would be an antibiotic. A common prescrib

189.) Prostaglandin E1 is prescribed for a child with transposition of the great arteries. The mother of the child asks the nurse why the child needs the medication. The nurse tells the mother that the medication:
1. Prevents hypercyanotic (blue or tet) s

4. Provides adequate oxygen saturation and maintains cardiac output
Rationale:
A child with transposition of the great arteries may receive prostaglandin E1 temporarily to increase blood mixing if systemic and pulmonary mixing are inadequate to maintain a

190.) A child is hospitalized with a diagnosis of lead poisoning. The nurse assisting in caring for the child would prepare to assist in administering which of the following medications?
1. Activated charcoal
2. Sodium bicarbonate
3. Syrup of ipecac syrup

4. Dimercaprol (BAL in Oil)
Rationale:
Dimercaprol is a chelating agent that is administered to remove lead from the circulating blood and from some tissues and organs for excretion in the urine. Sodium bicarbonate may be used in salicylate poisoning. Syr

191.) A child is brought to the emergency department for treatment of an acute asthma attack. The nurse prepares to administer which of the following medications first?
1. Oral corticosteroids
2. A leukotriene modifier
3. A ?2 agonist
4. A nonsteroidal an

3. A ?2 agonist
Rationale:
In treating an acute asthma attack, a short acting ?2 agonist such as albuterol (Proventil HFA) will be given to produce bronchodilation. Options 1, 2, and 4 are long-term control (preventive) medications.

192.) A nurse is collecting medication information from a client, and the client states that she is taking garlic as an herbal supplement. The nurse understands that the client is most likely treating which of the following conditions?
1. Eczema
2. Insomn

4. Hyperlipidemia
Rationale:
Garlic is an herbal supplement that is used to treat hyperlipidemia and hypertension. An herbal supplement that may be used to treat eczema is evening primrose. Insomnia has been treated with both valerian root and chamomile.

193.) Sodium hypochlorite (Dakin's solution) is prescribed for a client with a leg wound containing purulent drainage. The nurse is assisting in developing a plan of care for the client and includes which of the following in the plan?
1. Ensure that the s

1. Ensure that the solution is freshly prepared before use.
Rationale:
Dakin solution is a chloride solution that is used for irrigating and cleaning necrotic or purulent wounds. It can be used for packing necrotic wounds. It cannot be used to pack purule

194.) A nurse provides instructions to a client regarding the use of tretinoin (Retin-A). Which statement by the client indicates the need for further instructions?
1. "Optimal results will be seen after 6 weeks."
2. "I should apply a very thin layer to m

2. "I should apply a very thin layer to my skin."
Rationale:
Tretinoin is applied liberally to the skin. The hands are washed thoroughly immediately after applying. Therapeutic results should be seen after 2 to 3 weeks but may not be optimal until after 6

195.) A nurse is caring for a client who is taking metoprolol (Lopressor). The nurse measures the client's blood pressure (BP) and apical pulse (AP) immediately before administration. The client's BP is 122/78 mm/Hg and the AP is 58 beats/min. Based on th

1. Withhold the medication.
Rationale:
Metoprolol (Lopressor) is classified as a beta-adrenergic blocker and is used in the treatment of hypertension, angina, and myocardial infarction. Baseline nursing assessments include measurement of BP and AP immedia

196.) A client has been prescribed amikacin (Amikin). Which of the following priority baseline functions should be monitored?
1. Apical pulse
2. Liver function
3. Blood pressure
4. Hearing acuity

4. Hearing acuity
Rationale:
Amikacin (Amikin) is an antibiotic. This medication can cause ototoxicity and nephrotoxicity; therefore, hearing acuity tests and kidney function studies should be performed before the initiation of therapy. Apical pulse, live

197.) Collagenase (Santyl) is prescribed for a client with a severe burn to the hand. The nurse provides instructions to the client regarding the use of the medication. Which statement by the client indicates an accurate understanding of the use of this m

3. "I will apply the ointment once a day and cover it with a sterile dressing."
Rationale:
Collagenase is used to promote debridement of dermal lesions and severe burns. It is usually applied once daily and covered with a sterile dressing.

198.) Coal tar has been prescribed for a client with a diagnosis of psoriasis, and the nurse provides instructions to the client about the medication. Which statement by the client indicates a need for further instructions?
1. "The medication can cause ph

4. "The medication can cause systemic effects."
Rationale:
Coal tar is used to treat psoriasis and other chronic disorders of the skin. It suppresses DNA synthesis, mitotic activity, and cell proliferation. It has an unpleasant odor, can frequently stain

199.) A nurse is applying a topical glucocorticoid to a client with eczema. The nurse monitors for systemic absorption of the medication if the medication is being applied to which of the following body areas?
1. Back
2. Axilla
3. Soles of the feet
4. Pal

2. Axilla
Rationale:
Topical glucocorticoids can be absorbed into the systemic circulation. Absorption is higher from regions where the skin is especially permeable (scalp, axillae, face, eyelids, neck, perineum, genitalia), and lower from regions where p

200.) A client is seen in the clinic for complaints of skin itchiness that has been persistent over the past several weeks. Following data collection, it has been determined that the client has scabies. Lindane is prescribed, and the nurse is asked to pro

4. Leave the cream on for 8 to 12 hours and then remove by washing.
Rationale:
Lindane is applied in a thin layer to the entire body below the head. No more than 30 g (1 oz) should be used. The medication is removed by washing 8 to 12 hours later. Usually

201.) A nurse is preparing to administer eardrops to an infant. The nurse plans to:
1. Pull up and back on the ear and direct the solution onto the eardrum.
2. Pull down and back on the ear and direct the solution onto the eardrum.
3. Pull down and back o

3. Pull down and back on the ear and direct the solution toward the wall of the canal.
Rationale:
When administering eardrops to an infant, the nurse pulls the ear down and straight back. In the adult or a child older than 3 years, the ear is pulled up an

202.) A nurse is collecting data from a client about medications being taken, and the client tells the nurse that he is taking herbal supplements for the treatment of varicose veins. The nurse understands that the client is most likely taking which of the

1. Bilberry
Rationale:
Bilberry is an herbal supplement that has been used to treat varicose veins. This supplement has also been used to treat cataracts, retinopathy, diabetes mellitus, and peripheral vascular disease. Ginseng has been used to improve me

203.) A nurse is preparing to give the postcraniotomy client medication for incisional pain. The family asks the nurse why the client is receiving codeine sulfate and not "something stronger." In formulating a response, the nurse incorporates the understa

4. Does not alter respirations or mask neurological signs as do other opioids
Rationale:
Codeine sulfate is the opioid analgesic often used for clients after craniotomy. It is frequently combined with a nonopioid analgesic such as acetaminophen for added

204.) A client receives a dose of edrophonium (Enlon). The client shows improvement in muscle strength for a period of time following the injection. The nurse interprets that this finding is compatible with:
1. Multiple sclerosis
2. Myasthenia gravis
3. M

2. Myasthenia gravis
Rationale:
Myasthenia gravis can often be diagnosed based on clinical signs and symptoms. The diagnosis can be confirmed by injecting the client with a dose of edrophonium . This medication inhibits the breakdown of an enzyme in the n

205.) A nurse is assisting in preparing to administer acetylcysteine (Mucomyst) to a client with an overdose of acetaminophen (Tylenol). The nurse prepares to administer the medication by:
1. Administering the medication subcutaneously in the deltoid musc

4. Mixing the medication in a flavored ice drink and allowing the client to drink the medication through a straw
Rationale:
Because acetylcysteine has a pervasive odor of rotten eggs, it must be disguised in a flavored ice drink. It is consumed preferably

206.) A client is receiving baclofen (Lioresal) for muscle spasms caused by a spinal cord injury. The nurse monitors the client, knowing that which of the following is a side effect of this medication?
1. Muscle pain
2. Hypertension
3. Slurred speech
4. P

Rationale:
Side effects of baclofen include drowsiness, dizziness, weakness, and nausea. Occasional side effects include headache, paresthesia of the hands and feet, constipation or diarrhea, anorexia, hypotension, confusion, and nasal congestion. Paradox

207.) A client is suspected of having myasthenia gravis, and the health care provider administers edrophonium (Enlon) to determine the diagnosis. After administration of this medication, which of the following would indicate the presence of myasthenia gra

3. An increase in muscle strength
Rationale:
Edrophonium is a short-acting acetylcholinesterase inhibitor used as a diagnostic agent. When a client with suspected myasthenia gravis is given the medication intravenously, an increase in muscle strength woul

208.) A client with myasthenia gravis verbalizes complaints of feeling much weaker than normal. The health care provider plans to implement a diagnostic test to determine if the client is experiencing a myasthenic crisis and administers edrophonium (Enlon

auto-define "A client with myasthen..."
Rationale:
Edrophonium (Enlon) is administered to determine whether the client is reacting to an overdose of a medication (cholinergic crisis) or to an increasing severity of the disease (myasthenic crisis). When th

209.) A client with multiple sclerosis is receiving diazepam (Valium), a centrally acting skeletal muscle relaxant. Which of the following would indicate that the client is experiencing a side effect related to this medication?
1. Headache
2. Drowsiness
3

2. Drowsiness
Rationale:
Incoordination and drowsiness are common side effects resulting from this medication. Options 1, 3, and 4 are incorrect.

210.) Dantrolene (Dantrium) is prescribed for a client with a spinal cord injury for discomfort resulting from spasticity. The nurse tells the client about the importance of follow-up and the need for which blood study?
1. Creatinine level
2. Sedimentatio

3. Liver function studies
Rationale:
Dantrolene can cause liver damage, and the nurse should monitor liver function studies. Baseline liver function studies are done before therapy starts, and regular liver function studies are performed throughout therap

211.) A client with epilepsy is taking the prescribed dose of phenytoin (Dilantin) to control seizures. A phenytoin blood level is drawn, and the results reveal a level of 35 mcg/ml. Which of the following symptoms would be expected as a result of this la

3. Slurred speech
Rationale:
The therapeutic phenytoin level is 10 to 20 mcg/mL. At a level higher than 20 mcg/mL, involuntary movements of the eyeballs (nystagmus) appear. At a level higher than 30 mcg/mL, ataxia and slurred speech occur.

212.) Mannitol (Osmitrol) is being administered to a client with increased intracranial pressure following a head injury. The nurse assisting in caring for the client knows that which of the following indicates the therapeutic action of this medication?
1

4. Induces diuresis by raising the osmotic pressure of glomerular filtrate, thereby inhibiting tubular reabsorption of water and solutes
Rationale:
Mannitol is an osmotic diuretic that induces diuresis by raising the osmotic pressure of glomerular filtrat

213.) A client is admitted to the hospital with complaints of back spasms. The client states, "I have been taking two or three aspirin every 4 hours for the past week and it hasn't helped my back." Aspirin intoxication is suspected. Which of the following

1. Tinnitus
Rationale:
Mild intoxication with acetylsalicylic acid (aspirin) is called salicylism and is commonly experienced when the daily dosage is higher than 4 g. Tinnitus (ringing in the ears) is the most frequently occurring effect noted with intox

214.) A health care provider initiates carbidopa/levodopa (Sinemet) therapy for the client with Parkinson's disease. A few days after the client starts the medication, the client complains of nausea and vomiting. The nurse tells the client that:
1. Taking

2. Taking the medication with food will help to prevent the nausea.
Rationale:
If carbidopa/levodopa is causing nausea and vomiting, the nurse would tell the client that taking the medication with food will prevent the nausea. Additionally, the client sho

215.) A client with rheumatoid arthritis is taking acetylsalicylic acid (aspirin) on a daily basis. Which medication dose should the nurse expect the client to be taking?
1. 1 g daily
2. 4 g daily
3. 325 mg daily
4. 1000 mg daily

2. 4 g daily
Rationale:
Aspirin may be used to treat the client with rheumatoid arthritis. It may also be used to reduce the risk of recurrent transient ischemic attack (TIA) or brain attack (stroke) or reduce the risk of myocardial infarction (MI) in cli

216.) A nurse is caring for a client with gout who is taking Colcrys (colchicine). The client has been instructed to restrict the diet to low-purine foods. Which of the following foods should the nurse instruct the client to avoid while taking this medica

2. Scallops
Rationale:
Colchicine is a medication used for clients with gout to inhibit the reabsorption of uric acid by the kidney and promote excretion of uric acid in the urine. Uric acid is produced when purine is catabolized. Clients are instructed t

217.) A health care provider prescribes auranofin (Ridaura) for a client with rheumatoid arthritis. Which of the following would indicate to the nurse that the client is experiencing toxicity related to the medication?
1. Joint pain
2. Constipation
3. Rin

4. Complaints of a metallic taste in the mouth
Rationale:
Ridaura is the one gold preparation that is given orally rather than by injection. Gastrointestinal reactions including diarrhea, abdominal pain, nausea, and loss of appetite are common early in th

218.) A film-coated form of diflunisal has been prescribed for a client for the treatment of chronic rheumatoid arthritis. The client calls the clinic nurse because of difficulty swallowing the tablets. Which initial instruction should the nurse provide t

4. "Swallow the tablets with large amounts of water or milk."
Rationale:
Diflunisal may be given with water, milk, or meals. The tablets should not be crushed or broken open. Taking the medication with a large amount of water or milk should be tried befor

219.) A health care provider instructs a client with rheumatoid arthritis to take ibuprofen (Motrin). The nurse reinforces the instructions, knowing that the normal adult dose for this client is which of the following?
1. 100 mg orally twice a day
2. 200

3. 400 mg orally three times a day
Rationale:
For acute or chronic rheumatoid arthritis or osteoarthritis, the normal oral adult dose is 400 to 800 mg three or four times daily.

220.) A adult client with muscle spasms is taking an oral maintenance dose of baclofen (Lioresal). The nurse reviews the medication record, expecting that which dose should be prescribed?
1. 15 mg four times a day
2. 25 mg four times a day
3. 30 mg four t

1. 15 mg four times a day
Rationale:
Baclofen is dispensed in 10- and 20-mg tablets for oral use. Dosages are low initially and then gradually increased. Maintenance doses range from 15 to 20 mg administered three or four times a day.

221.) A nurse is reviewing the health care provider's prescriptions for an adult client who has been admitted to the hospital following a back injury. Carisoprodol (Soma) is prescribed for the client to relieve the muscle spasms; the health care provider

1. The normal adult dosage
Rationale:
The normal adult dosage for carisoprodol is 350 mg orally three or four times daily.

222.) A nurse has administered a dose of diazepam (Valium) to a client. The nurse would take which important action before leaving the client's room?
1. Giving the client a bedpan
2. Drawing the shades or blinds closed
3. Turning down the volume on the te

4. Per agency policy, putting up the side rails on the bed
Rationale:
Diazepam is a sedative-hypnotic with anticonvulsant and skeletal muscle relaxant properties. The nurse should institute safety measures before leaving the client's room to ensure that t

223.) A client with a psychotic disorder is being treated with haloperidol (Haldol). Which of the following would indicate the presence of a toxic effect of this medication?
1. Nausea
2. Hypotension
3. Blurred vision
4. Excessive salivation

4. Excessive salivation
Rationale:
Toxic effects include extrapyramidal symptoms (EPS) noted as marked drowsiness and lethargy, excessive salivation, and a fixed stare. Akathisia, acute dystonias, and tardive dyskinesia are also signs of toxicity. Hypoten

224.) Neuroleptic malignant syndrome is suspected in a client who is taking chlorpromazine. Which medication would the nurse prepare in anticipation of being prescribed to treat this adverse effect related to the use of chlorpromazine?
1. Protamine sulfat

2. Bromocriptine (Parlodel)
Rationale:
Bromocriptine is an antiparkinsonian prolactin inhibitor used in the treatment of neuroleptic malignant syndrome. Vitamin K is the antidote for warfarin (Coumadin) overdose. Protamine sulfate is the antidote for hepa

225.) A nursing student is assigned to care for a client with a diagnosis of schizophrenia. Haloperidol (Haldol) is prescribed for the client, and the nursing instructor asks the student to describe the action of the medication. Which statement by the nur

4. It blocks the binding of dopamine to the postsynaptic dopamine receptors in the brain.
Rationale:
Haloperidol acts by blocking the binding of dopamine to the postsynaptic dopamine receptors in the brain. Imipramine hydrochloride (Tofranil) blocks the r

226.) A client receiving lithium carbonate (Lithobid) complains of loose, watery stools and difficulty walking. The nurse would expect the serum lithium level to be which of the following?
1. 0.7 mEq/L
2. 1.0 mEq/L
3. 1.2 mEq/L
4. 1.7 mEq/L

4. 1.7 mEq/L
Rationale:
The therapeutic serum level of lithium ranges from 0.6 to 1.2 mEq/L. Serum lithium levels above the therapeutic level will produce signs of toxicity.

227.) When teaching a client who is being started on imipramine hydrochloride (Tofranil), the nurse would inform the client that the desired effects of the medication may:
1. Start during the first week of administration
2. Not occur for 2 to 3 weeks of a

2. Not occur for 2 to 3 weeks of administration
Rationale:
The therapeutic effects of administration of imipramine hydrochloride may not occur for 2 to 3 weeks after the antidepressant therapy has been initiated. Therefore options 1, 3, and 4 are incorrec

228.) A client receiving an anxiolytic medication complains that he feels very "faint" when he tries to get out of bed in the morning. The nurse recognizes this complaint as a symptom of:
1. Cardiac dysrhythmias
2. Postural hypotension
3. Psychosomatic sy

2. Postural hypotension
Rationale:
Anxiolytic medications can cause postural hypotension. The client needs to be taught to rise to a sitting position and get out of bed slowly because of this adverse effect related to the medication. Options 1, 3, and 4 a

229.) A client who is taking lithium carbonate (Lithobid) is scheduled for surgery. The nurse informs the client that:
1. The medication will be discontinued a week before the surgery and resumed 1 week postoperatively.
2. The medication is to be taken un

3. The medication will be discontinued 1 to 2 days before the surgery and resumed as soon as full oral intake is allowed.
Rationale:
The client who is on lithium carbonate must be off the medication for 1 to 2 days before a scheduled surgical procedure an

230.) A client is placed on chloral hydrate (Somnote) for short-term treatment. Which nursing action indicates an understanding of the major side effect of this medication?
1. Monitoring neurological signs every 2 hours
2. Monitoring the blood pressure ev

3. Instructing the client to call for ambulation assistance
Rationale:
Chloral hydrate (a sedative-hypnotic) causes sedation and impairment of motor coordination; therefore, safety measures need to be implemented. The client is instructed to call for assi

231.) A client admitted to the hospital gives the nurse a bottle of clomipramine (Anafranil). The nurse notes that the medication has not been taken by the client in 2 months. What behaviors observed in the client would validate noncompliance with this me

4. Frequent handwashing with hot, soapy water
Rationale:
Clomipramine is commonly used in the treatment of obsessive-compulsive disorder. Handwashing is a common obsessive-compulsive behavior. Weight gain is a common side effect of this medication. Tachyc

232.) A client in the mental health unit is administered haloperidol (Haldol). The nurse would check which of the following to determine medication effectiveness?
1. The client's vital signs
2. The client's nutritional intake
3. The physical safety of oth

4. The client's orientation and delusional status
Rationale:
Haloperidol is used to treat clients exhibiting psychotic features. Therefore, to determine medication effectiveness, the nurse would check the client's orientation and delusional status. Vital

233.) Diphenhydramine hydrochloride (Benadryl) is used in the treatment of allergic rhinitis for a hospitalized client with a chronic psychotic disorder. The client asks the nurse why the medication is being discontinued before hospital discharge. The nur

3. Addictive properties are enhanced in the presence of psychotropic medications.
Rationale:
The addictive properties of diphenhydramine hydrochloride are enhanced when used with psychotropic medications. Allergic symptoms may not be short term and will o

234.) A hospitalized client is started on phenelzine sulfate (Nardil) for the treatment of depression. At lunchtime, a tray is delivered to the client. Which food item on the tray will the nurse remove?
1. Yogurt
2. Crackers
3. Tossed salad
4. Oatmeal coo

1. Yogurt
Rationale:
Phenelzine sulfate is a monoamine oxidase inhibitor (MAOI). The client should avoid taking in foods that are high in tyramine. These foods could trigger a potentially fatal hypertensive crisis. Foods to avoid include yogurt, aged chee

235.) A tricyclic antidepressant is administered to a client daily. The nurse plans to monitor for the common side effects of the medication and includes which of the following in the plan of care?
1. Offer hard candy or gum periodically.
2. Offer a nutri

1. Offer hard candy or gum periodically.
Rationale:
Dry mouth is a common side effect of tricyclic antidepressants. Frequent mouth rinsing with water, sucking on hard candy, and chewing gum will alleviate this common side effect. It is not necessary to mo

236.) A client is being treated for depression with amitriptyline hydrochloride. During the initial phases of treatment, the most important nursing intervention is:
1. Prescribing the client a tyramine-free diet
2. Checking the client for anticholinergic

4. Getting baseline postural blood pressures before administering the medication and each time the medication is administered
Rationale:
Amitriptyline hydrochloride is a tricyclic antidepressant often used to treat depression. It causes orthostatic change

237.) A client who is on lithium carbonate (Lithobid) will be discharged at the end of the week. In formulating a discharge teaching plan, the nurse will instruct the client that it is most important to:
1. Avoid soy sauce, wine, and aged cheese.
2. Have

4. Check with the psychiatrist before using any over-the-counter (OTC) medications or prescription medications.
Rationale:
Lithium is the medication of choice to treat manic-depressive illness. Many OTC medications interact with lithium, and the client is

238.) Ribavirin (Virazole) is prescribed for the hospitalized child with respiratory syncytial virus (RSV). The nurse prepares to administer this medication via which of the following routes?
1. Orally
2. Via face mask
3. Intravenously
4. Intramuscularly

2. Via face mask
Rationale:
Ribavirin is an antiviral respiratory medication used mainly in hospitalized children with severe RSV and in high-risk children. Administration is via hood, face mask, or oxygen tent. The medication is most effective if adminis

239.) Which of the following precautions will the nurse specifically take during the administration of ribavirin (Virazole) to a child with respiratory syncytial virus (RSV)?
1. Wearing goggles
2. Wearing a gown
3. Wearing a gown and a mask
4. Handwashing

1. Wearing goggles
Rationale:
Some caregivers experience headaches, burning nasal passages and eyes, and crystallization of soft contact lenses as a result of administration of ribavirin. Specific to this medication is the use of goggles. A gown is not ne

240.) A client with Parkinson's disease has been prescribed benztropine (Cogentin). The nurse monitors for which gastrointestinal (GI) side effect of this medication?
1. Diarrhea
2. Dry mouth
3. Increased appetite
4. Hyperactive bowel sounds

2. Dry mouth
Rationale:
Common GI side effects of benztropine therapy include constipation and dry mouth. Other GI side effects include nausea and ileus. These effects are the result of the anticholinergic properties of the medication.
*
Eliminate options

241.) A client with a history of simple partial seizures is taking clorazepate (Tranxene), and asks the nurse if there is a risk of addiction. The nurse's response is based on the understanding that clorazepate:
1. Is not habit forming, either physically

3. Leads to physical and psychological dependence with prolonged high-dose therapy
Rationale:
Clorazepate is classified as an anticonvulsant, antianxiety agent, and sedative-hypnotic (benzodiazepine). One of the concerns with clorazepate therapy is that t

242.) A client who was started on anticonvulsant therapy with clonazepam (Klonopin) tells the nurse of increasing clumsiness and unsteadiness since starting the medication. The client is visibly upset by these manifestations and asks the nurse what to do.

4. Are worse during initial therapy and decrease or disappear with long-term use
Rationale:
Drowsiness, unsteadiness, and clumsiness are expected effects of the medication during early therapy. They are dose related and usually diminish or disappear altog

243.) A hospitalized client is having the dosage of clonazepam (Klonopin) adjusted. The nurse should plan to:
1. Weigh the client daily.
2. Observe for ecchymosis.
3. Institute seizure precautions.
4. Monitor blood glucose levels.

3. Institute seizure precautions.
Rationale:
Clonazepam is a benzodiazepine used as an anticonvulsant. During initial therapy and during periods of dosage adjustment, the nurse should initiate seizure precautions for the client. Options 1, 2, and 4 are no

244.) A client has a prescription for valproic acid (Depakene) orally once daily. The nurse plans to:
1. Administer the medication with an antacid.
2. Administer the medication with a carbonated beverage.
3. Ensure that the medication is administered at t

3. Ensure that the medication is administered at the same time each day.
Rationale:
Valproic acid is an anticonvulsant, antimanic, and antimigraine medication. It may be administered with or without food. It should not be taken with an antacid or carbonat

245.) A client taking carbamazepine (Tegretol) asks the nurse what to do if he misses one dose. The nurse responds that the carbamazepine should be:
1. Withheld until the next scheduled dose
2. Withheld and the health care provider is notified immediately

3. Taken as long as it is not immediately before the next dose
Rationale:
Carbamazepine is an anticonvulsant that should be taken around the clock, precisely as directed. If a dose is omitted, the client should take the dose as soon as it is remembered, a

Becouse of the high incidence of sleep disorders, the most frequently presribed drugs are:

Sedative/Hypnotics

Which drugs are frequently prescribed to control seizures

Long acting barbituates

Pt returns from surgery with spinal anethesia, to decrease the possibility of spinal headache you should:

Increase fluids
Caffeine
Position on flat bed

The reason for the clients position after spinal anethesia would be

To decrease leakage of spinal fluids

Postoperative pt requests sleeping pill, which of the following would be used for pt that is having difficulty falling asleep, and nonpharmacologic alt have not been effective?

secoarbital- short term drug used for insomnia- causes pt to awake early.

Lidocaine is frequently used for: what type anesthesia

Local anesthesia

Primary ingredient in OTC drugs

Anti-histamines

which of the following drugs are considered safer than barbiturates in the elderly?

Estazolam(ProSom)
Temazepam(restoril)
Triazolam(Halcion)
All of which are sleep agents hypnotic/sedatives not barbiturates.

Advantage of balanced anesthesia

Reduction of drugs to maintain desired state of anesthesia

Local anesthesia indicated for which procedures

Dental
Diagnostic
Suturing lacerations

Balanced Anesthesia comprised of

Inhaled Gas
Premedicated 1Hr prior Narcotic Analgesic/Anticholinergic
Hypnotic night prior
Muscle relaxant

complications result from spinal anesthesia

Respiratory Distress
Headache

Rn needs to be aware Client returning from surgery recieved spinal anesthesia needs to be kept in which position for few hours (4)

Flat Bed

Pt has recieved local anesthesia in throat for bronch, Rn should intervene when

CNA offers pt glass of water

The LPN student observing a surgical procedure, which stage is pt most at risk:

Paralysis- Medulla regions responsible for- respiratory and cardiac effects.

Pt w/ known cardiovascular disease is to recieve local anesthesia for large laceration, which will need to be closely monitored

palpatations

Pt to recieve nitrous oxide, which adv rxn is expected

agitation

Succinycholine(Anectine) neuromuscular block agent,given during surgery expected response would be

relaxation muscles- paralysis

Soon after admin of neuromuscular agent anesthesia intibates pt what is the purpose

To counteract the paralysis of the diaphragm the result of the meds, to ensure breathing for pt.

Thiopental an iv anesthetic barbiturate administed, intended side effect would be

Rapid, Loss unconciousness

a pt with toothache is unable to make it to dentist, wishes to utilize natural alternative. Pt most likely to benefit from

Oil Cloves-
Common for Dentistry
natural analgesic/Antiseptic

Another name for local anesthesia is

Sodium Channel Blocker

Client recieved spinal anesthesia, what is most important for RN to monitor

HTN,Headaches

Client recieve conscious sedation, which drugs should the Rn expect to administer

Propofol(diprivan) Promotes natural sleep
Midazolam(Versed) Promotes Sedative rxn.
Some type combo of pain med and sedative

It is important for RN to teach client when taking secoarbital(seconal)

Short acting treats insomnia- causes early awakening

client taking lorazepam(ativan) asks how drug works, proper response would be:

increases action of Inhibitory NT Gamma-amniobutyric-Acid GABA to receptors

client taking zolpidem(ambien) for insomnia- RN prepares a care plan that includes monitoring pt with S/E
Which is side effect?

Headache

Rn teaching client about zolpidem, what is important for Rn to teach

May lead to pyschological dependance

Client taking triazolam(halcion) What are some important instructions for Rn to discuss

Avoid alcohol,smoking prevent rebound insomnia

Pt has been recieving Benzodiazepines for several yrs, what condition may this cause

W/D s/s if pt abruptly d/c

which form is most rapidly absorbed in body

*Suspension,
extended release
others ( IV,SbL IM)
Oral Rectal slow

Enteric Coated tablets absorbed where

Intestines

Usually food has what affect on drug dissolution/absorbtion

Interferes counteracts

sequence of the 4 processes of pharmocokinetocs

Absorb,Distribute,Metab,Excrete

Drugs that pass rapid the GI include:

Lipid soluble Non ionized

Factors that most commonly affect drug actions are

Poor circulation pain stress hunger fasting

Pt takes drug highly protein bound, several days pass pt takes 2nd drug that is 90% protien bound, what happens to first drug?

More of the first drug is released from the protein and becomes more pharmacologically active.

major site for drug metabolism

Liver

Route for drug absorption that has the greatest bioavailability

IV- fastest

Serum half life of drug is the time required

after absorption for 1/2 the drug to be eliminated

drugs with 1/2 life 24-30hrs would probably be adminsiterd on a dose schedule of

once daily

Elimination thru kidneys a drug must

be water soluble to pass thru.

Pt has renal disorder creatinine clearance 40ml/min Her drug dose should be

decreased
Normal creatinine is 0.6-1.2

Biological activiy of a drug is determined by the

fit of the drug at the receptor site
must fit or be recognized for drug to become active.

Drugs that prevent or inhibit a response are known as

antagonist

Receptor located in different parts of the body may initiate a variety of responses dependant upon its anatomic site the recptor is:

nonspecific

valid indicator that measures the margin of safety of a drug is its

therapeutic index
TD50/ED50

Drugs that narrow therapeutic ranges such as digoxin require plasma/serum levels to be monitored to avoid drug toxicity

Periodic interval

After drug administration, The highest plasma/serum concentrations of a drug at spec time is called:

Peak Level

Before admin of medication, Rn should check a drug reference to obtain all EXCEPT

Maximum efficacy
Therefore all of the following should be examined
Protien bind effect
Half life
theuraputic ranges

physiological affects NOT related to DESIRED effects (predicatable or associated with the use of a drug) is called:

Side effects

When an immediate drug response is desired a large initial dose is given to rapidly achieve an MEC in the plasma this is called:

Loading Dose

A time response curve that evaluates three parameters of drug action, does NOT include:

Theraputic Range
Includes:
Duration
Onset
Peak
Trough

Rn interventions concearning drug therapy include the following EXCEPT

Teach client to wait week after occurence of S/S to see if the disappear

Low protien level in blood decreases the # of protien binding sites and causes an increase in amount of free drug in the plasma, potential result in drug overdose.
Examples of what can cause low protein levels include:

malnutrition, Elderly
Both do not obtain suffecient nutrients.

Which following statements are correct?

Drugs are absorbed faster in acidic fluid
The very young have decreased gastric acidity
Liquid drugs are more rapid available in GI absorbtion than of solids

What resources provides basis for standards in drug strength and composition throughout the world?

International Pharmacopeia

Current authoratative source for drug standards

U.S Pharmacopia/Natl. Formulary

Primary purpose of federal legislation r/t drug standards

Ensure Safety

The FDA madates to monitor/control the manufacture and marketing of drugs come from (what act)

The food,drug,and cosmetic act 1938

Which legislation ID those drugs that req a prescription and a new prescription for refills.

Durham Humphrey Amendment

Kefauver-Harris amendment was est. to improve safety by requiring which of the following in the literature

Adv./SE and Contraindications

What state law controls drug administration by nurses

RN practice Act

Controlled substances are grouped in schedules/categories what is the Number of schedules?

5

Which schedule accepted medical use

II thru V 2 thru 5

What is correct about schedule drugs that have potential for abuse

1 highest 3 moderate (4 and 5 not so much they are OTC)
1-3

Your client abuses drugs- LSD Herroin which belong to what schedule

Scedule 1

The nurse needs to know that codeine in cough syrup belongs to which schedule

5 Codeine in cough meds
3-codiene alone

In institutions agencies controlled substances must be stored

In locked locations

Drugs in canada Schedule G moderate potential for abuse and require prescriptions for both initial allocation and refills.
Drugs are similair to what US schedule

3 moderate potential

In canada OTC preparations are admin by what groups of the respective provinces?

Pharmacy acts

Accord, FDA drugs un which preg category are considered not to present risk to fetus

A- no risk to fetus

Two advantages assoc w/ use of generic drugs include

Lower costs
Same active Ingredients

Two DIS-advantages assoc w/ use of generic drugs include

Less extense of testing
Variation in response (tablets not 100 med/ may contain fillers)

which drug resource is published annually and updated monthly?

American Hospital Formulary

Which legislation decreases the time for approval of drugs used for treating aids cancer

Drug Regulation Act 1992

the nurse should be alert to counterfeit prescription drugs, the following are clues to counterfeit

Variation in labels
Differed appearance of drug
Different dose remained in applicator/vial etc

Implications of HIPPA include

Release of information only to client
Private area for pharm consults
Client signs for copy of privacy statement

Risk for injury: included in which Rn phase of nursing process

potential RN dx

Which is an example of Primary source information

Subjective date from pt

Obtains CL weight to be used for future comparison- included in what phase RN process

Assessment
Collect date- baseline data

The CL will recieve adequate nutritional support through enteral feedings is included in which phase the RN process

Planning

The CL will be free from hyperactivity is included in which phase RN process

Planning

instruct pt to aviod caffeine is what part of the RN process

Implementation

Evaluate the effectiveness of the drug is included in what part of RN process

evaluation

Sleep pattern disturbance Is included in what part of the RN Process

Assessment/Rn Dx

Advise CL to report rxns to nausea/vomiting to healhcare provider is included in what part RN process

Implementation

Anxiety Is included in what part of the RN Process

Potential Rn Dx

instruct CL not to d/c meds abrubtly Is included in what part of the RN Process

implementation

The Rn is developing a teaching plan for the CL which of the following is suggested to be included?

Actively involve CL
Provide written instructions
Consider use of variety media sources
Provide for return demonstration

factors commonly resulting in non adherance with a drug therapy plan includes:

Forgetfullness
Knowledge defecit
Side Effects
Language culture barriers
Finances

Pt refuses an essential heart medication, what does the RN do next?

Seek reason from pt.

Pt complains to his RN about heartburn, nurse notes in the medication profile that an antacid has been ordered PRN, what will the RN do next

Verify last time med given, determine appropriateness to give next dose

which statement about verbal order is correct

should only be used in ER situations

what process is used to eliminate med errors in healthcare enviroments as pts transition from one setting to another

medication reconcilliation records

A Rn is having difficulty reading a physicians order, the RN knows the Physician is very busy, does not like to be called what should RN do?

Call MD clarify order

What Rn action causes most med errors

administering unfamiliar medications

What is evidence based medicine

scientific research used for making clinical decisions
Quality core measures used on basis-pts conditions
Data voluntarily submitted by healthcare institutions providers

most drug testing is US has which ethnic group for research participants

European Americans- trialed more often in europe prior to us.

african americans

Are less responsive to
Beta Blockers- Genetics

Many cultural groups have lactose intolerances, S/S include:

Bloating

Dietary counseling to meet calcium requirements include encouragement to eat

Leafy greens

Communication on of the domains of culture include

Maintenence of eye contact

cultural woman per mothers requests eats red clay to provide for her unborn fetus- cultural practice she intends to continue- you as an RN should

calculate determine amount clay she eats
Do not discourage any cultural practice unless harmful to self others

Newly Dx diabetic pt taking oral hypoglycemic- his currando recomended that he drinks sabila tea 3x day- as home health Rn you should

Do not discourage
Encourage his practice to drink tea- ensure he follows regimen with other meds- hypoglycemic meds

factors that affect CL physiological responses to medications include

Genetics
Diet
Age

Factors that affect CL adhereance with medications include:

Poverty
Trust in health care providers
Access to healthcare

The Rn planning to instruct Hispanic American Female about drug regimen presribed for newly Dx HTN when developing the plan the RN actions should be based on which of the following most relevant information

CL lives in an extended hispanic family setting in which men are the decision makers

IM - Needle size,Location,Angle of admin

IM
20-22Needle 1-1 1/2
Locations
Vastus lateralis
Hip(ventrogluteal)
deltoid no more 1 ml
dorsogluteus
Angle
90
or 45
dependant upon Fat content

SubQ Needle size,Location,Angle of admin

Needle size
24-28g 5/8 inch
Location
Buttocks
Abd
back region arm
Fatty areas upon body
Angle 90*

Intrdermal Needle size,Location,Angle of admin

needle size 5/8
Location usually forearm
angle 10-15*

IV Needle size,Location,Angle of admin

20-22 common sizes dependant upon needs
location arms Med Ac hands etc
Angle no angle required

Injection site well defined by bony prominence landmarks is:

ventogluteal -hip

Preffered site for IN injections in infants and children

Vastus lateralis-infants

Site easily accessible but NOt suitable for repeated injections or injections of ml or more

Deltoid

Preffered site for Ztrack

ventrogluteal

Order states Give Multi V ii caps Po OD example of what category of order

standing order

Order to recieve Demoral mg STAT is an example of what kind of order?

STAT order

When you calculate the dose for cardiac med, the drug dose is LARGE the best initial action is to:

Check you calculation

Preffered method to correct a charting error is to:

Draw single line thru error DTI

you read in the chart that your pt is allergic to one of his prescribed meds your first action is to

Withhold the medication call the provider

One of your pt medications is in a liquid form you pour the medication with the container at eye level and read the meniscus at what point

Lower part curve

Your Cl is not wearing Id band what should be your first Rn Action

report find-have new id put on Cl

universal Precautions require that you do all the following EXCEPT

Cap needles

before storing unused stable solutions from opern vials in refrigerator, the Rn should write the following info on bottle

DTI

When Cl refuses to take medication, RN must:

Document reason not taken

For CL who are vomiting/Comatose medication administration is conraindicated via which route

Oral

When is the best time to administer oral medications if food interferes with absorbtion

On an empty stomach

When applying medication topically, the Rn should use all of the following EXCEPT what to avoid contact with the medication

Hands

If glucocorticoid is ordered with a broncho-dilator you need to wait how many min b/w admin medications

5 min

When adminsitering ear drops, the CL should be sitting with the head tilted towards which side

unaffected side

In an adult, gently pull the auricle in what direction before instilling ear drops

Up,Back

ear drops best administered

At room temperature- maintains normal equilibrium

which of the following are not allowed by the Joint Commission

U, IU, trailing zero,Qd
abreviations of most

The Right to education includes

*Client recieves correct information about the drug and how it relates to situation
*Possible Side effects
*Laboratory Monitoring

Drugs with simalair actions, such as pennicillins/cephalosporins can result in

cross resistance

condition that occurs when normal flora is distributed during Ab therapy AKA

Super Infection

All the following statements True about pharmocokinetics of penicillin derivatives amoxacillin and cloxacillin EXCEPT

Both drugs absorbed well from GI

Allergic effects occur in what percentage of persons recieving penicillin compounds

5-10%

Drug interactions occur with cephalosporins and which of the following

Alcohol

when probenicid is administered with cefazolin or cefamandole which of the following results

drug action is increased

pt suffering from Ecoli infection- unable to swallow pills - an oral suspension of cephalexin 250mg ordered
Usual dose is? How often is medication generally scheduled for?

250 -500 mg Q6h

how many mL of drug should pt recieve per dose and per 24hrs.

250 ordered/250mg per 5mL on hand
((5ml per 6hrs))
6x4=24hrs

which of the following drugs/foods are known to change the action of keflex

uricosurics

The health teaching plan would include all the following EXCEPT:

finish Ab (from previous) False!!

The drug that may be used as a substitute for penicillin is

Erythromycin

Broad Spectrum penicillins may decrease the effectiveness of which of the following

Oral Contraceptives

the Rn should observe a Cl taking cephalosporin for which S/S

Nausea
Vomiting
Diarhea
Itching

specific Rn interventions for a CL taking ceftazidime would include which following

Take C&S
Assess Allergic rxn
Monitor I/O

Sulfonamide derivatives for treatment of second and third degree burns

Mafenide Acetate-sulfamylon

Drug used to treat seborrheic dermatitis

sulfacetamide sodium-isopto cetamide

45 yo pt admitted for treatment of Severe UTI- current meds include Septra and digoxin
Usual dose of septa is?

160mg TMP/800mg SMZq12h

Septra Tablets- e scored tablets contains 80mg trimethoprim and 400 mg sulfamethoxazole. How many tablets should pt take per 24 hrs?

80mg ordered/400mg on hand =2 tablets per 12 hrs =4daily

Rn should

Admin medications and extra Fl.
Monitor Urinary Output
Observe allergic response

Which possible S/E trimethoprim/sulfamethoxazole should the RN advise the CL

Anorexia
Crystalluria
Photosensitivity
decreased WBC/Platelets

Which following true regarding sulfonamides

increase anticoags effect of warfarin
SJS is an adverse rxn
Increased hypoglycemic effect w/ sulfonylureas

After Cl w influenza makes which of the following statements: RN concludes that the Cl appropriatly understands principle of self admin of prescribed oral Ab.

i will continue to take the Ab as ordered even though i no longer have S/S

after investigating Amoxicillin for 10 days the Cl develops diarhea which inclded eight watery stools per day- Rn should anticipate the need for which following

monitor clinical manifestations of metabolic acidosis
collect stool specimen for cytotoxin assay to detect C-diff

A Cl with pneumonia being treated with amoxil what assessment data would be best indicator to the Rn of the effectiveness of drug therapy

Pulse Ox

after receiving penicillin for days-Cl reports muscle weakness,numbness,tingling, weak pulses,irregular heart rate decreased bowl sounds- rn should review labs for

Hypokalemia

which of the following is NOT a flouroquinalone w/ daily dosing

ofloxacin-floxacin

Usual dose of tetracycline is

250-500 q6h

a laboratory test is influenced by tetracycline is

blood urea nitrogen BUN- Nephrotoxicity

For best results it is recommended that tetracycline be taken

With Extra Fluid- reduce toxicity flush kidney
On an empty stomach-inhibits drug rxn

The health teach plan for a pt taking Tetracycline would NOT include which of the following?

Anticipate Urinary Urgency

Order Zithromax for Mild moderate Strep, How many mL of suspension are req for 1st dose and each of the daily doses for next 4days.

500 mg max daily dose/200mg per 5ml hand=12.5mL 1st Load dose
250 mg normal daily dose/200mg per5ml hand=6.25mL remaining 4 days

The Rn knows that food ( ) absorbtion of the drug by 50% therefore rn should instruct pt to

Decreases absorption, Take medication on an empty stomach

The Rn assess a CL taking Gentamycin for S/E including:

Nausea
Ototoxicity
Photosensativity
Thrombocytopenia

The Rn knows that which of the following drugs modify the action of tetracycline

Iron
Antacids
Warfarin
Milk Products

Specific Rn interventions for female client taking tetracycline include

Storing drug away from light
Monitoring lab results
Obtain specimen for C&S
Advising CL to use adtl. contraceptives

Rn assess CL recieving Gentamycin for which of the following toxicities

Nephrotoxicity Ototoxicity

After Liquid tetracycline is ordered for 2yr old, Rn should provide which important instruction to LPN who is admin med.

Withhold drug Telephone MD
Below 8 year old- tetracycline causes Causes tooth discoloration

a CL has been prescribed to take both tetracycline and sulfonamide drugs- when providing teaching what is the priority information should the Rn give the CL r/t adverse drug affects

Use protective measures when exposed to the sun
sulfonamides- photosensitivity

Cl taking Azithromycin Rn should apply which interventions

Monitor periodic LFT's
Tell CL to report Hearing loss
Instruct CL to report evidence of superinfection
Teach Cl to take oral drug 1hr AC 2hr PC
Avoid antacids from 2 hrs prior to 2hrs after taking Azithromycin admin.

The Rn closely monitors the CL taking lincosamides for which serious adv. effects.

Pseudomembranous Collitis

The Rn enters CL room to find that pt heart rate 120 BP 70/50 he is flushed. Vacocin running IVPB- the Rn interprets this as severe adv. affects of red man syndrome- which should the Rn do?

Reduce the infusion to 10mg/min
vanco- has peak/troph and should not be stopped immediatly!-must be weaned

The Rn is admin tetracycline to client what would be appropriate teaching

Take sunscreen precautions- photosensitivity tooth discoloration

CL is taking levoflaxacin(levaquin Rn knows that which is true regarding this drug

Adv affects include dysrythmias

What should the Rn include when teaching about gentamycin

Cl should report Hearing loss
Cl must use sunscreen
Cl will be monitored for Ulcers and Vaginitis
Cl should increase Fl intake

The rn acknowledges which RN interventions for client taking cipro

Obtain C&S
Tell Cl to avoid taking Cipro w/ antacids
Monitor the CL for hearing loss
Encourage Fl to prevent crystalluria

A Cl is recieving levoflaxacin in addition to oral anticoag what treatment should the RN anticipate admin if CL experiences an adv. effect as a result of his combo.

Phytonadione Vit K

The Rn notes that the Cl taking doxycycline vibramycin are jaundiced and lethargic. What lab tests results would be most specific for the RN to review

ALT SGPt
Alanie aminotransferase -Specific to Dx liver disease or impairment

A Cl is recieving an AI drug and begins to wheeze the rn anticipates initial administration of what drug

Epinephrine -treatment for anaphalaxis

What teaching intervention is appropriate for CL taking an Ab that causes diarhea secondary to elimination of normal intestinal flora

Include yogurt /buttermilk in diet

Serious Adv.Effect of isoniazid is

Hepatotoxicity

A Contraindication for prophalytic treatment of TB is

Alcoholism

Colistin S is excreted via which of the following

Feces

Collistin S given Im special considerations

Adding 1 % lido to medication-prevent pain when admin

During Interview the admission- you should obtain which following

Last PPD,Chest X ray results

when CL taking INH frequent monitoring of which of the following is req.

Liver Enzymes

Which of the following drugs/food changes the action of INH

Cheese(high in thymine) antacids

Health teaching would include all following EXCEPT

Monitor daily weights

Which following is are true about the drug rifapentine

Newest drug for treatment of TB
Twice weekly dosing
Taken in combo with another TB med-to avoid resistance

67 y/o pt treated for hisptoplasmosis recieving amohotericin B Med generally admin which route

IV

Amphoteracin B should monitor which lab results

BUN

What health teaching plan for ampho B would include all except

Consume NO alcohol

Metronidazole primarily used for treatment of disorders caused by organisms in which of the following

GI tract

In combo of agents metronidazole commonly used for Hpylori assoc w/ reoccurent

Peptic Ulcers

S/E of metronidazole may include

headache/ depression

Which of the following must be frequently monitored for the CL taking fluconazole

AST ALT BUN

S/E of peptide include which of the following

dizziness,Neuro/Nephro toxicity Numbness/tingling

S/E rxns of Amphotericin B include which of the following

flushing, hypotension, Hypokalemia, thrombophlebitis

Serious Adv ganciclovir used for treatment of cytomegalovirus

Throbocytopenia

Most common side effect for helminthiasis is in the

Intestines

Acyclovir is effective against herpes virus was 1st introduced as an:

Antineoplastic

Drug interaction occurs b/w acyclovir and which following drugs

probenicid

Which of the following drugs are effective in combating herpes simplex viruses HSV-1/2

Familclovir
Ganciclovir
Valacyclovir
All of above**

Which of the following is true about relenza & tamiflu

Should be taken w/in 48 hrs of flu symptoms

Today malaria is a common disease caused by

protozoa

Drug choice for treatment of chloroquine-resistant malaria is

combo of antimalarials

Chloroquinalone affects all of the following lab tests Except!

Hemoglobin

Cl taking cloroquin needs to know about which of the following possible side effects

Pruritus
Fatigue
Anorexia

RN interventions during antimaliral drug therapy include:

Monitoring urinary output/liver fxn

what is recommended shcedule for taking chloroquine in prep for a visit to a country infested w malaria

antimalrial drug-
before,during,after visit

Assessment Cl for treatment w antihelmintics includes which of the following

History food intake
Collection stool
determine if other household member have same s/s
All above***

Which of the following drugs are commonly used in the treatment of Roundworms/pinworms

Mebendazole

long term therapy with anti-helmintic is req with all the following drugs except

Mebendazole

Cl teaching for antihelmintic

Importance of hygeine

When teaching Cl taking Acyclovir about S/E that are assoc with this drug which of the following should the RN include

Nausea
Headache
Lethargy

chloroquine increases effects of which of the following drugs

Digoxin
Anti co Ags
Neuormuscular Blocks

Neuro problems r/t antihelmintics include which of the following

Dizziness headache weakness drowsiness

Rifampin is being initiated prophylactically for CL who lives w family member who has haemophillus influenza meningitis what point should the RN include in client teaching about this med.

Protect undergarments becouse rifampin will change the urine to Orange red color

A CL w benign prostatic hyperplasia BPH is recieving amantadine for influenza A the RN should monitor which of the following S/E

Increased Risk for Urinary retention

which intervention is of highest priority for RN working w client who has Herpes Zoster shingles and who has recently began drug therapy with acyclovir

Encourage Fl intake of 2500-300 mL daily unless otherwise contrindicared

While teaching the CL about taking new prescription of oral metronidazole the Rn should include which of the following sig elements

Avoid alcohol

Which of the following statements best indicates that an immuneocomprimised cl understand self app. of topical drug acyclovir

I need to touch the lesions and the open side of container with finger cot.

The Rn assess for which most common side effect in a cl w tb taking INH

Parathesia in limbs
numbness tingling

A Cl dx with Hsv3 Rn understands that illness is better known by which name

Shingles-an adult

Cl with dx of intestinal ambeisas develops n/v/d flushing/tachy/hypotension palpatation- pt states consumption of several alcoholic beverages Rn should obtain drug history for which drug.

Metronidazole- ambeiasis

Cl has developed Vag candidiasis the Rn should know that which med is approp treatment for this condition

Terconazole

Asprin,NSAID's and acetaminophen act by:

Competitively blocking the neuronal terminals

Salicylate intoxication(salicylism) is characterized by which of the following symptoms

Hypoventilation

Which of the following statements about asprin is TRUE?

Asprin should generally be avoided for 1-2 weeks before and after surgery

Which of the followung drugs on overdose is known to cause live failure

Acetaminophen (tylenol) APAP

At team conference, Rn explains that the anti-inflammatory actions of salicylate are MOST LIKELY DUE TO:

Inhibition of prostaglandins

Which of the following symptoms would the RN expect in a patient experiencing salicylism

Dizziness,Tinnitus,Mental Confusion

When administering a salicylate, the RN correctly administers the drug

W food or milk

While taking acetaminophen, pts consume alcohol habitually are monitored by the RN for symptoms of toxicity, which includes:

Liver Tenderness

Which of the following drugs would the RN MOST LIKELY deliver to a child w/ an elevated temperature

Acetaminophen (asprin is toxic to children under the age of 8)

A RN instructs the Pt taking aspirin to avoid foods that contain salicylates becouse of the risk of adverse RXNS - which foods should the pt avoid?

Prunes, Tea

The physian orders acetaminophen elixir 180 mg orally- acetaminophen elixir is available in a 120mg/mL solution. The RN administers?

180/120= 1.5 mL

Aspirin 650 mg oral is prescribed On hand is aspirin 325mg tablets. The RN should administer?

650/325= 2 tablets

The vascular phase of inflammation is associated w/

Vasodilation w/ increased permeability

The following are among 8 groups of NSAIDS w/ which EXCEPTION

Macrolides- their own class

Which of the following is NOT a property of anti-inflammatory agents

Anti-Hypertensive

A common side effect of NSAID are

GI distress

The mechanism for action of non-steroidal anti-inflammatory drugs include

Inhibition of synthesis of prostaglandins

The cl is taking large doses of aspirin for an arthritic condition the RN needs to know ALL the following except:

Aspirin can lower blood sugar in cl. w/ diabetes, causing hypoglycemia

The analgesic drug to give to a child w/ a virus or the flu is

Acetaminophen(aspirin toxic children under age 8)

The medication for a child w a virus should be one that prevents

REYES SYNDROME

J.W is taking Ibuprofen
Ibuprofen is a rapid acting NSAID that inhibits prostaglandin synthesis the agent drug is classified as an

Propionic Acid Derivative

Ibuprofen is generally scheduled to be taken

3-4 times per day
8-900 mg max 3600

Usual dose of Ibuprofen is

300-800mg per dose

Ibuprofen (motrin,advil,nuprin) is frequently taken anti inflammatory analgesic and atipyretic agent- Which of the following is true about Ibuprofen

It causes less GI upset that other NSAIDS

J.W(ibuprofen) teaching plan would include all the following EXCEPT

Suggesting a decreased fluid intake

Piroxicam - feldene is a NSAID- Its advantage over othe agents is which of the following

Long Half Life

Nursing Stratagies r/t administering NSAIDS include ALL of the following EXCEPT

Advising the Cl alcohol can be taken w/ NSAIDs

what characteristics are associated w celecoxib-celebrex

NSAID-Relieves pain and inflammation w/ out causing GI distress

which of the following statements are true regarding Ketorolac-Torodol SELECT ALL THAT APPLY

Can be administered Orally
Can be administered IM
Can be administered IV
May be given only 5 days or LESS
Incorrect answers begin w/ HAS
Has an efficacy equal to morphine
Has a usual dose of 15 mg IMq6h

The RN monitors for which common adverse reactions when administering naproxen-naprosyn to a pt.

HA ,DYSPEPCIA

An elderly Pt is reciving sulindac- the Rn is aware that older adults taking NSAIDS are at increased risks for:

ULCERS- Disease

When a pt is recieving an NSAID the RN must monitor the pt for

GI symptoms which can be serious and sometimes fatal

which of the foollowing statements would the RN be cerain to include in a teaching plan for the pt taking NSAID

Avoid the use of aspirin or other salicylates

The RN teaches a pt to self administer sumatriptan-immatrex when is this injection contraindicated?

Weekly prophylactic use- Expensive not used unless necassary

Naproxen -naprosyn oral suspension 250 mg is prescribed -the dose on hand is oral suspension 125mg/5mL
How much should the RN administer

250/125x5mL= 10mL

The physician orders celecoxib celebrex 200 mg orally the nurse has celecoxib 100 mg tablets. How much shou,d the RN administer

200/100=2 tablets

EA age 53 just returned from OR-48hrs prior meperdine demorol has been ordered for pain control
The usual dose of meperidine postoperative is

50-100mg q4h PRN

While pt is taking meperidine frequent monitoring of _____ is required

Blood pressure

The nurse assesses for toxic affects of the drug- which of the following is an adverse reaction

Constricted Pupils

Which nursing assessment would be least important when monitoring a CL recieving meperidine

Fluid intake

The teaching plans for EA taking meperidine would include

ALL OF ABOVE
Not to use alcohol or CNS depressants while taking meperidine
To report side effects
Prevention of constipation

Based on your knowledge of analgesia, which factor is MOST relevant to the relief of chronic pain

Use of drugs w/ long half lifes

The opiod antagonist used to treat an overdose of morphine like substance is

Naloxone(NARCAN)

Mixed Opiod agonist-antagonists were developed in hope of decreasing

OPIOD ABUSE

Withdrawl symptoms usually occur_____ hrs after the last opiod dose

24-48HRS

Methdaone Treatment programs can be effective in helping the opioid addicted person withdrawl- which of the following is the recomended maintenence dose of methadone

40-120mg/d

The benefits of methadone over other opioids is (are)

Less dependancy -Daily Dosing
A and C Only

Elderly Cl frequently require a reduction in opiod dosage to avoid severe side effects-Reasons for this include which of the following

Decreased excretion of drug
Decreased metabolism of drug
Polypharmacy
ALL OF ABOVE!

Which of the following drugs tends to be more toxic in elderly CL than of middle aged?

DEMEROL

It may be difficult to assess pain in children-
Pain management is more apt to be successful if the RN does which of the following

Uses age appropriate communication skills
Uses ouch scale
Discusses childs response w. parents
ALL OF ABOVE!

Drug- Food Interaction w/ acetaminophen include which of the following

Increased effect w/ caffeine and diflunisal (excedrin and migraines?) Assists with speedy interactions

Side effects of opioid analgesics/ agonists include
SELECT ALL THAT APPLY

SEDATION
CONSTIPATION
NAUSEA/VOMITING
RESPIRATORY DEPRESSION
DOES NOT INCLUDE HTN,URINARY FREQUENCY

Which of the following is true regarding naloxone(NARCAN)
SELECT ALL THAT APPLY

Is used to treat an overdose
Has higher infinity to opiate receptor sites than opiods
Is approved for use in neonates to reverse respiratory depression
-NOT CORRECT ANSWERS
Is only administered IV, May be given every 1 min to max of 10mg

After administering an opioid analgesic to a cl the RN would make which priority to follow up assessments in addition to pain relief

Respiratory Rate , LOC

A Cl is recieving both Selegiline (eldepryl) and MAOI and meperidine Demerol -what instructions should the RN provide to the Unlicensed Assistive Personel UAP

Use auto BP to measure BP

The Nurse explains to the PT that some opiods may be used as a part of the pre-operative medication regimen to:

Lessen anxiety and sedate the Pt

Each time the pt requests an opioid analgesic , the RN must

Determine the exact location and intensity of the pain

When administering opioid analgesics to an elderly pt the RN must monitor the pt closely for

CONFUSION

When administering a timed released medication to a pt , the RN must be aware that

IT SHOULD NOT BE CRUSHED

Which of the following findings requires that the RN immediatly contacts the primary health care provider

Complaints of Breakthrough Pain

A pt is prescribed oral morphine sulfate 12mg- the dose available is 10mg/ml the RN administers

12/10=1.2

A pt is prescribed fentanyl sublimaze 50mcg IM 30 min before surgery.The Rn has a vial w dose 0.05 mg/dl the RN calculate and administers how much medication

50mcg /3=0.05
0.05/0.05=1mcg

Antipyschotic drugs are useful in the management of

Pyschotic illnesses

A full effective therapuetic response to anti-pyschotics usually takes

3-6 weeks

Cl. education for use of these drugs is important - The RN actions for these individuals include advising the CL and significant others that

The drug should be taken as prescribed and to consult the health care provider before discontinuing the medication

Typical or traditional anti-pyschotics may cause extrpyramidal symptoms EPS or pseudoparkinsisms Which of the following symptoms is NOT Caused by EPS

Downward Eye movement

Cl taking high potency typical antipsychotics may develop adverse EPS reactions such as:

AKATHASIA

The most severe adverse extrapyramidal reaction is

Tardive Dyskinesia

Anticholinergic agents are used to decrease these EPS Examples include

Benzotropine (cogentin) and tirhexyphenidyl (artane)

Ds has been Dx w Schitzophrenia-The health care provider prescribed fluphenazine prolixin
Fluohenazine Prolixin is classafied as

PiperaZINE Phenothiazine

Fluphenazine Prolixin commonly produces effects of

EPS

The nurse is aware that an overdose of fluphenazine would require which of the following treatments

Activated charcoal administration

Antipsychotic dosage for older adults should be

25%-50% less than that of a normal adult dose

Haloperidol (haldol) is frequently used as an antipyschotic the RN should know that it

Has a sedative effect on agitated combatative persons

A cl is taking Haldol 5 mg TID and C/O nearly falling down when he gets OOB the cl is likely experiencing

Orthostatic Hypo Tension

The category for atypical antipsychotics is

Serotonin/Dopamine antagonist

The atypical antipyschotics marketed in the US since 1990 have a weak affinty for the D2 receptors; thus these agents cause

Fewer EPS

Atypical Antipsychotics have a stronger affinity to

D4 Receptor, They block serotonin receptors

Antipyschotic drugs are NOT used treat

Violent behaviors

Serotonin antagonists atypical antipsychotic are effective for treating which types of scizophrenia

both positive and negative symptoms

The anxiolytic alprazolam xanax is from which drug group

benzodiazepines

Dependancy S/S that can occur when taking benzodiazepines and are abruptly stopped include:

Irratibility and nervousness

JT has anxiety over recent terrorist attacks
The health care provider prescribed diazepam(valium) 5mg BID
Valium is classified as an

Benzodiazepine

JT asks what BID means your response would be

Twice daily

JT should be told NOT to take Valium w/

Alcohol, becouse it can cause CNS depression/Respiratory distress

Lorazepam is an anxiolytic drug - however it may be prescribed for other clinical problems. Which of the following may it be prescribed-Select All that APPLY

Correct
Anxiety
Status Epilepticus
PreOperative Sadation
Incorrect answers
-Manage Scitzophrenia
-Depression and Delusions

Contraindication for taking fluphenazine include which of following: Select all tha APPLY

Blood Dyscrasias (increased/Decreased RBC)
Hepatic Dysfunction
Incorrect answers
Neuromuscular Pain
Subcortical Brain Damage
Narrow Angle Glaucoma

Which is drug of choice for treatment of enuresis in children

Imipramin-Trofranil

PH is taking MAOI for chronic anxiety and fear
PH begins taking phenelzine(nardil) This medication is generally scheduled to be taken

2-3times daily

Usual adult dose of NARDIL

15mgTID

Assessment is essential w/ Cl taking MAOIs frequent monitoring of which of the following is required

Blood Pressure

The Nurse assesses PH for Side effects of the drug Which of the following is NOT a side effect of MAOIs

Urinary Retention

PH were to ingest drugs and or foods that interact which of the following is likely to occur

Hypertensive Crisis

PH is having difficulty selecting foods from his menu. Tyramine rich foods to be avoided include

Cheese - Chocolate- Raisins

The two herbs that may be used for management of mild depression with health care providers approval are

St Johns Wart, Gingko

before surgery the use of many herbal products should be discontinued

1-2 weeks before surgery

The SSRIs tend to be more popular than TCAs because they have fewer Side effects SSRIs

Cause Less sedation and fewer hypotensive side effects

Many subcategories are listed as atypical antidepressants which of the following groups is NOT classified as an atypical antidepressant

SSRIs

RT is an acutely manic cl prescribed lithium for 1st time
Nursing Interventions associated with lithium Carbonate in the management of bipolar disorders include all of the following EXCEPT

Understanding the drug is most effective in the depressive phase

Specific Nursing interventions w RT would include monitoring ALL of the following EXCEPT

Daily ECG

Understanding RT need for hydration you encourage him to have at least______mL of fluid daily

2000mL

After taking lithium carbonate 1200mg day for 5 days RT remains agitated and hyperactive todays plasma level was 0.8mEq/L and RT c/o feeling slow and having increased thirst.
Your analysis is that the CL is:

Still manic w/o signs of toxicity

Health teaching for RT includes all of the following EXCEPT

If the medication is stopped the depressive symptoms will reappear

You are knowledgable about the drug and or food interactions with phenelzine (nardil) Which of the following types of drugs /foods are known to interact with Nardil SELECT ALL THAT APPLY

Beer Cheese Cold Medications
Incorrect answers are pork, citrus

Potential Side effects/adverse reactions for a cl taking fluoxetine (prozac) include :SELECT ALL THAT APPlY

Tremors,Seizures,Insomnia,HA
Incorrect = dysrhythmias

A pt is prescribed phenytoin for a recurrent convulsive disorder the nurse informs the pt that the most common adverse reactions are

Related to CNS

When administering phenobarbital to an elderly pt, the nurse should monitor the pt for unusual effects of the drug such as:

Marked Excitement

When caring for a pt taking succinamide for seizure control, the nurse monitors the pt for blood dyscrasias. Which of the following symptoms would indicate that the pt may be developing a blood dyscrasia?

Dyscrasia- Abnormal Blood Count Levels
Sore throat and General Malaise

Which statement would be included when educating the patient taking trimethadione for seizures

Wear sunscreen and protective clothing when exposed to sunlight

Which of the following adverse reactions, if observed in a pt prescribed phenytoin, would indicate

ATAXIA- lack of voluntary control of movement

The nurse is preparing to administer an anticonvulsant for status epilepticus. The PCP prescibed lorazepam-ativanmg IV. The drug is availble in a dosage of mg/mL the Rn administers

4mg/2mg/mL= 2mL

Zonisimide 200 mg is prescribed. The drug is availble in 100mg tablet- the Rn administers

200mg/100mg= 2Tablets

The PCP prescribes ethosuximide syrup 500 mg for a pt with abscence seizures.The drug is available in a strength of 250mg/5mL the Rn administers.

500mg/250mg x 5 mL = 10mL

The drug of choice for seizure disorders that have not responded to other anticonvulsant drug therapy is

Carbamazepine

Which of the following statements is true about anticonvulsant drug therapy is

Carbamazepine

Which of the following statements is true about anticonvulsants

Phenytoin has been linked to cardiac defects

Vigabatrin is an anticonvulsant with qualities including which of the following

Inhibits enzyme that destroys GABA
Adult dose is 1-4g/d in divided doses
Used to treat complex partial seizures
ALL OF ABOVE ARE CORRECT

JA 24 old has a seizure disorder and is going to start taking phenytoin -the drug of choice to control seizure activity
JA will initially recieve IV phenytoin which should be administered at a maximum rate of

50mg/minute

After several days of IV medication,JA's medication is changed to the oral form.
Oral doses of phenytoin are generally scheduled:

2-3 times daily

When JA begins taking oral medication the nurse should know that the dose will most likely be

LOW- Start low increase as needed until therapuetic levels are reached

JA asks how long she will need these medications. The nurses answer is based on the understanding that

The medication are taken for a lifetime

For maintence control of seizures in the adult, the usual dose of phenytoin is

100mg TID

Serum phenytoin levels should be monitored to determine if the blood serum level is within the therapuetic range, thus avoiding toxic levels.The therapuetic range of phenytoin is

10-20 mcg/mL

You are knowledgable about drug interactions with phenytoin.Which of the following drugs are NOT known to alter the action of phenytoin.

Laxatives

In the event that JA experiences a seizure, you would document all of the following EXCEPT the

Ability to stop the movements

you observe that JA is reciving adequate nutrition.Phenytoin may cause all of the following EXCEPT

Diaphoresis

EK has been taking PHENYTOIN for 20 years. He has not reported seizure activity while taking the maintence dose.
You would assess the Cl. for which common side effect of the drug.

Ginigival Hyperplasia

As a result of EK long term use of phenytoin, which laboratory test results would be monitored?

Blood Sugar-Hypoglycemia

Trimethadione and succinamides are effective drug therapy for

Petite Mal Seizures

Diazepam is the drug of choice for the treatment of

Status Epilepticus

Phenytoin is effective in the treatment of

Grand Mal Seizures

Zonisamide -Zonegran is contraindicated if the Cl. is allergic/sensitive to which of the following

Sulfonamides

Which of the following is are true about seizures and anticonvulsant use during pregnancy (SELECT ALL APPLY

Seizures increase 25% in epileptic women
Many Anticonvulasants have teratogenic properties
Anitconvulsant use increases the loss of folic acid
InCORRECT ANSWERS
Anticonvulsant use increases the effects of vit K
Valproic Acid causes major malformations in

Your Health teaching plan for the Cl taking phenytoin would include which of the following SELECT ALL APPLY

Restrict fluids while taking phenytoin
Correct answers
Urine may be harmless pink or reddish brown color
Alcoholic Beverages are not recommended
Drug may have teratogenic effect of fetus
Avoid Aspirin while taking phenytoin

Which of the following drugs is used to treat glaucoma

Pilocarpine

In excessive doses, acetylchoilesterase inhibitors can paradoxically potentiate a non-depolarizing neurumuscular blockade TRUE OR FALSE

True

Which of the following changes in parasympathetic nervous system function are produced by the administration of antimuscular drugs? CHOOSE TWO

BRADYCARDIA, BRONCHODILATION

You are scheduled to provide a general anesthetic to a pt recieving treatment with echothiophate eye drops.Which of the following drugs should be avoided in the perioperative period?

Succinylcholine____________
Mivacurium______________
Atracurium
A&B----------------------CORRECT ANSWER
B&C
ALL OF ABOVE

After spending the day treating his crops with insectisides, a 65 year old farmer presents to the emergency room with blurred vision, severe abdominal cramping, wheezing, and bradycardia. You determine that he is experiencing a______________ and should be

Cholinergic Crisis, Atropine

Which of the following drugs crosses the BBB the LEAST

Glycopyrrolate

Which of the following drug types is used in the treatement of myasthenia gravis

Cholinesterase Inhibitors

In which of the following areas are muscarinic receptors located? CHOOSE TWO

Parasympathetic Postganglionic Effector Sites
Skeletal Muscle

If your goal is to decrease salivation AND promote sedation for a pt. perioperatively, which of the following is the BEST choice

Scopolamine

A pt. with glaucoma is prescribed pilocarpine eye drops. One adverse reaction that the nurse will expect with the use of this drug is?

Temporary Loss of Visual Acuity

The primary care provider allows the pt to keep pilocarpine eye drops at the bedside and to self administer the eye drops four times daily. The nurse:

Must check with the pt to ensure the medication is used properly and at the righy time

A pt has recived a Dx of myasthenia gravis and begins a regimen of ambenonium. The nursing assessment is important becouse the dose of the drug

IS frequently increased or decreased early in therapy- helps to determine therapuetic levels.

The dosage of pyridostigmine bromide-mestinon is 600mg/day.How many tablets will the pt take if they are 60mg each.

600mg/60mg= 10 tablets

The primary care provider prescribed 2.5 mg of bethanechol subQ. The drug is available in a solution of 5 mg/mL. The nurse should administer

2.5mg/5mg/mL = 0.05mL

The pt taking solifenacin -vesicare for an overactive bladder c/o dry mouth. the nurse should

Encourage the pt to take frequent sips of water

which of the following adverse reaction would the nurse expect after the administration of atropine as part of a pt. preoperative medication regimen

Enhanced secretions of the Upper Respiratory tract

Becouse of the effects of cholinergic blocking drugs on intestinal motility, the nurse must monitor the pt taking these drugs for the development of

Constipation

Cholinergic blocking drugs are contraindicated in pt's w/

Glaucoma

A pt. is prescribed glycopyrrolate-rubinol 0.1mg IMThe drugs is available in a solution of 0.2mg/mL The nurse administers

0.1mg/0.2mg = 0.5mL

Oral trihexyphenidyl 4 mg is ordered. The drug is available as an elixir w/ a strength of 2mg/5mL the Nurse should administer

4/2 x 5 =10 mL

The cholinergic drug used primarily to increase urination is

Bethanechol chloride- URECHOLINE

The drug used to treat myasthenia gravis by increasing muscle strength is

NEOSTIGMINE- PROSTIGMIN

Which is NOT an effect of anticholinegic drugs

Diarhea
Cholinergic Drugs dry -up!

Atropine is frequently prescribed in all of the following situations EXCEPT

To treat urinary retention

Atropine like drugs are contraindicated in clients with

Glaucoma

A specific group of anticholinergics may be prescribed in teh early treatement of which of the following neuromuscular disorders

Parkinsonism

Which of the following is NOT and action of anticholinergics

Increased Gastrointestinal Peristalsis

Which of the following drugs is used to treat an overdose of organophosphate pesticides that cause paralysis

Pralidoxime Chloride

Advice for the CL taking anticholinergic drugs would include which of the following

Increase intake of foods and fluids high in fiber- prevents consitpation

Effects of anticholinergic are to SELECT ALL THAT APPLY

Incorrect answers
Increase Gi Motility
Constrict Pupils
CORRECT ANSWERS
INCREASE HEART RATE
DECREASE SALIVATION
DECREASE MUSCLE RIDGITY
RELAX BLADDER DETURSOR MUSCLE

Methanamine -mandelamine should not be used with sulfonamides becouse of the risk for:

Crystalluria

All of the following substances can be taken to decrease urine PH EXCEPT

Potassium Chloride

LP is 53 years old and recieving nalidxic acid(neg GRAM ) for a chronic UTI
The Dose of NegGram for long term use is

1g bid

Your health teaching plan for LP would include all of the following EXCEPT

Urine May Turn Orange

Common urinary antiseptic drug-drug interactions include ALL of the following EXCEPT that

Sodium Bicarbonate inhibits the action of methanamine

Cl taking nitrofurantoin should report which of the following to the health care provider

FEVER

A specific adverse effects from nitrofurantoin is

Peripheral Neuropathy

Urinary analgesics are used to relieve all of the following EXCEPT

Retention

A commonly prescribed urinary analgesics is

Phenazopyridine hyrdochloride-Pyridium

Nursing interventions associated with administering urinary analgesics include all of the following EXCEPT

Monitoring Blood Pressure

Urinary tract spasms are commonly treated with which of the following drugs?

Flavoxate -Urispas

The drug used to control an overactive bladder and contraindicated in cl with narrow-angle glaucoma is which of the following

Tolterodine Tartrate-Detrol

The groups of urinary antiseptics include which of the following? SELECT ALL THAT APPLY

AZTREONAM
METHANAMINE
QUINALONES
NITROFURANTION
INCORRECT
Phenothiazide

Your assessment of a cl for S/E of a fluoroquinalone would include which of the following SELECT ALL THAT APPLY

Headache and Rash
Syncope and Visual Disturbances
Peripheral Neuritis

Cl taking bethanechol should report which of the following to the health care provider SELECT ALL THAT APPLY

ABDOMINAL DISCOMFORT
INCREASED SALIVATION
URGENCY
ABDOMINAL CRAMPS
Incorrect-HEADACHE

An expected action to opiates is to

Decrease Intestinal Motility

Contraindications for the use of laxatives/cathartics include all of the following EXCEPT

Cirrhosis

RT 50 Y/O receiving prochlorperazine-compazine for nausea and vomiting.
The usual dose of compazine varies with form and includes all of the following EXCEPT

25mg Q3H PRN

The nurse assesses RT for all of the following S/E of compazine EXCEPT

EXCESSIVE SALIVATION

Specific Nursing Interventions for RT would include all of the following EXCEPT

Encourage Fluids

RT health teaching plan would include all of the following EXCEPT

Stay Busy to KEEP Mind off Nausea

Client is taking dronabinol marinol for nauseas and vomiting caused by CA chemotherapy-IT is recomended that this drug be administered

Q3-Q4 H PRN

The Usual dose of dronabinol marinol is

5-7mg

Antidiarheals are contraindicated in Cl with all of the following EXCEPT

Congestive Heart Failure

W severe diarhea, it is important to monitor all of the following EXCEPT

WBC

Which of the folllowing drugs is a somatostatin analogue frequently prescribed for metastatic cancer related to severe diarrhea

Octreotide-sandostatin

Bulk forming laxatives should be mixed in at least _____ounces of fluid

8 ounces

Use of opiate related drugs for severe diarhea may cause which of the following SELECT ALL THAT APPLY

Drowsiness
Constipation
Paralytic Ilues

The Nurse recomends that a cl w dirahea avoid which of the following SELECT ALL THAT APPLY

Fried Foods
Raw Vegtables
Milk Products

The Nurse should encourage a cl with constipation to ingest which of the following-SELECT ALL APPLY

BRAN
GRAINS
FRUIT
WATER

Your cl has a medication ordered to dilate his eyes such a medication belongs to which groups of drugs

Mydriatics

Your cl is taking a carbonic anhydrase inhibitor You will assess for which side effect

RENAL CALICULI

Which of the following drugs is most effective in treating glaucoma in african american cl.

Travaprost-Travatan

Your cl. is taking a prostaglandin analogue and you assess for the most common adverse reaction, which is

Occular Hyperemia-Excess Blood

Your Cl had her ears irrigated. To determine the results of this irrigation,visualization of which structure is required?

Tympanic Membrane

Preperations that are helpful for loosening wax from the ear canal belong to which group

Ceruminolytics

EQ is recieving isopto-eserine eye drops for treatment of glaucoma.
You would assess EQ for possible side effects, including all of the following EXCEPT

Nausea

Nursing care for EQ would include all of the following EXCEPT

Increased fluid intake

health teaching for this medication include all of the following except:

Increased Caloric Intake

Solutions commonly used to irrigate the ear include the following SELECT ALL APPLY

Burrows Solution
HYDROGEN PEROXIDE 3%
ACETIC ACID

CG 7 years old, reciving bactrim for an ear infection
NURSING CARE FOR CG INCLUDES THE FOLLOWING
SELECT ALL THAT APPLY

Obtain C&S
ASSES for hematuria/oliguira
monitor intake and output

When administering eardrops to a 7 year old child, the nurse should do the following SELECT ALL THAT APPLY

Pull down/back on the auricle
Tilt head to unaffected side
Instill medicationat room temperature

Which of the following adverse reactions would the nurse suspect in a pt reciving prolonged treatment with an AB drug

Superinfection

When administering an opthalmic solution, the drug is instilled into the _____

Lower Conjuctival sac

Which of the following instructions would be included in a teaching plan for the pt prescribed an opthalmic solution

Temporary sting/burning may be felt at the time the drug is instilled

What is the rationale for warming an otic solution that has been refrigerated before instilling the drops into the Pt's Ear.

Prevent dizziness- nausea

The actions of centrally acting muscle relaxants include which of the following

Decrease pain and increase range of motion

Side effects related to peripherally acting muscle relaxants include
SELECT ALL THAT APPLY

NAUSEA,DIPLOPIA,DROWSINESS

Which of the following is a centrally acting muscle relaxant
SELECT ALL THAT APPLY

Baclofen
Orphenadrine
cyclobenzaprine

Which of the following causes drug dependency
SELECT ALL THAT APPLY

Carisoprodol-soma
Metaxalone-skelaxin
Methocarbamol-Robaxin
Chlorzoxazone-paraflex

When administering a skeletal muscle relaxant, the nurse observes the pt for the most common adverse reaction, which is

Drowsiness

When a pt is prescribed alendronate-fosamax for osteoporosis the nurse administers

30 minutes prior to breakfast

When allopurinol-zyloprim is used for treating gout-the nurse

Encourages liberal fluid intake

What teaching points would the nurse include when educationg the pt who will begin taking risendronate-actonel

After taking the drug remain upright for at least 30 minutes

When administering a DMARD drug subQ the nurse should

rotate sites to minimize tissue irratation